Palliative care: Earlier is better

Article Type
Changed
Mon, 01/14/2019 - 11:38
Display Headline
Palliative care: Earlier is better

PRACTICE CHANGER

Recommend a palliative care consultation at the time of diagnosis. Early palliative care can improve quality of life, decrease depressive symptoms, and prolong life in patients with metastatic cancer.1

STRENGTH OF RECOMMENDATION

B: Based on a single well-done randomized controlled trial (RCT).

Temel JS, Greer JA, Muzikansky A, et al. Early palliative care for patients with metastatic non–small-cell lung cancer. N Engl J Med. 2010;363:733-742.

ILLUSTRATIVE CASE

A 73-year-old patient you’ve known for your entire career comes in for follow-up after a recent hospitalization, during which he was diagnosed with metastatic non–small-cell lung cancer. “I know things don’t look good,” he says. “I don’t want to die a miserable, painful death. But I’m not going to just roll over and die without fighting this.” What can you do to improve his quality of life while he undergoes cancer treatment?

Palliative care focuses on the prevention and treatment of pain and other debilitating effects of serious illness, with a goal of improving quality of life for patients and their families. Unlike hospice care, which requires a prognosis of less than 6 months of life to qualify for Medicare reimbursement,2 eligibility for palliative care is not dependent on prognosis. Indeed, palliative care can occur at the same time as curative or life-prolonging treatment. Palliative care programs include psychosocial and spiritual care for patient and family; management of symptoms such as pain, fatigue, shortness of breath, depression, constipation, and nausea; support for complex decisions, such as discussions of goals, do not resuscitate (DNR) orders, and requests for treatment; and coordination of care across various health care settings.3

Palliative care lowers health care spending
One study found that palliative care consultation was associated with an average savings of $1700 per admission for patients who were discharged, and $4900, on average, for every patient who died in the hospital.4 Another study demonstrated an association between states with a higher percentage of hospitals with palliative care services and fewer Medicare hospital deaths; fewer admissions to, and days in, intensive care units in the last 6 months of life; and lower total Medicare spending per enrollee.5

A 2008 systematic review of the effectiveness of palliative care revealed that there were methodological limitations in all the existing studies of palliative care, and called for higher quality studies.6 The RCT detailed here is a first step toward filling the gap in palliative care research.

STUDY SUMMARY: Intervention group lived longer and felt better

Temel et al enrolled 151 ambulatory patients with biopsy-proven non–small-cell lung cancer. The average age of the enrollees was 64 years, and slightly more than half (51.6%) were female. All had been diagnosed with metastatic cancer within 8 weeks of enrollment in the study.

The patients were randomized to receive either an early referral to the palliative care team along with standard oncology care or standard oncology care alone. Race, marital status, smoking history, presence of brain metastases, and initial cancer therapy—radiation, chemotherapy, or a combination—were similar for both groups.

The study ran for 12 weeks. Those in the intervention group had an initial meeting with a member of the palliative care team, which consisted of board-certified palliative care physicians and advanced practice nurses. Follow-up meetings with the team were scheduled at least monthly, and more frequently if requested by the patient or recommended by either the palliative care team or the oncology team—with an average of 4 meetings over the course of the study. Palliative care team members worked with patients to assess physical and emotional symptoms, coordinate care, and determine and document goals of treatment.

The primary outcome was the change in quality of life (QOL) from baseline to 12 weeks after the initial meeting with the palliative care team. QOL was measured with the Functional Assessment of Cancer Therapy-Lung (FACT-L) tool; scores range from 0 to 136, with higher scores indicating a higher QOL. The researchers used 3 subscales of the FACT-L—physical well-being, functional well-being, and a lung-cancer subscale (LCS) based on questions about 7 symptoms—to create a Trial Outcome Index (TOI), the main outcome measure. The TOI, which is the sum of the subscales, has a range of 0 to 84, with higher scores indicating higher QOL.

Secondary outcome measures were mood, use of health care services, and survival. The researchers assessed mood with 2 tools: the Patient Health Questionnaire-9 (PHQ-9) and the Hospital Anxiety and Depression Scale (HADS). The PHQ-9 is a 9-question survey that uses criteria from the Diagnostic and Statistical Manual of Psychiatric Disorders, 4th edition (DSM-IV) to diagnose depression. HADS is a 14-question survey with subscales for depression (HADS-D) and anxiety (HADS-A).

 

 

Intervention group had better scores. At study’s end, the control group had average scores of 91.5, 19.3, and 53.0 on the FACT-L, LCS, and TOI, respectively, vs 98.0, 21.0, and 59.0 for the intervention group. The palliative care group had an average increase on the TOI of 2.3 points, while the average for the control group decreased by 2.3 points (P=.04). A comparison of the mean change in scores between the 2 groups indicated statistically significant improvements in the FACT-L and TOI results for the intervention group. The improvement in LCS was not statistically significant.

The palliative care group also had a lower prevalence of depression compared with the controls (4% vs 17% on the PHQ-9 [P=.04]; 16% vs 38% on the HADS-D [P=.01]). For every 8 patients who received early palliative care, 1 less patient was diagnosed with depression. The prevalence of anxiety was not significantly different between groups.

Among patients who died during the study period, those in the palliative care group were less likely to have received aggressive end-of-life interventions compared with the controls (33% vs 54%, respectively, P=.05). Aggressive care was defined as chemotherapy within 14 days of death or little or no hospice care. Those in the early palliative care group also lived significantly longer; median survival was 11.6 months, vs 8.9 months for the control group (P=.02).

WHAT’S NEW: This study highlights the need for early referral

This is the first high-quality RCT to demonstrate improved patient outcomes when palliative care is begun close to the time of cancer diagnosis. Previous studies of late palliative care referrals did not demonstrate improved QOL or more appropriate use of health care services. This study established that patients with lung cancer are less depressed and live longer when they receive palliative care services soon after diagnosis. It also showed a link between palliative care and a reduction in aggressive, possibly inappropriate, end-of-life treatment of metastatic cancer.

Several recent practice guidelines, including that of the Institute for Clinical Systems Improvement (ICSI), recommend that palliative care referrals be made early in the course of a progressive, debilitating illness, regardless of the patient’s life expectancy.7 Other organizations, including the Institute of Medicine and the World Health Organization, recommend palliative care as an essential component of comprehensive cancer care.8 This study supports both of these recommendations.

CAVEATS: Would extra attention from any clinician work equally well?

No attempt was made to control for the extra attention (an average of 4 visits) that the palliative care team provided to those in the intervention group. Thus, it is possible that the study results could be replicated by having patients meet with their primary care physician or another health professional instead of a palliative care team.

The reduction in depression and increase in survival are clinically significant outcomes. But the improvement in QOL (an average of 7 points better on the 136-point FACT-L scale, or 6 points on the 84-point TOI scale) may not be.

It is important to note, too, that the survival benefits the researchers found may not be generalizable to other kinds of cancers. In addition, most patients (97%) in this study were white, so the findings may be less generalizable to patients of other races. Nonetheless, we think it’s likely that the improvements in QOL and mood revealed in this study would be realized by most patients with terminal cancer who received early palliative care.

CHALLENGES TO IMPLEMENTATION: Palliative care must be explained—and available

Physicians must be able to explain to their patients the difference between palliative care and hospice—most notably, that patients can continue to receive anticancer treatment while receiving palliative care. The recommendation to seek palliative care should not be considered “giving up” on the patient.

In order to refer patients to palliative care early in the course of cancer care, physicians must have access to a palliative care team, which may not be available in all cases. In 2006, only 53% of hospitals with more than 50 beds reported having a palliative care program.5 If there is no such program available, physicians can refer to the ICSI guideline on palliative care for more information on how to implement elements of palliative care for their patients with advanced cancer.7

References

1. Temel JS, Greer JA, Muzikansky A, et al. Early palliative care for patients with metastatic non–small-cell lung cancer. N Engl J Med. 2010;363:733-742.

2. Centers for Medicare and Medicaid Services. Medicare hospice benefits. Available at: http://www.medicare.gov/publications/pubs/pdf/02154.pdf. Accessed September 30, 2010.

3. Center to Advance Palliative Care. The case for hospital palliative care: improving quality, reducing cost. Available at: http://www.capc.org/building-a-hospital-based-palliative-care-program/case/support-from-capc/capc_publications/making-the-case.pdf. Accessed September 30, 2010.

4. Morrison RS, Penrod JD, Cassel JB, et al. Cost savings associated with US hospital palliative care consultation programs. Arch Intern Med. 2008;168:1783-1790.

5. Goldsmith B, Dietrich J, Du Q, et al. Variability in access to hospital palliative care in the United States. J Palliat Med. 2008;11:1094-1102.

6. Zimmermann C, Riechelmann R. Effectiveness of specialized palliative care: a systematic review. JAMA. 2008;299:1698-1709.

7. Institute for Clinical Systems Improvement (ICSI) 2009 palliative care guideline. Available at: http://www.icsi.org/guidelines_and_more/gl_os_prot/other_health_care_conditions/palliative_care/palliative_care_11875.html. Accessed September 30, 2010.

8. Ferris FD, Bruera E, Cherny N, et al. Palliative cancer care a decade later: accomplishments, the need, next steps—from the American Society of Clinical Oncology. J Clin Oncol. 2009;27:3052-3058.

Article PDF
Author and Disclosure Information

Kate Rowland, MD
Sarah-Anne Schumann, MD
Department of Family Medicine, University of Chicago

PURLs EDITOR
John Hickner, MD, MSc
Cleveland Clinic

Issue
The Journal of Family Practice - 59(12)
Publications
Topics
Page Number
695-698
Legacy Keywords
Kate Rowland; palliative care; PURLs; metastatic cancer; early palliative care; palliative care consultation; quality of life
Sections
Author and Disclosure Information

Kate Rowland, MD
Sarah-Anne Schumann, MD
Department of Family Medicine, University of Chicago

PURLs EDITOR
John Hickner, MD, MSc
Cleveland Clinic

Author and Disclosure Information

Kate Rowland, MD
Sarah-Anne Schumann, MD
Department of Family Medicine, University of Chicago

PURLs EDITOR
John Hickner, MD, MSc
Cleveland Clinic

Article PDF
Article PDF

PRACTICE CHANGER

Recommend a palliative care consultation at the time of diagnosis. Early palliative care can improve quality of life, decrease depressive symptoms, and prolong life in patients with metastatic cancer.1

STRENGTH OF RECOMMENDATION

B: Based on a single well-done randomized controlled trial (RCT).

Temel JS, Greer JA, Muzikansky A, et al. Early palliative care for patients with metastatic non–small-cell lung cancer. N Engl J Med. 2010;363:733-742.

ILLUSTRATIVE CASE

A 73-year-old patient you’ve known for your entire career comes in for follow-up after a recent hospitalization, during which he was diagnosed with metastatic non–small-cell lung cancer. “I know things don’t look good,” he says. “I don’t want to die a miserable, painful death. But I’m not going to just roll over and die without fighting this.” What can you do to improve his quality of life while he undergoes cancer treatment?

Palliative care focuses on the prevention and treatment of pain and other debilitating effects of serious illness, with a goal of improving quality of life for patients and their families. Unlike hospice care, which requires a prognosis of less than 6 months of life to qualify for Medicare reimbursement,2 eligibility for palliative care is not dependent on prognosis. Indeed, palliative care can occur at the same time as curative or life-prolonging treatment. Palliative care programs include psychosocial and spiritual care for patient and family; management of symptoms such as pain, fatigue, shortness of breath, depression, constipation, and nausea; support for complex decisions, such as discussions of goals, do not resuscitate (DNR) orders, and requests for treatment; and coordination of care across various health care settings.3

Palliative care lowers health care spending
One study found that palliative care consultation was associated with an average savings of $1700 per admission for patients who were discharged, and $4900, on average, for every patient who died in the hospital.4 Another study demonstrated an association between states with a higher percentage of hospitals with palliative care services and fewer Medicare hospital deaths; fewer admissions to, and days in, intensive care units in the last 6 months of life; and lower total Medicare spending per enrollee.5

A 2008 systematic review of the effectiveness of palliative care revealed that there were methodological limitations in all the existing studies of palliative care, and called for higher quality studies.6 The RCT detailed here is a first step toward filling the gap in palliative care research.

STUDY SUMMARY: Intervention group lived longer and felt better

Temel et al enrolled 151 ambulatory patients with biopsy-proven non–small-cell lung cancer. The average age of the enrollees was 64 years, and slightly more than half (51.6%) were female. All had been diagnosed with metastatic cancer within 8 weeks of enrollment in the study.

The patients were randomized to receive either an early referral to the palliative care team along with standard oncology care or standard oncology care alone. Race, marital status, smoking history, presence of brain metastases, and initial cancer therapy—radiation, chemotherapy, or a combination—were similar for both groups.

The study ran for 12 weeks. Those in the intervention group had an initial meeting with a member of the palliative care team, which consisted of board-certified palliative care physicians and advanced practice nurses. Follow-up meetings with the team were scheduled at least monthly, and more frequently if requested by the patient or recommended by either the palliative care team or the oncology team—with an average of 4 meetings over the course of the study. Palliative care team members worked with patients to assess physical and emotional symptoms, coordinate care, and determine and document goals of treatment.

The primary outcome was the change in quality of life (QOL) from baseline to 12 weeks after the initial meeting with the palliative care team. QOL was measured with the Functional Assessment of Cancer Therapy-Lung (FACT-L) tool; scores range from 0 to 136, with higher scores indicating a higher QOL. The researchers used 3 subscales of the FACT-L—physical well-being, functional well-being, and a lung-cancer subscale (LCS) based on questions about 7 symptoms—to create a Trial Outcome Index (TOI), the main outcome measure. The TOI, which is the sum of the subscales, has a range of 0 to 84, with higher scores indicating higher QOL.

Secondary outcome measures were mood, use of health care services, and survival. The researchers assessed mood with 2 tools: the Patient Health Questionnaire-9 (PHQ-9) and the Hospital Anxiety and Depression Scale (HADS). The PHQ-9 is a 9-question survey that uses criteria from the Diagnostic and Statistical Manual of Psychiatric Disorders, 4th edition (DSM-IV) to diagnose depression. HADS is a 14-question survey with subscales for depression (HADS-D) and anxiety (HADS-A).

 

 

Intervention group had better scores. At study’s end, the control group had average scores of 91.5, 19.3, and 53.0 on the FACT-L, LCS, and TOI, respectively, vs 98.0, 21.0, and 59.0 for the intervention group. The palliative care group had an average increase on the TOI of 2.3 points, while the average for the control group decreased by 2.3 points (P=.04). A comparison of the mean change in scores between the 2 groups indicated statistically significant improvements in the FACT-L and TOI results for the intervention group. The improvement in LCS was not statistically significant.

The palliative care group also had a lower prevalence of depression compared with the controls (4% vs 17% on the PHQ-9 [P=.04]; 16% vs 38% on the HADS-D [P=.01]). For every 8 patients who received early palliative care, 1 less patient was diagnosed with depression. The prevalence of anxiety was not significantly different between groups.

Among patients who died during the study period, those in the palliative care group were less likely to have received aggressive end-of-life interventions compared with the controls (33% vs 54%, respectively, P=.05). Aggressive care was defined as chemotherapy within 14 days of death or little or no hospice care. Those in the early palliative care group also lived significantly longer; median survival was 11.6 months, vs 8.9 months for the control group (P=.02).

WHAT’S NEW: This study highlights the need for early referral

This is the first high-quality RCT to demonstrate improved patient outcomes when palliative care is begun close to the time of cancer diagnosis. Previous studies of late palliative care referrals did not demonstrate improved QOL or more appropriate use of health care services. This study established that patients with lung cancer are less depressed and live longer when they receive palliative care services soon after diagnosis. It also showed a link between palliative care and a reduction in aggressive, possibly inappropriate, end-of-life treatment of metastatic cancer.

Several recent practice guidelines, including that of the Institute for Clinical Systems Improvement (ICSI), recommend that palliative care referrals be made early in the course of a progressive, debilitating illness, regardless of the patient’s life expectancy.7 Other organizations, including the Institute of Medicine and the World Health Organization, recommend palliative care as an essential component of comprehensive cancer care.8 This study supports both of these recommendations.

CAVEATS: Would extra attention from any clinician work equally well?

No attempt was made to control for the extra attention (an average of 4 visits) that the palliative care team provided to those in the intervention group. Thus, it is possible that the study results could be replicated by having patients meet with their primary care physician or another health professional instead of a palliative care team.

The reduction in depression and increase in survival are clinically significant outcomes. But the improvement in QOL (an average of 7 points better on the 136-point FACT-L scale, or 6 points on the 84-point TOI scale) may not be.

It is important to note, too, that the survival benefits the researchers found may not be generalizable to other kinds of cancers. In addition, most patients (97%) in this study were white, so the findings may be less generalizable to patients of other races. Nonetheless, we think it’s likely that the improvements in QOL and mood revealed in this study would be realized by most patients with terminal cancer who received early palliative care.

CHALLENGES TO IMPLEMENTATION: Palliative care must be explained—and available

Physicians must be able to explain to their patients the difference between palliative care and hospice—most notably, that patients can continue to receive anticancer treatment while receiving palliative care. The recommendation to seek palliative care should not be considered “giving up” on the patient.

In order to refer patients to palliative care early in the course of cancer care, physicians must have access to a palliative care team, which may not be available in all cases. In 2006, only 53% of hospitals with more than 50 beds reported having a palliative care program.5 If there is no such program available, physicians can refer to the ICSI guideline on palliative care for more information on how to implement elements of palliative care for their patients with advanced cancer.7

PRACTICE CHANGER

Recommend a palliative care consultation at the time of diagnosis. Early palliative care can improve quality of life, decrease depressive symptoms, and prolong life in patients with metastatic cancer.1

STRENGTH OF RECOMMENDATION

B: Based on a single well-done randomized controlled trial (RCT).

Temel JS, Greer JA, Muzikansky A, et al. Early palliative care for patients with metastatic non–small-cell lung cancer. N Engl J Med. 2010;363:733-742.

ILLUSTRATIVE CASE

A 73-year-old patient you’ve known for your entire career comes in for follow-up after a recent hospitalization, during which he was diagnosed with metastatic non–small-cell lung cancer. “I know things don’t look good,” he says. “I don’t want to die a miserable, painful death. But I’m not going to just roll over and die without fighting this.” What can you do to improve his quality of life while he undergoes cancer treatment?

Palliative care focuses on the prevention and treatment of pain and other debilitating effects of serious illness, with a goal of improving quality of life for patients and their families. Unlike hospice care, which requires a prognosis of less than 6 months of life to qualify for Medicare reimbursement,2 eligibility for palliative care is not dependent on prognosis. Indeed, palliative care can occur at the same time as curative or life-prolonging treatment. Palliative care programs include psychosocial and spiritual care for patient and family; management of symptoms such as pain, fatigue, shortness of breath, depression, constipation, and nausea; support for complex decisions, such as discussions of goals, do not resuscitate (DNR) orders, and requests for treatment; and coordination of care across various health care settings.3

Palliative care lowers health care spending
One study found that palliative care consultation was associated with an average savings of $1700 per admission for patients who were discharged, and $4900, on average, for every patient who died in the hospital.4 Another study demonstrated an association between states with a higher percentage of hospitals with palliative care services and fewer Medicare hospital deaths; fewer admissions to, and days in, intensive care units in the last 6 months of life; and lower total Medicare spending per enrollee.5

A 2008 systematic review of the effectiveness of palliative care revealed that there were methodological limitations in all the existing studies of palliative care, and called for higher quality studies.6 The RCT detailed here is a first step toward filling the gap in palliative care research.

STUDY SUMMARY: Intervention group lived longer and felt better

Temel et al enrolled 151 ambulatory patients with biopsy-proven non–small-cell lung cancer. The average age of the enrollees was 64 years, and slightly more than half (51.6%) were female. All had been diagnosed with metastatic cancer within 8 weeks of enrollment in the study.

The patients were randomized to receive either an early referral to the palliative care team along with standard oncology care or standard oncology care alone. Race, marital status, smoking history, presence of brain metastases, and initial cancer therapy—radiation, chemotherapy, or a combination—were similar for both groups.

The study ran for 12 weeks. Those in the intervention group had an initial meeting with a member of the palliative care team, which consisted of board-certified palliative care physicians and advanced practice nurses. Follow-up meetings with the team were scheduled at least monthly, and more frequently if requested by the patient or recommended by either the palliative care team or the oncology team—with an average of 4 meetings over the course of the study. Palliative care team members worked with patients to assess physical and emotional symptoms, coordinate care, and determine and document goals of treatment.

The primary outcome was the change in quality of life (QOL) from baseline to 12 weeks after the initial meeting with the palliative care team. QOL was measured with the Functional Assessment of Cancer Therapy-Lung (FACT-L) tool; scores range from 0 to 136, with higher scores indicating a higher QOL. The researchers used 3 subscales of the FACT-L—physical well-being, functional well-being, and a lung-cancer subscale (LCS) based on questions about 7 symptoms—to create a Trial Outcome Index (TOI), the main outcome measure. The TOI, which is the sum of the subscales, has a range of 0 to 84, with higher scores indicating higher QOL.

Secondary outcome measures were mood, use of health care services, and survival. The researchers assessed mood with 2 tools: the Patient Health Questionnaire-9 (PHQ-9) and the Hospital Anxiety and Depression Scale (HADS). The PHQ-9 is a 9-question survey that uses criteria from the Diagnostic and Statistical Manual of Psychiatric Disorders, 4th edition (DSM-IV) to diagnose depression. HADS is a 14-question survey with subscales for depression (HADS-D) and anxiety (HADS-A).

 

 

Intervention group had better scores. At study’s end, the control group had average scores of 91.5, 19.3, and 53.0 on the FACT-L, LCS, and TOI, respectively, vs 98.0, 21.0, and 59.0 for the intervention group. The palliative care group had an average increase on the TOI of 2.3 points, while the average for the control group decreased by 2.3 points (P=.04). A comparison of the mean change in scores between the 2 groups indicated statistically significant improvements in the FACT-L and TOI results for the intervention group. The improvement in LCS was not statistically significant.

The palliative care group also had a lower prevalence of depression compared with the controls (4% vs 17% on the PHQ-9 [P=.04]; 16% vs 38% on the HADS-D [P=.01]). For every 8 patients who received early palliative care, 1 less patient was diagnosed with depression. The prevalence of anxiety was not significantly different between groups.

Among patients who died during the study period, those in the palliative care group were less likely to have received aggressive end-of-life interventions compared with the controls (33% vs 54%, respectively, P=.05). Aggressive care was defined as chemotherapy within 14 days of death or little or no hospice care. Those in the early palliative care group also lived significantly longer; median survival was 11.6 months, vs 8.9 months for the control group (P=.02).

WHAT’S NEW: This study highlights the need for early referral

This is the first high-quality RCT to demonstrate improved patient outcomes when palliative care is begun close to the time of cancer diagnosis. Previous studies of late palliative care referrals did not demonstrate improved QOL or more appropriate use of health care services. This study established that patients with lung cancer are less depressed and live longer when they receive palliative care services soon after diagnosis. It also showed a link between palliative care and a reduction in aggressive, possibly inappropriate, end-of-life treatment of metastatic cancer.

Several recent practice guidelines, including that of the Institute for Clinical Systems Improvement (ICSI), recommend that palliative care referrals be made early in the course of a progressive, debilitating illness, regardless of the patient’s life expectancy.7 Other organizations, including the Institute of Medicine and the World Health Organization, recommend palliative care as an essential component of comprehensive cancer care.8 This study supports both of these recommendations.

CAVEATS: Would extra attention from any clinician work equally well?

No attempt was made to control for the extra attention (an average of 4 visits) that the palliative care team provided to those in the intervention group. Thus, it is possible that the study results could be replicated by having patients meet with their primary care physician or another health professional instead of a palliative care team.

The reduction in depression and increase in survival are clinically significant outcomes. But the improvement in QOL (an average of 7 points better on the 136-point FACT-L scale, or 6 points on the 84-point TOI scale) may not be.

It is important to note, too, that the survival benefits the researchers found may not be generalizable to other kinds of cancers. In addition, most patients (97%) in this study were white, so the findings may be less generalizable to patients of other races. Nonetheless, we think it’s likely that the improvements in QOL and mood revealed in this study would be realized by most patients with terminal cancer who received early palliative care.

CHALLENGES TO IMPLEMENTATION: Palliative care must be explained—and available

Physicians must be able to explain to their patients the difference between palliative care and hospice—most notably, that patients can continue to receive anticancer treatment while receiving palliative care. The recommendation to seek palliative care should not be considered “giving up” on the patient.

In order to refer patients to palliative care early in the course of cancer care, physicians must have access to a palliative care team, which may not be available in all cases. In 2006, only 53% of hospitals with more than 50 beds reported having a palliative care program.5 If there is no such program available, physicians can refer to the ICSI guideline on palliative care for more information on how to implement elements of palliative care for their patients with advanced cancer.7

References

1. Temel JS, Greer JA, Muzikansky A, et al. Early palliative care for patients with metastatic non–small-cell lung cancer. N Engl J Med. 2010;363:733-742.

2. Centers for Medicare and Medicaid Services. Medicare hospice benefits. Available at: http://www.medicare.gov/publications/pubs/pdf/02154.pdf. Accessed September 30, 2010.

3. Center to Advance Palliative Care. The case for hospital palliative care: improving quality, reducing cost. Available at: http://www.capc.org/building-a-hospital-based-palliative-care-program/case/support-from-capc/capc_publications/making-the-case.pdf. Accessed September 30, 2010.

4. Morrison RS, Penrod JD, Cassel JB, et al. Cost savings associated with US hospital palliative care consultation programs. Arch Intern Med. 2008;168:1783-1790.

5. Goldsmith B, Dietrich J, Du Q, et al. Variability in access to hospital palliative care in the United States. J Palliat Med. 2008;11:1094-1102.

6. Zimmermann C, Riechelmann R. Effectiveness of specialized palliative care: a systematic review. JAMA. 2008;299:1698-1709.

7. Institute for Clinical Systems Improvement (ICSI) 2009 palliative care guideline. Available at: http://www.icsi.org/guidelines_and_more/gl_os_prot/other_health_care_conditions/palliative_care/palliative_care_11875.html. Accessed September 30, 2010.

8. Ferris FD, Bruera E, Cherny N, et al. Palliative cancer care a decade later: accomplishments, the need, next steps—from the American Society of Clinical Oncology. J Clin Oncol. 2009;27:3052-3058.

References

1. Temel JS, Greer JA, Muzikansky A, et al. Early palliative care for patients with metastatic non–small-cell lung cancer. N Engl J Med. 2010;363:733-742.

2. Centers for Medicare and Medicaid Services. Medicare hospice benefits. Available at: http://www.medicare.gov/publications/pubs/pdf/02154.pdf. Accessed September 30, 2010.

3. Center to Advance Palliative Care. The case for hospital palliative care: improving quality, reducing cost. Available at: http://www.capc.org/building-a-hospital-based-palliative-care-program/case/support-from-capc/capc_publications/making-the-case.pdf. Accessed September 30, 2010.

4. Morrison RS, Penrod JD, Cassel JB, et al. Cost savings associated with US hospital palliative care consultation programs. Arch Intern Med. 2008;168:1783-1790.

5. Goldsmith B, Dietrich J, Du Q, et al. Variability in access to hospital palliative care in the United States. J Palliat Med. 2008;11:1094-1102.

6. Zimmermann C, Riechelmann R. Effectiveness of specialized palliative care: a systematic review. JAMA. 2008;299:1698-1709.

7. Institute for Clinical Systems Improvement (ICSI) 2009 palliative care guideline. Available at: http://www.icsi.org/guidelines_and_more/gl_os_prot/other_health_care_conditions/palliative_care/palliative_care_11875.html. Accessed September 30, 2010.

8. Ferris FD, Bruera E, Cherny N, et al. Palliative cancer care a decade later: accomplishments, the need, next steps—from the American Society of Clinical Oncology. J Clin Oncol. 2009;27:3052-3058.

Issue
The Journal of Family Practice - 59(12)
Issue
The Journal of Family Practice - 59(12)
Page Number
695-698
Page Number
695-698
Publications
Publications
Topics
Article Type
Display Headline
Palliative care: Earlier is better
Display Headline
Palliative care: Earlier is better
Legacy Keywords
Kate Rowland; palliative care; PURLs; metastatic cancer; early palliative care; palliative care consultation; quality of life
Legacy Keywords
Kate Rowland; palliative care; PURLs; metastatic cancer; early palliative care; palliative care consultation; quality of life
Sections
Article Source

PURLs Copyright

Copyright © 2010 The Family Physicians Inquiries Network.
All rights reserved.

Inside the Article

Article PDF Media

Glucose control: How low should you go with the critically ill?

Article Type
Changed
Tue, 05/03/2022 - 16:03
Display Headline
Glucose control: How low should you go with the critically ill?
Practice changer

For hyperglycemic patients admitted to an intensive care unit (ICU), the target blood glucose level should be ≤180 mg/dL, not 81 to 108 mg/dL. More aggressive glucose lowering is associated with a higher mortality rate.1

Strength of recommendation

B: Based on a single, high-quality randomized clinical trial.

Finfer S, Chittock DR, Su SY, et al; NICE-SUGAR Study Investigators. Intensive versus conventional glucose control in critically ill patients. N Engl J Med. 2009;360:1283-1297.

 

ILLUSTRATIVE CASE

A 71-year-old woman with diabetes and coronary artery disease has just been admitted to the ICU, where she’ll receive treatment for sepsis, multilobar pneumonia, and respiratory failure requiring mechanical ventilation. Her blood sugar is 253 mg/dL. In writing her admission orders, you contemplate targets for glycemic control. How low should you go?

Hyperglycemia is common in patients admitted to intensive care, whether or not they have diabetes. Elevated blood sugar is associated with stress and trauma and affects both postoperative and critically ill medical patients. A wealth of evidence has demonstrated that hyperglycemia is associated with poorer outcomes and increased mortality in this patient population, including those with myocardial infarction, stroke, trauma, and other medical conditions.2-5 Thus, intensive glucose control is the standard of care in the ICU, based on consensus guidelines from such groups as the American Diabetes Association (ADA) and the Surviving Sepsis Campaign—an initiative developed by 3 critical care organizations and endorsed by 16 specialty groups.6-8

Is intense therapy better? Study results differ
The association between hyperglycemia and an increased risk of death led investigators to study the effectiveness of aggressive treatment with insulin in decreasing morbidity and mortality. A 2004 meta-analysis of 35 trials comparing insulin vs no insulin in critically ill hospitalized patients demonstrated a 15% reduction in short-term mortality among patients treated with insulin.9 A 2008 meta-analysis of 29 randomized trials, including data from 8432 adult ICU patients, compared intensive insulin therapy with conventional therapy—and found that intensive therapy did not lower hospital mortality rates compared with conventional therapy. In addition, this meta-analysis revealed a marked increase in severe hypoglycemia (blood sugar ≤40 mg/dL) in the intensive therapy group.10 (The intensive therapy group included studies with glucose goals of ≤110 mg/dL and <150 mg/dL in about equal numbers; conventional therapy goals were generally between 180 and 200 mg/dL.)

The studies included in both the meta-analyses, however, were mostly small, single-center trials, and of low-to-medium quality. In addition, methods for achieving glycemic control varied. Nonetheless, current consensus guidelines set a goal for glucose levels of 80 to 110 mg/dL for all critically ill hospitalized patients.6-8 But because of the lack of sufficient high-quality evidence from a single large RCT, Finfer et al conducted the large study described here to clearly establish that intensive glycemic control decreases all-cause mortality. Given their hypothesis, the results were surprising.

STUDY SUMMARY: Intensive therapy does more harm than help

NICE-SUGAR (Normoglycaemia in Intensive Care Evaluation-Survival Using Glucose Algorithm Regulation) was a large-scale, multicenter, multinational trial comparing aggressive blood sugar control (goal 81-108 mg/dL) with conventional therapy (goal ≤180 mg/dL) in 6104 critically ill hospitalized patients with hyperglycemia. Patients were followed for 90 days. The primary end point was death from any cause 90 days after randomization. Secondary outcomes included survival time during the first 90 days, specific cause of death, duration of mechanical ventilation, renal replacement therapy, and length of stays in the ICU and in the hospital. Other outcomes included death from any cause within 28 days, place of death, new organ failure, positive blood culture, blood transfusion, and units of blood transfused.

The study was conducted in 42 hospitals in Canada, Australia, and New Zealand. Patients had to have an anticipated ICU admission of 3 days or more and randomization had to occur within 24 hours of admission. The study protocol was discontinued when patients began eating or were discharged from the ICU; if they were readmitted to the ICU within 90 days of randomization, the study protocol was resumed.

Treatment assignment was revealed to clinical staff after randomization, and was determined by a specific algorithm ( https://studies.thegeorgeinstitute.org/nice/ ). Blood sugar levels were managed with insulin infusions.

In the conventional group, insulin was started at 1 unit/h for glucose levels >180 mg/dL, and decreased or stopped when levels were <144 mg/dL, depending on previous glucose value and current rate of drip. In the intensive therapy group, insulin was initiated for lower levels (blood sugar >109 mg/dL) and at a higher rate (2 units/h). The insulin rate was decreased or maintained for glucose levels from 64 to 80 mg/dL, depending on previous glucose value and current rate of drip. Insulin was withheld for blood sugar levels of <64 mg/dL.

Contrary to the hypothesis, intensive therapy spelled trouble. Patients with intensive glycemic control had an all-cause mortality rate of 27.5%, compared with a rate of 24.9% for patients in the conventional therapy group (P=.04, number needed to harm [NNH]=38). Severe hypoglycemia (glucose ≤40 mg/dL) occurred in 6.8% of those in the intensive therapy group, compared with 0.5% in the conventional therapy group (P=.03, NNH=16).

Most of the deaths in both groups occurred in the ICU or in the hospital. Deaths from cardiovascular causes were more common among those in the intensive therapy group. There were no significant differences in any other outcomes. The mean glucose level in the intensive therapy group was 118, vs 145 mg/dL in the conventional therapy group.

For multivariate and subgroup analyses, the patients were assigned strata (Canada or Australia/New Zealand; operative vs nonoperative admission) or classified into groups (traumatic vs atraumatic; diabetes vs no diabetes; corticosteroids in previous 72 hours or not; high vs low critical illness symptom severity) based on predefined characteristics. No subgroups had significantly improved outcomes with intensive therapy.1

 

 

 

WHAT’S NEW: Now we know: Don’t go too low

This study, in contrast to a number of smaller studies of lower quality, demonstrates a higher all-cause mortality rate at 90 days for critically ill patients receiving intensive glucose therapy. It is now clear that, among critically ill hospitalized patients, aiming for intensive glucose control (81-108 mg/dL) is associated with an increased rate of severe hypoglycemic events and all-cause mortality at 90 days. The previously used goal of conventional therapy (≤180 mg/dL) is safer.

CAVEATS: Study population may not reflect primary care

There are 2 caveats to this study. The first is that because of the nature of the research, it was impossible to maintain blinding of the clinical staff to patient assignments. The second important caveat pertains to the severity of illness among participants in this multicenter study: Most of these patients were in ICUs at tertiary care medical centers and had an expected ICU length of stay of 3 or more days. Although many family physicians manage patients in ICUs, the patients randomized in this study may represent a sicker than average patient population for some hospitals.

CHALLENGES TO IMPLEMENTATION: Some may doubt validity of this outcome

Less aggressive glycemic control for critically ill patients should be easier to achieve, not more difficult. However, a change in glucose targets may require new admission order sets and, notably, reeducation of physicians and nurses who have been convinced by earlier studies that more intensive glucose control is superior.

Acknowledgments

The PURLs Surveillance System is supported in part by Grant Number UL1RR024999 from the National Center for Research Resources, a Clinical Translational Science Award to the University of Chicago. The content is solely the responsibility of the authors and does not necessarily represent the official views of the National Center for Research Resources or the National Institutes of Health.

PURLs methodology

This study was selected and evaluated using FPIN’s Priority Updates from the Research Literature (PURL) Surveillance System methodology. The criteria and findings leading to the selection of this study as a PURL can be accessed at www.jfponline.com/purls.

Files
References

1. Finfer S, Chittock DR, Su SY, et al. NICE-SUGAR Study Investigators. Intensive versus conventional glucose control in critically ill patients. N Engl J Med. 2009;360:1283-1297.

2. Capes SE, Hunt D, Malmberg K, et al. Stress hyperglycaemia and increased risk of death after myocardial infarction in patients with and without diabetes: a systematic overview. Lancet. 2000;355:773-778.

3. Capes SE, Hunt D, Malmberg K, et al. Stress hyperglycemia and prognosis of stroke in nondiabetic and diabetic patients: a systematic overview. Stroke. 2001;32:2426-2432.

4. Gale SC, Sicoutris C, Reilly PM, et al. Poor glycemic control is associated with increased mortality in critically ill trauma patients. Am Surg. 2007;73:454-460.

5. Krinsley JS. Association between hyperglycemia and increased hospital mortality in a heterogeneous population of critically ill patients. Mayo Clin Proc. 2003;78:1471-1478.

6. Standards of medical care in diabetes—2008. Diabetes Care. 2008;31(suppl 1):S12-S54.

7. Rodbard HW, Blonde L, Braithwaite SS, et al. American Association of Clinical Endocrinologists medical guidelines for clinical practice for the management of diabetes mellitus. Endocr Pract. 2007;13(suppl 1):1-68.

8. Dellinger RP, Levy MM, Carlet JM, et al. Surviving Sepsis Campaign: international guidelines for management of severe sepsis and septic shock: 2008. Crit Care Med. 2008;36:296-327.

9. Pittas AG, Siegel RD, Lau J. Insulin therapy for critically ill hospitalized patients: a meta-analysis of randomized controlled trials. Arch Intern Med. 2004;164:2005-2011.

10. Wiener RS, Wiener DC, Larson RJ. Benefits and risks of tight glucose control in critically ill adults: a meta-analysis. JAMA. 2008;300:933-944.

Article PDF
Author and Disclosure Information

Adam J. Zolotor, MD, MPH
Department of Family Medicine, University of North Carolina, Chapel Hill

Sarah-Anne Schumann, MD;
Lisa Vargish, MD, MS
Department of Family Medicine, The University of Chicago

PURLs EDITOR
John Hickner, MD, MSc
Department of Family Medicine, Cleveland Clinic
The University of Chicago

Issue
The Journal of Family Practice - 58(8)
Publications
Topics
Page Number
424-426
Sections
Files
Files
Author and Disclosure Information

Adam J. Zolotor, MD, MPH
Department of Family Medicine, University of North Carolina, Chapel Hill

Sarah-Anne Schumann, MD;
Lisa Vargish, MD, MS
Department of Family Medicine, The University of Chicago

PURLs EDITOR
John Hickner, MD, MSc
Department of Family Medicine, Cleveland Clinic
The University of Chicago

Author and Disclosure Information

Adam J. Zolotor, MD, MPH
Department of Family Medicine, University of North Carolina, Chapel Hill

Sarah-Anne Schumann, MD;
Lisa Vargish, MD, MS
Department of Family Medicine, The University of Chicago

PURLs EDITOR
John Hickner, MD, MSc
Department of Family Medicine, Cleveland Clinic
The University of Chicago

Article PDF
Article PDF
Practice changer

For hyperglycemic patients admitted to an intensive care unit (ICU), the target blood glucose level should be ≤180 mg/dL, not 81 to 108 mg/dL. More aggressive glucose lowering is associated with a higher mortality rate.1

Strength of recommendation

B: Based on a single, high-quality randomized clinical trial.

Finfer S, Chittock DR, Su SY, et al; NICE-SUGAR Study Investigators. Intensive versus conventional glucose control in critically ill patients. N Engl J Med. 2009;360:1283-1297.

 

ILLUSTRATIVE CASE

A 71-year-old woman with diabetes and coronary artery disease has just been admitted to the ICU, where she’ll receive treatment for sepsis, multilobar pneumonia, and respiratory failure requiring mechanical ventilation. Her blood sugar is 253 mg/dL. In writing her admission orders, you contemplate targets for glycemic control. How low should you go?

Hyperglycemia is common in patients admitted to intensive care, whether or not they have diabetes. Elevated blood sugar is associated with stress and trauma and affects both postoperative and critically ill medical patients. A wealth of evidence has demonstrated that hyperglycemia is associated with poorer outcomes and increased mortality in this patient population, including those with myocardial infarction, stroke, trauma, and other medical conditions.2-5 Thus, intensive glucose control is the standard of care in the ICU, based on consensus guidelines from such groups as the American Diabetes Association (ADA) and the Surviving Sepsis Campaign—an initiative developed by 3 critical care organizations and endorsed by 16 specialty groups.6-8

Is intense therapy better? Study results differ
The association between hyperglycemia and an increased risk of death led investigators to study the effectiveness of aggressive treatment with insulin in decreasing morbidity and mortality. A 2004 meta-analysis of 35 trials comparing insulin vs no insulin in critically ill hospitalized patients demonstrated a 15% reduction in short-term mortality among patients treated with insulin.9 A 2008 meta-analysis of 29 randomized trials, including data from 8432 adult ICU patients, compared intensive insulin therapy with conventional therapy—and found that intensive therapy did not lower hospital mortality rates compared with conventional therapy. In addition, this meta-analysis revealed a marked increase in severe hypoglycemia (blood sugar ≤40 mg/dL) in the intensive therapy group.10 (The intensive therapy group included studies with glucose goals of ≤110 mg/dL and <150 mg/dL in about equal numbers; conventional therapy goals were generally between 180 and 200 mg/dL.)

The studies included in both the meta-analyses, however, were mostly small, single-center trials, and of low-to-medium quality. In addition, methods for achieving glycemic control varied. Nonetheless, current consensus guidelines set a goal for glucose levels of 80 to 110 mg/dL for all critically ill hospitalized patients.6-8 But because of the lack of sufficient high-quality evidence from a single large RCT, Finfer et al conducted the large study described here to clearly establish that intensive glycemic control decreases all-cause mortality. Given their hypothesis, the results were surprising.

STUDY SUMMARY: Intensive therapy does more harm than help

NICE-SUGAR (Normoglycaemia in Intensive Care Evaluation-Survival Using Glucose Algorithm Regulation) was a large-scale, multicenter, multinational trial comparing aggressive blood sugar control (goal 81-108 mg/dL) with conventional therapy (goal ≤180 mg/dL) in 6104 critically ill hospitalized patients with hyperglycemia. Patients were followed for 90 days. The primary end point was death from any cause 90 days after randomization. Secondary outcomes included survival time during the first 90 days, specific cause of death, duration of mechanical ventilation, renal replacement therapy, and length of stays in the ICU and in the hospital. Other outcomes included death from any cause within 28 days, place of death, new organ failure, positive blood culture, blood transfusion, and units of blood transfused.

The study was conducted in 42 hospitals in Canada, Australia, and New Zealand. Patients had to have an anticipated ICU admission of 3 days or more and randomization had to occur within 24 hours of admission. The study protocol was discontinued when patients began eating or were discharged from the ICU; if they were readmitted to the ICU within 90 days of randomization, the study protocol was resumed.

Treatment assignment was revealed to clinical staff after randomization, and was determined by a specific algorithm ( https://studies.thegeorgeinstitute.org/nice/ ). Blood sugar levels were managed with insulin infusions.

In the conventional group, insulin was started at 1 unit/h for glucose levels >180 mg/dL, and decreased or stopped when levels were <144 mg/dL, depending on previous glucose value and current rate of drip. In the intensive therapy group, insulin was initiated for lower levels (blood sugar >109 mg/dL) and at a higher rate (2 units/h). The insulin rate was decreased or maintained for glucose levels from 64 to 80 mg/dL, depending on previous glucose value and current rate of drip. Insulin was withheld for blood sugar levels of <64 mg/dL.

Contrary to the hypothesis, intensive therapy spelled trouble. Patients with intensive glycemic control had an all-cause mortality rate of 27.5%, compared with a rate of 24.9% for patients in the conventional therapy group (P=.04, number needed to harm [NNH]=38). Severe hypoglycemia (glucose ≤40 mg/dL) occurred in 6.8% of those in the intensive therapy group, compared with 0.5% in the conventional therapy group (P=.03, NNH=16).

Most of the deaths in both groups occurred in the ICU or in the hospital. Deaths from cardiovascular causes were more common among those in the intensive therapy group. There were no significant differences in any other outcomes. The mean glucose level in the intensive therapy group was 118, vs 145 mg/dL in the conventional therapy group.

For multivariate and subgroup analyses, the patients were assigned strata (Canada or Australia/New Zealand; operative vs nonoperative admission) or classified into groups (traumatic vs atraumatic; diabetes vs no diabetes; corticosteroids in previous 72 hours or not; high vs low critical illness symptom severity) based on predefined characteristics. No subgroups had significantly improved outcomes with intensive therapy.1

 

 

 

WHAT’S NEW: Now we know: Don’t go too low

This study, in contrast to a number of smaller studies of lower quality, demonstrates a higher all-cause mortality rate at 90 days for critically ill patients receiving intensive glucose therapy. It is now clear that, among critically ill hospitalized patients, aiming for intensive glucose control (81-108 mg/dL) is associated with an increased rate of severe hypoglycemic events and all-cause mortality at 90 days. The previously used goal of conventional therapy (≤180 mg/dL) is safer.

CAVEATS: Study population may not reflect primary care

There are 2 caveats to this study. The first is that because of the nature of the research, it was impossible to maintain blinding of the clinical staff to patient assignments. The second important caveat pertains to the severity of illness among participants in this multicenter study: Most of these patients were in ICUs at tertiary care medical centers and had an expected ICU length of stay of 3 or more days. Although many family physicians manage patients in ICUs, the patients randomized in this study may represent a sicker than average patient population for some hospitals.

CHALLENGES TO IMPLEMENTATION: Some may doubt validity of this outcome

Less aggressive glycemic control for critically ill patients should be easier to achieve, not more difficult. However, a change in glucose targets may require new admission order sets and, notably, reeducation of physicians and nurses who have been convinced by earlier studies that more intensive glucose control is superior.

Acknowledgments

The PURLs Surveillance System is supported in part by Grant Number UL1RR024999 from the National Center for Research Resources, a Clinical Translational Science Award to the University of Chicago. The content is solely the responsibility of the authors and does not necessarily represent the official views of the National Center for Research Resources or the National Institutes of Health.

PURLs methodology

This study was selected and evaluated using FPIN’s Priority Updates from the Research Literature (PURL) Surveillance System methodology. The criteria and findings leading to the selection of this study as a PURL can be accessed at www.jfponline.com/purls.

Practice changer

For hyperglycemic patients admitted to an intensive care unit (ICU), the target blood glucose level should be ≤180 mg/dL, not 81 to 108 mg/dL. More aggressive glucose lowering is associated with a higher mortality rate.1

Strength of recommendation

B: Based on a single, high-quality randomized clinical trial.

Finfer S, Chittock DR, Su SY, et al; NICE-SUGAR Study Investigators. Intensive versus conventional glucose control in critically ill patients. N Engl J Med. 2009;360:1283-1297.

 

ILLUSTRATIVE CASE

A 71-year-old woman with diabetes and coronary artery disease has just been admitted to the ICU, where she’ll receive treatment for sepsis, multilobar pneumonia, and respiratory failure requiring mechanical ventilation. Her blood sugar is 253 mg/dL. In writing her admission orders, you contemplate targets for glycemic control. How low should you go?

Hyperglycemia is common in patients admitted to intensive care, whether or not they have diabetes. Elevated blood sugar is associated with stress and trauma and affects both postoperative and critically ill medical patients. A wealth of evidence has demonstrated that hyperglycemia is associated with poorer outcomes and increased mortality in this patient population, including those with myocardial infarction, stroke, trauma, and other medical conditions.2-5 Thus, intensive glucose control is the standard of care in the ICU, based on consensus guidelines from such groups as the American Diabetes Association (ADA) and the Surviving Sepsis Campaign—an initiative developed by 3 critical care organizations and endorsed by 16 specialty groups.6-8

Is intense therapy better? Study results differ
The association between hyperglycemia and an increased risk of death led investigators to study the effectiveness of aggressive treatment with insulin in decreasing morbidity and mortality. A 2004 meta-analysis of 35 trials comparing insulin vs no insulin in critically ill hospitalized patients demonstrated a 15% reduction in short-term mortality among patients treated with insulin.9 A 2008 meta-analysis of 29 randomized trials, including data from 8432 adult ICU patients, compared intensive insulin therapy with conventional therapy—and found that intensive therapy did not lower hospital mortality rates compared with conventional therapy. In addition, this meta-analysis revealed a marked increase in severe hypoglycemia (blood sugar ≤40 mg/dL) in the intensive therapy group.10 (The intensive therapy group included studies with glucose goals of ≤110 mg/dL and <150 mg/dL in about equal numbers; conventional therapy goals were generally between 180 and 200 mg/dL.)

The studies included in both the meta-analyses, however, were mostly small, single-center trials, and of low-to-medium quality. In addition, methods for achieving glycemic control varied. Nonetheless, current consensus guidelines set a goal for glucose levels of 80 to 110 mg/dL for all critically ill hospitalized patients.6-8 But because of the lack of sufficient high-quality evidence from a single large RCT, Finfer et al conducted the large study described here to clearly establish that intensive glycemic control decreases all-cause mortality. Given their hypothesis, the results were surprising.

STUDY SUMMARY: Intensive therapy does more harm than help

NICE-SUGAR (Normoglycaemia in Intensive Care Evaluation-Survival Using Glucose Algorithm Regulation) was a large-scale, multicenter, multinational trial comparing aggressive blood sugar control (goal 81-108 mg/dL) with conventional therapy (goal ≤180 mg/dL) in 6104 critically ill hospitalized patients with hyperglycemia. Patients were followed for 90 days. The primary end point was death from any cause 90 days after randomization. Secondary outcomes included survival time during the first 90 days, specific cause of death, duration of mechanical ventilation, renal replacement therapy, and length of stays in the ICU and in the hospital. Other outcomes included death from any cause within 28 days, place of death, new organ failure, positive blood culture, blood transfusion, and units of blood transfused.

The study was conducted in 42 hospitals in Canada, Australia, and New Zealand. Patients had to have an anticipated ICU admission of 3 days or more and randomization had to occur within 24 hours of admission. The study protocol was discontinued when patients began eating or were discharged from the ICU; if they were readmitted to the ICU within 90 days of randomization, the study protocol was resumed.

Treatment assignment was revealed to clinical staff after randomization, and was determined by a specific algorithm ( https://studies.thegeorgeinstitute.org/nice/ ). Blood sugar levels were managed with insulin infusions.

In the conventional group, insulin was started at 1 unit/h for glucose levels >180 mg/dL, and decreased or stopped when levels were <144 mg/dL, depending on previous glucose value and current rate of drip. In the intensive therapy group, insulin was initiated for lower levels (blood sugar >109 mg/dL) and at a higher rate (2 units/h). The insulin rate was decreased or maintained for glucose levels from 64 to 80 mg/dL, depending on previous glucose value and current rate of drip. Insulin was withheld for blood sugar levels of <64 mg/dL.

Contrary to the hypothesis, intensive therapy spelled trouble. Patients with intensive glycemic control had an all-cause mortality rate of 27.5%, compared with a rate of 24.9% for patients in the conventional therapy group (P=.04, number needed to harm [NNH]=38). Severe hypoglycemia (glucose ≤40 mg/dL) occurred in 6.8% of those in the intensive therapy group, compared with 0.5% in the conventional therapy group (P=.03, NNH=16).

Most of the deaths in both groups occurred in the ICU or in the hospital. Deaths from cardiovascular causes were more common among those in the intensive therapy group. There were no significant differences in any other outcomes. The mean glucose level in the intensive therapy group was 118, vs 145 mg/dL in the conventional therapy group.

For multivariate and subgroup analyses, the patients were assigned strata (Canada or Australia/New Zealand; operative vs nonoperative admission) or classified into groups (traumatic vs atraumatic; diabetes vs no diabetes; corticosteroids in previous 72 hours or not; high vs low critical illness symptom severity) based on predefined characteristics. No subgroups had significantly improved outcomes with intensive therapy.1

 

 

 

WHAT’S NEW: Now we know: Don’t go too low

This study, in contrast to a number of smaller studies of lower quality, demonstrates a higher all-cause mortality rate at 90 days for critically ill patients receiving intensive glucose therapy. It is now clear that, among critically ill hospitalized patients, aiming for intensive glucose control (81-108 mg/dL) is associated with an increased rate of severe hypoglycemic events and all-cause mortality at 90 days. The previously used goal of conventional therapy (≤180 mg/dL) is safer.

CAVEATS: Study population may not reflect primary care

There are 2 caveats to this study. The first is that because of the nature of the research, it was impossible to maintain blinding of the clinical staff to patient assignments. The second important caveat pertains to the severity of illness among participants in this multicenter study: Most of these patients were in ICUs at tertiary care medical centers and had an expected ICU length of stay of 3 or more days. Although many family physicians manage patients in ICUs, the patients randomized in this study may represent a sicker than average patient population for some hospitals.

CHALLENGES TO IMPLEMENTATION: Some may doubt validity of this outcome

Less aggressive glycemic control for critically ill patients should be easier to achieve, not more difficult. However, a change in glucose targets may require new admission order sets and, notably, reeducation of physicians and nurses who have been convinced by earlier studies that more intensive glucose control is superior.

Acknowledgments

The PURLs Surveillance System is supported in part by Grant Number UL1RR024999 from the National Center for Research Resources, a Clinical Translational Science Award to the University of Chicago. The content is solely the responsibility of the authors and does not necessarily represent the official views of the National Center for Research Resources or the National Institutes of Health.

PURLs methodology

This study was selected and evaluated using FPIN’s Priority Updates from the Research Literature (PURL) Surveillance System methodology. The criteria and findings leading to the selection of this study as a PURL can be accessed at www.jfponline.com/purls.

References

1. Finfer S, Chittock DR, Su SY, et al. NICE-SUGAR Study Investigators. Intensive versus conventional glucose control in critically ill patients. N Engl J Med. 2009;360:1283-1297.

2. Capes SE, Hunt D, Malmberg K, et al. Stress hyperglycaemia and increased risk of death after myocardial infarction in patients with and without diabetes: a systematic overview. Lancet. 2000;355:773-778.

3. Capes SE, Hunt D, Malmberg K, et al. Stress hyperglycemia and prognosis of stroke in nondiabetic and diabetic patients: a systematic overview. Stroke. 2001;32:2426-2432.

4. Gale SC, Sicoutris C, Reilly PM, et al. Poor glycemic control is associated with increased mortality in critically ill trauma patients. Am Surg. 2007;73:454-460.

5. Krinsley JS. Association between hyperglycemia and increased hospital mortality in a heterogeneous population of critically ill patients. Mayo Clin Proc. 2003;78:1471-1478.

6. Standards of medical care in diabetes—2008. Diabetes Care. 2008;31(suppl 1):S12-S54.

7. Rodbard HW, Blonde L, Braithwaite SS, et al. American Association of Clinical Endocrinologists medical guidelines for clinical practice for the management of diabetes mellitus. Endocr Pract. 2007;13(suppl 1):1-68.

8. Dellinger RP, Levy MM, Carlet JM, et al. Surviving Sepsis Campaign: international guidelines for management of severe sepsis and septic shock: 2008. Crit Care Med. 2008;36:296-327.

9. Pittas AG, Siegel RD, Lau J. Insulin therapy for critically ill hospitalized patients: a meta-analysis of randomized controlled trials. Arch Intern Med. 2004;164:2005-2011.

10. Wiener RS, Wiener DC, Larson RJ. Benefits and risks of tight glucose control in critically ill adults: a meta-analysis. JAMA. 2008;300:933-944.

References

1. Finfer S, Chittock DR, Su SY, et al. NICE-SUGAR Study Investigators. Intensive versus conventional glucose control in critically ill patients. N Engl J Med. 2009;360:1283-1297.

2. Capes SE, Hunt D, Malmberg K, et al. Stress hyperglycaemia and increased risk of death after myocardial infarction in patients with and without diabetes: a systematic overview. Lancet. 2000;355:773-778.

3. Capes SE, Hunt D, Malmberg K, et al. Stress hyperglycemia and prognosis of stroke in nondiabetic and diabetic patients: a systematic overview. Stroke. 2001;32:2426-2432.

4. Gale SC, Sicoutris C, Reilly PM, et al. Poor glycemic control is associated with increased mortality in critically ill trauma patients. Am Surg. 2007;73:454-460.

5. Krinsley JS. Association between hyperglycemia and increased hospital mortality in a heterogeneous population of critically ill patients. Mayo Clin Proc. 2003;78:1471-1478.

6. Standards of medical care in diabetes—2008. Diabetes Care. 2008;31(suppl 1):S12-S54.

7. Rodbard HW, Blonde L, Braithwaite SS, et al. American Association of Clinical Endocrinologists medical guidelines for clinical practice for the management of diabetes mellitus. Endocr Pract. 2007;13(suppl 1):1-68.

8. Dellinger RP, Levy MM, Carlet JM, et al. Surviving Sepsis Campaign: international guidelines for management of severe sepsis and septic shock: 2008. Crit Care Med. 2008;36:296-327.

9. Pittas AG, Siegel RD, Lau J. Insulin therapy for critically ill hospitalized patients: a meta-analysis of randomized controlled trials. Arch Intern Med. 2004;164:2005-2011.

10. Wiener RS, Wiener DC, Larson RJ. Benefits and risks of tight glucose control in critically ill adults: a meta-analysis. JAMA. 2008;300:933-944.

Issue
The Journal of Family Practice - 58(8)
Issue
The Journal of Family Practice - 58(8)
Page Number
424-426
Page Number
424-426
Publications
Publications
Topics
Article Type
Display Headline
Glucose control: How low should you go with the critically ill?
Display Headline
Glucose control: How low should you go with the critically ill?
Sections
PURLs Copyright

Copyright © 2009 The Family Physicians Inquiries Network.
All rights reserved.

Disallow All Ads
Alternative CME
Use ProPublica
Hide sidebar & use full width
render the right sidebar.
Conference Recap Checkbox
Not Conference Recap
Clinical Edge
Article PDF Media
Media Files

Suspect an eating disorder? Suggest CBT

Article Type
Changed
Fri, 06/19/2020 - 14:20
Display Headline
Suspect an eating disorder? Suggest CBT
Practice changer

Refer patients with eating disorder not otherwise specified (NOS) for cognitive behavioral therapy. CBT, which has proven to be the most useful behavioral treatment for bulimia,1 has now been shown to be effective for patients in the NOS category.2

Strength of recommendation

B: 1 high-quality, randomized controlled trial (RCT).

Fairburn CG, Cooper Z, Doll HA, et al. Transdiagnostic cognitive-behavioral therapy for patients with eating disorders: a two-site trial with 60-week follow-up. Am J Psychiatry. 2009;166:311-319.

 

ILLUSTRATIVE CASE

A 23-year-old patient with a body mass index (BMI) of 18 tells you she’s fat and she’s afraid of gaining weight. Further questioning reveals that your patient binges on cookies and potato chips about once a week, then compensates for overeating by taking laxatives or exercising excessively—a practice she’s been following since she started college several years ago. The eating disorder she describes does not meet the criteria in the Diagnostic and Statistical Manual of Mental Disorders (DSM-IV-TR) for bulimia or anorexia nervosa, although it has elements of both. Rather, it fits the diagnostic criteria for eating disorder NOS. You’re aware that CBT is the first-line behavioral treatment for bulimia, and wonder whether it would be helpful for your patient.

Eating disorders often go undetected and untreated in primary care practices,3 as many patients don’t volunteer information about their weight or behaviors related to food, and physicians often fail to ask. Overall, as few as 10% of those with eating disorders receive any form of treatment.1

Would you recognize this loosely defined disorder?

In the United States, the lifetime prevalence of eating disorders is 0.6% for anorexia nervosa (0.3% for men and 0.9% for women), 1.0% for bulimia (0.5% for men and 1.5% for women), and 2.8% for binge-eating disorder (2.0% for men, 3.5% for women).4 Eating disorder NOS, which encompasses subthreshold cases of anorexia or bulimia, patients with elements of both anorexia and bulimia, and patients with binge-eating disorder, accounts for 50% to 80% of eating disorder diagnoses in outpatient settings. Yet there have been few studies of the treatment of these patients.2,5,6

A review of DSM-IV criteria

The diagnostic criteria for anorexia nervosa include a refusal to maintain a weight of at least 85% of normal body weight (or having a BMI ≤17.5), intense fear of gaining weight, disturbance in the way one’s body shape is experienced, and amenorrhea in females who are post-menarche.

Criteria for bulimia include recurrent episodes of binge eating (consuming a large amount of food with a sense of lack of control over eating) and recurrent inappropriate compensatory behaviors to prevent weight gain (self-induced vomiting, excessive exercise, fasting, laxatives, diuretics, or enemas) at least twice weekly for 3 months; and self-evaluation that is unduly influenced by body shape and weight.7 Most patients with eating disorder NOS have clinical features of both anorexia and bulimia.6

APA guidelines are silent on NOS

CBT has consistently proven to be the most useful behavioral treatment for patients with bulimia.1 Selective serotonin reuptake inhibitors (SSRIs) such as fluoxetine—the only medication with Food and Drug Administration approval for the treatment of an eating disorder8 —are about as effective as CBT, and the combination of CBT and an SSRI is superior to either treatment alone.9 CBT has also been found to be somewhat effective in treating binge-eating disorder.10

Anorexia nervosa, the most deadly eating disorder (the mortality rate is 6.6%11 ) and the most difficult to treat, is the exception. Several studies have assessed CBT for treating anorexia, but it has not been found to be very effective.10,12,13

The 2006 American Psychiatric Association practice guidelines for the treatment of patients with eating disorders feature recommendations for anorexia, bulimia, and binge-eating disorder, but do not address eating disorder NOS.10 The National Institute for Clinical Excellence (NICE) in the United Kingdom issued guidelines for the treatment of eating disorders in 2004. In response to the lack of evidence for treating eating disorder NOS, NICE recommended basing treatment on the form of eating disorder that most closely resembles the patient’s presentation.14 Fairburn et al addressed the lack of evidence for treatment of eating disorder NOS with the study summarized here.

 

 

 

STUDY SUMMARY: Both broad and focused CBT delivered results

Conducted at 2 eating disorder centers in the United Kingdom, this RCT included 154 patients, 18 to 65 years of age, who met DSM-IV criteria for either bulimia or eating disorder NOS. Exclusion criteria included prior treatment with CBT or other evidence-based treatment for the same eating disorder, and a BMI ≤17.5.

Most of the patients were female (95.5%) and white (90.3%), with a median age of 26 years and a median duration of eating disorder of 8.6 years. Sixty-two percent of the patients had a diagnosis of eating disorder NOS, and 38% were diagnosed with bulimia. Half of the patients had another current psychiatric diagnosis—a major depressive disorder, an anxiety disorder, or substance abuse.

The patients were randomized into 4 groups: Two received immediate treatment, and the other 2, referred to as waiting list controls, waited 8 weeks before beginning treatment. Treatment consisted of 1 of 2 forms of CBT-E, an enhanced form of CBT used to treat adult outpatients with eating disorders. Patients either received CBT-Ef, a focused form of CBT that exclusively targets eating disorder psychopathology, or CBT-Eb, a broader form of therapy that also addresses other problems that are common in patients with eating disorders, such as perfectionism and low self-esteem.

Both types of CBT-E featured a 90-minute preparatory session, 20 50-minute sessions, and 1 review session 20 weeks after completion of treatment. In the first 4 sessions, CBT-Ef and CBT-Eb were identical—addressing the eating disorder exclusively. CBT-Ef continued to focus on the eating disorder for the rest of the sessions, while subsequent CBT-Eb sessions also dealt with mood intolerance, interpersonal difficulties, and related issues. Five therapists—4 psychologists and 1 nurse-therapist—conducted the treatments.

Patients were weaned from ongoing psychiatric therapy during the study, but those who were on antidepressant therapy (n=76) were able to continue it. Patients were assessed before treatment, at the end of the waiting period for those in the control groups, after 8 weeks of treatment, at the end of treatment, and 20, 40, and 60 weeks after completion of treatment. (Twenty-two percent of the enrollees did not complete treatment.)

Primary outcomes were based on the Eating Disorder Examination (EDE), administered by assessors who were not involved in the treatment and were blinded to the patients’ group assignment. Change in severity of eating disorder features was measured by the global EDE score (0-6) and attaining a global EDE score <1.74 (<1 standard deviation above the community mean).

No treatment vs CBT. The waiting period left little doubt of the short-term efficacy of CBT: After 8 weeks, there was significant improvement in eating disorder behaviors and overall severity in both the CBT-Ef and CBT-Eb groups (EDE scores fell from 4.15 at baseline to 3.26 and from 4.04 to 2.89, respectively). In the same time period, scores for the waiting list control groups remained flat (from 4.08 at baseline to 3.99).

At the end of treatment and at the 60-week follow-up, patients in both forms of CBT-E showed significant improvement across all measures, with no significant difference between treatments. By the end of treatment, 66.4% of those who completed all of the CBT sessions had global EDE scores <1.74 (considered clinically significant).

Subgroup analysis offers opportunity for fine-tuning

When analyzed separately, the patients with bulimia and those with eating disorder NOS did equally well at the end of treatment: 52.7% of those with bulimia and 53.3% of those with eating disorder NOS had global EDE scores <1.74. At the 60-week follow-up, the patients with bulimia maintained their improvement slightly more: 61.4% had global EDE scores <1.74, compared with 45.7% of the patients with eating disorder NOS.

The researchers also compared the outcomes of patients with the most complex additional psychopathology with those of patients with less complex problems. Greater complexity was defined as moderate ratings in at least 2 of the following domains: mood intolerance, clinical perfectionism, low self-esteem, and interpersonal difficulties.

Broad focus more effective for high complexity. Overall, those in the more complex subgroup did not respond as well; 48% had global EDE scores <1.74, vs 60% of those in the less complex group. However, those in the more complex subgroup did better with the broad form of CBT (at 60-week follow up, 60% had scores <1.74 with CBT-Eb, compared with 40% in the CBT-Ef treatment arm), while the less complex subgroup did better with the more tightly focused CBT-Ef. 2

WHAT’S NEW: Evidence supports CBT for NOS diagnosis

The most recent (2004) Cochrane review of “psychotherapy for bulimia nervosa and binging” included 40 RCTs of patients with bulimia, binge-eating disorder, and eating disorder NOS with recurrent binge-eating episodes (included in 7 studies). While the review confirmed that CBT is effective for bulimia and “similar syndromes,” it identified a need for larger and higher quality trials of CBT, particularly in patients with eating disorder NOS.1 The study reviewed in this PURL—the first large, high-quality trial to include a number of patients with eating disorder NOS—provides strong evidence that CBT is effective for this group of patients.2

 

 

 

CAVEATS: Limited wait time leaves unanswered questions

One limitation of this study is the lack of a control group beyond the 8-week waiting period. Prior studies of CBT for bulimia that delayed therapy for those in the control groups for a longer duration have consistently shown that patients receiving CBT did significantly better than those in the control group.9 While a “no treatment” group would have made the results more robust in this case, it would not have been ethical to withhold treatment for the entire length of the study.

It is noteworthy, too, that this study only included patients with a BMI >17.5. Patients with a diagnosis of anorexia nervosa, who by definition have a lower BMI, will need other treatments, including hospitalization in some cases.

CHALLENGES TO IMPLEMENTATION: Identifying patients and therapists

The primary challenge is to determine which of your patients have eating disorders. When discussing diet, adding a simple question such as, “Are you happy with your current weight?” can help you identify those who meet the criteria for an eating disorder or are at high risk.3

Identifying local mental health providers who are trained to provide CBT for patients with eating disorders is another concern. Insurance coverage for this intensive treatment may also be a limiting factor in some cases.

Many studies support the use of fluoxetine for patients with bulimia, and combined treatment with SSRIs and CBT has been shown to be superior to either treatment alone.8,10,14 Consider starting the patient on an antidepressant while she (or he) awaits the start of CBT.

Acknowledgements

The PURLs Surveillance System is supported in part by Grant Number UL1RR024999 from the National Center for Research Resources, a Clinical Translational Science Award to the University of Chicago. The content is solely the responsibility of the authors and does not necessarily represent the official views of the National Center for Research Resources or the National Institutes of Health.

PURLs methodology

This study was selected and evaluated using FPIN’s Priority Updates from the Research Literature (PURLs) Surveillance System methodology. The criteria and findings leading to the selection of this study as a PURL can be accessed at www.jfponline.com/purls.

Files
References

1. Hay PJ, Bacaltchuk J, Stefano S. Psychotherapy for bulimia nervosa and binging. Cochrane Database Syst Rev. 2004;(3):CD000562.

2. Fairburn CG, Cooper Z, Doll HA, et al. Transdiagnostic cognitive-behavioral therapy for patients with eating disorders: a two-site trial with 60-week follow-up. Am J Psychiatry. 2009;166:311-319.

3. Pritts SD, Susman J. Diagnosis of eating disorders in primary care. Am Fam Physician. 2003;67:297-304.

4. Hudson JI, Hiripi E, Pope HG Jr, et al. The prevalence and correlates of eating disorders in the National Comorbidity Survey Replication. Biol Psychiatry. 2007;61:348-358.

5. Button EJ, Benson E, Nollett C, et al. Don’t forget EDNOS (eating disorder not otherwise specified): patterns of service use in an eating disorders service. Psychiatr Bull. 2005;29:134-136.

6. Fairburn CG, Cooper Z, Bohn K, et al. The severity and status of eating disorder NOS: implications for DSM-V. Behav Res Ther. 2007;45:1705-1715.

7. American Psychiatric Association. Diagnostic and Statistical Manual of Mental Disorders. 4th ed. rev. Washington, DC: American Psychiatric Association; 2000:583-595,787.

8. Berkman ND, Bulik CM, Brownley KA, et al. Management of eating disorders. Evidence Report/Technology Assessment No.135. AHRQ Publication No. 06-E010. Rockville, MD: Agency for Healthcare Research and Quality; April 2006.

9. Shapiro JR, Berkman ND, Brownley KA, et al. Bulimia nervosa treatment: a systematic review of randomized controlled trials. Int J Eat Disord. 2007;40:321-336.

10. American Psychiatric Association Practice Guideline. Treatment of patients with eating disorders. 3rd ed Available at: http://www.psychiatryonline.com/pracGuide/pracGuideTopic_12.aspx. Accessed April 9, 2009.

11. Eckert ED, Halmi KA, Marchi P, et al. Ten-year follow-up of anorexia nervosa: clinical course and outcome. Psychol Med. 1995;25:143-156.

12. Hay PJ, Bacaltchuk J, Byrnes RT, et al. Individual psychotherapy in the outpatient treatment of adults with anorexia nervosa. Cochrane Database Syst Rev. 2009;(1):CD003909.

13. Morris J, Twaddle S. Anorexia Nervosa. BMJ. 2007;334:894-898.

14. National Collaborating Centre for Mental Health. Eating disorders: core interventions in the treatment and management of anorexia nervosa, bulimia nervosa and related eating disorders. Clinical guideline 9. Available at: http://www.nice.org.uk/guidance/CG9/niceguidance/pdf/English. Accessed June 28, 2007.

Article PDF
Author and Disclosure Information

Sarah-Anne Schumann, MD
Department of Family Medicine, The University of Chicago

PURLs EDITOR
John Hickner, MD, MSc
Department of Family Medicine, Cleveland Clinic

Issue
The Journal of Family Practice - 58(5)
Publications
Topics
Page Number
265-268
Legacy Keywords
Sarah-Anne Schumann; eating disorder not otherwise specified (NOS); cognitive behavioral therapy (CBT); bulimia; binge-eating disorder; selective serotonin reuptake inhibitors (SSRIs)
Sections
Files
Files
Author and Disclosure Information

Sarah-Anne Schumann, MD
Department of Family Medicine, The University of Chicago

PURLs EDITOR
John Hickner, MD, MSc
Department of Family Medicine, Cleveland Clinic

Author and Disclosure Information

Sarah-Anne Schumann, MD
Department of Family Medicine, The University of Chicago

PURLs EDITOR
John Hickner, MD, MSc
Department of Family Medicine, Cleveland Clinic

Article PDF
Article PDF
Practice changer

Refer patients with eating disorder not otherwise specified (NOS) for cognitive behavioral therapy. CBT, which has proven to be the most useful behavioral treatment for bulimia,1 has now been shown to be effective for patients in the NOS category.2

Strength of recommendation

B: 1 high-quality, randomized controlled trial (RCT).

Fairburn CG, Cooper Z, Doll HA, et al. Transdiagnostic cognitive-behavioral therapy for patients with eating disorders: a two-site trial with 60-week follow-up. Am J Psychiatry. 2009;166:311-319.

 

ILLUSTRATIVE CASE

A 23-year-old patient with a body mass index (BMI) of 18 tells you she’s fat and she’s afraid of gaining weight. Further questioning reveals that your patient binges on cookies and potato chips about once a week, then compensates for overeating by taking laxatives or exercising excessively—a practice she’s been following since she started college several years ago. The eating disorder she describes does not meet the criteria in the Diagnostic and Statistical Manual of Mental Disorders (DSM-IV-TR) for bulimia or anorexia nervosa, although it has elements of both. Rather, it fits the diagnostic criteria for eating disorder NOS. You’re aware that CBT is the first-line behavioral treatment for bulimia, and wonder whether it would be helpful for your patient.

Eating disorders often go undetected and untreated in primary care practices,3 as many patients don’t volunteer information about their weight or behaviors related to food, and physicians often fail to ask. Overall, as few as 10% of those with eating disorders receive any form of treatment.1

Would you recognize this loosely defined disorder?

In the United States, the lifetime prevalence of eating disorders is 0.6% for anorexia nervosa (0.3% for men and 0.9% for women), 1.0% for bulimia (0.5% for men and 1.5% for women), and 2.8% for binge-eating disorder (2.0% for men, 3.5% for women).4 Eating disorder NOS, which encompasses subthreshold cases of anorexia or bulimia, patients with elements of both anorexia and bulimia, and patients with binge-eating disorder, accounts for 50% to 80% of eating disorder diagnoses in outpatient settings. Yet there have been few studies of the treatment of these patients.2,5,6

A review of DSM-IV criteria

The diagnostic criteria for anorexia nervosa include a refusal to maintain a weight of at least 85% of normal body weight (or having a BMI ≤17.5), intense fear of gaining weight, disturbance in the way one’s body shape is experienced, and amenorrhea in females who are post-menarche.

Criteria for bulimia include recurrent episodes of binge eating (consuming a large amount of food with a sense of lack of control over eating) and recurrent inappropriate compensatory behaviors to prevent weight gain (self-induced vomiting, excessive exercise, fasting, laxatives, diuretics, or enemas) at least twice weekly for 3 months; and self-evaluation that is unduly influenced by body shape and weight.7 Most patients with eating disorder NOS have clinical features of both anorexia and bulimia.6

APA guidelines are silent on NOS

CBT has consistently proven to be the most useful behavioral treatment for patients with bulimia.1 Selective serotonin reuptake inhibitors (SSRIs) such as fluoxetine—the only medication with Food and Drug Administration approval for the treatment of an eating disorder8 —are about as effective as CBT, and the combination of CBT and an SSRI is superior to either treatment alone.9 CBT has also been found to be somewhat effective in treating binge-eating disorder.10

Anorexia nervosa, the most deadly eating disorder (the mortality rate is 6.6%11 ) and the most difficult to treat, is the exception. Several studies have assessed CBT for treating anorexia, but it has not been found to be very effective.10,12,13

The 2006 American Psychiatric Association practice guidelines for the treatment of patients with eating disorders feature recommendations for anorexia, bulimia, and binge-eating disorder, but do not address eating disorder NOS.10 The National Institute for Clinical Excellence (NICE) in the United Kingdom issued guidelines for the treatment of eating disorders in 2004. In response to the lack of evidence for treating eating disorder NOS, NICE recommended basing treatment on the form of eating disorder that most closely resembles the patient’s presentation.14 Fairburn et al addressed the lack of evidence for treatment of eating disorder NOS with the study summarized here.

 

 

 

STUDY SUMMARY: Both broad and focused CBT delivered results

Conducted at 2 eating disorder centers in the United Kingdom, this RCT included 154 patients, 18 to 65 years of age, who met DSM-IV criteria for either bulimia or eating disorder NOS. Exclusion criteria included prior treatment with CBT or other evidence-based treatment for the same eating disorder, and a BMI ≤17.5.

Most of the patients were female (95.5%) and white (90.3%), with a median age of 26 years and a median duration of eating disorder of 8.6 years. Sixty-two percent of the patients had a diagnosis of eating disorder NOS, and 38% were diagnosed with bulimia. Half of the patients had another current psychiatric diagnosis—a major depressive disorder, an anxiety disorder, or substance abuse.

The patients were randomized into 4 groups: Two received immediate treatment, and the other 2, referred to as waiting list controls, waited 8 weeks before beginning treatment. Treatment consisted of 1 of 2 forms of CBT-E, an enhanced form of CBT used to treat adult outpatients with eating disorders. Patients either received CBT-Ef, a focused form of CBT that exclusively targets eating disorder psychopathology, or CBT-Eb, a broader form of therapy that also addresses other problems that are common in patients with eating disorders, such as perfectionism and low self-esteem.

Both types of CBT-E featured a 90-minute preparatory session, 20 50-minute sessions, and 1 review session 20 weeks after completion of treatment. In the first 4 sessions, CBT-Ef and CBT-Eb were identical—addressing the eating disorder exclusively. CBT-Ef continued to focus on the eating disorder for the rest of the sessions, while subsequent CBT-Eb sessions also dealt with mood intolerance, interpersonal difficulties, and related issues. Five therapists—4 psychologists and 1 nurse-therapist—conducted the treatments.

Patients were weaned from ongoing psychiatric therapy during the study, but those who were on antidepressant therapy (n=76) were able to continue it. Patients were assessed before treatment, at the end of the waiting period for those in the control groups, after 8 weeks of treatment, at the end of treatment, and 20, 40, and 60 weeks after completion of treatment. (Twenty-two percent of the enrollees did not complete treatment.)

Primary outcomes were based on the Eating Disorder Examination (EDE), administered by assessors who were not involved in the treatment and were blinded to the patients’ group assignment. Change in severity of eating disorder features was measured by the global EDE score (0-6) and attaining a global EDE score <1.74 (<1 standard deviation above the community mean).

No treatment vs CBT. The waiting period left little doubt of the short-term efficacy of CBT: After 8 weeks, there was significant improvement in eating disorder behaviors and overall severity in both the CBT-Ef and CBT-Eb groups (EDE scores fell from 4.15 at baseline to 3.26 and from 4.04 to 2.89, respectively). In the same time period, scores for the waiting list control groups remained flat (from 4.08 at baseline to 3.99).

At the end of treatment and at the 60-week follow-up, patients in both forms of CBT-E showed significant improvement across all measures, with no significant difference between treatments. By the end of treatment, 66.4% of those who completed all of the CBT sessions had global EDE scores <1.74 (considered clinically significant).

Subgroup analysis offers opportunity for fine-tuning

When analyzed separately, the patients with bulimia and those with eating disorder NOS did equally well at the end of treatment: 52.7% of those with bulimia and 53.3% of those with eating disorder NOS had global EDE scores <1.74. At the 60-week follow-up, the patients with bulimia maintained their improvement slightly more: 61.4% had global EDE scores <1.74, compared with 45.7% of the patients with eating disorder NOS.

The researchers also compared the outcomes of patients with the most complex additional psychopathology with those of patients with less complex problems. Greater complexity was defined as moderate ratings in at least 2 of the following domains: mood intolerance, clinical perfectionism, low self-esteem, and interpersonal difficulties.

Broad focus more effective for high complexity. Overall, those in the more complex subgroup did not respond as well; 48% had global EDE scores <1.74, vs 60% of those in the less complex group. However, those in the more complex subgroup did better with the broad form of CBT (at 60-week follow up, 60% had scores <1.74 with CBT-Eb, compared with 40% in the CBT-Ef treatment arm), while the less complex subgroup did better with the more tightly focused CBT-Ef. 2

WHAT’S NEW: Evidence supports CBT for NOS diagnosis

The most recent (2004) Cochrane review of “psychotherapy for bulimia nervosa and binging” included 40 RCTs of patients with bulimia, binge-eating disorder, and eating disorder NOS with recurrent binge-eating episodes (included in 7 studies). While the review confirmed that CBT is effective for bulimia and “similar syndromes,” it identified a need for larger and higher quality trials of CBT, particularly in patients with eating disorder NOS.1 The study reviewed in this PURL—the first large, high-quality trial to include a number of patients with eating disorder NOS—provides strong evidence that CBT is effective for this group of patients.2

 

 

 

CAVEATS: Limited wait time leaves unanswered questions

One limitation of this study is the lack of a control group beyond the 8-week waiting period. Prior studies of CBT for bulimia that delayed therapy for those in the control groups for a longer duration have consistently shown that patients receiving CBT did significantly better than those in the control group.9 While a “no treatment” group would have made the results more robust in this case, it would not have been ethical to withhold treatment for the entire length of the study.

It is noteworthy, too, that this study only included patients with a BMI >17.5. Patients with a diagnosis of anorexia nervosa, who by definition have a lower BMI, will need other treatments, including hospitalization in some cases.

CHALLENGES TO IMPLEMENTATION: Identifying patients and therapists

The primary challenge is to determine which of your patients have eating disorders. When discussing diet, adding a simple question such as, “Are you happy with your current weight?” can help you identify those who meet the criteria for an eating disorder or are at high risk.3

Identifying local mental health providers who are trained to provide CBT for patients with eating disorders is another concern. Insurance coverage for this intensive treatment may also be a limiting factor in some cases.

Many studies support the use of fluoxetine for patients with bulimia, and combined treatment with SSRIs and CBT has been shown to be superior to either treatment alone.8,10,14 Consider starting the patient on an antidepressant while she (or he) awaits the start of CBT.

Acknowledgements

The PURLs Surveillance System is supported in part by Grant Number UL1RR024999 from the National Center for Research Resources, a Clinical Translational Science Award to the University of Chicago. The content is solely the responsibility of the authors and does not necessarily represent the official views of the National Center for Research Resources or the National Institutes of Health.

PURLs methodology

This study was selected and evaluated using FPIN’s Priority Updates from the Research Literature (PURLs) Surveillance System methodology. The criteria and findings leading to the selection of this study as a PURL can be accessed at www.jfponline.com/purls.

Practice changer

Refer patients with eating disorder not otherwise specified (NOS) for cognitive behavioral therapy. CBT, which has proven to be the most useful behavioral treatment for bulimia,1 has now been shown to be effective for patients in the NOS category.2

Strength of recommendation

B: 1 high-quality, randomized controlled trial (RCT).

Fairburn CG, Cooper Z, Doll HA, et al. Transdiagnostic cognitive-behavioral therapy for patients with eating disorders: a two-site trial with 60-week follow-up. Am J Psychiatry. 2009;166:311-319.

 

ILLUSTRATIVE CASE

A 23-year-old patient with a body mass index (BMI) of 18 tells you she’s fat and she’s afraid of gaining weight. Further questioning reveals that your patient binges on cookies and potato chips about once a week, then compensates for overeating by taking laxatives or exercising excessively—a practice she’s been following since she started college several years ago. The eating disorder she describes does not meet the criteria in the Diagnostic and Statistical Manual of Mental Disorders (DSM-IV-TR) for bulimia or anorexia nervosa, although it has elements of both. Rather, it fits the diagnostic criteria for eating disorder NOS. You’re aware that CBT is the first-line behavioral treatment for bulimia, and wonder whether it would be helpful for your patient.

Eating disorders often go undetected and untreated in primary care practices,3 as many patients don’t volunteer information about their weight or behaviors related to food, and physicians often fail to ask. Overall, as few as 10% of those with eating disorders receive any form of treatment.1

Would you recognize this loosely defined disorder?

In the United States, the lifetime prevalence of eating disorders is 0.6% for anorexia nervosa (0.3% for men and 0.9% for women), 1.0% for bulimia (0.5% for men and 1.5% for women), and 2.8% for binge-eating disorder (2.0% for men, 3.5% for women).4 Eating disorder NOS, which encompasses subthreshold cases of anorexia or bulimia, patients with elements of both anorexia and bulimia, and patients with binge-eating disorder, accounts for 50% to 80% of eating disorder diagnoses in outpatient settings. Yet there have been few studies of the treatment of these patients.2,5,6

A review of DSM-IV criteria

The diagnostic criteria for anorexia nervosa include a refusal to maintain a weight of at least 85% of normal body weight (or having a BMI ≤17.5), intense fear of gaining weight, disturbance in the way one’s body shape is experienced, and amenorrhea in females who are post-menarche.

Criteria for bulimia include recurrent episodes of binge eating (consuming a large amount of food with a sense of lack of control over eating) and recurrent inappropriate compensatory behaviors to prevent weight gain (self-induced vomiting, excessive exercise, fasting, laxatives, diuretics, or enemas) at least twice weekly for 3 months; and self-evaluation that is unduly influenced by body shape and weight.7 Most patients with eating disorder NOS have clinical features of both anorexia and bulimia.6

APA guidelines are silent on NOS

CBT has consistently proven to be the most useful behavioral treatment for patients with bulimia.1 Selective serotonin reuptake inhibitors (SSRIs) such as fluoxetine—the only medication with Food and Drug Administration approval for the treatment of an eating disorder8 —are about as effective as CBT, and the combination of CBT and an SSRI is superior to either treatment alone.9 CBT has also been found to be somewhat effective in treating binge-eating disorder.10

Anorexia nervosa, the most deadly eating disorder (the mortality rate is 6.6%11 ) and the most difficult to treat, is the exception. Several studies have assessed CBT for treating anorexia, but it has not been found to be very effective.10,12,13

The 2006 American Psychiatric Association practice guidelines for the treatment of patients with eating disorders feature recommendations for anorexia, bulimia, and binge-eating disorder, but do not address eating disorder NOS.10 The National Institute for Clinical Excellence (NICE) in the United Kingdom issued guidelines for the treatment of eating disorders in 2004. In response to the lack of evidence for treating eating disorder NOS, NICE recommended basing treatment on the form of eating disorder that most closely resembles the patient’s presentation.14 Fairburn et al addressed the lack of evidence for treatment of eating disorder NOS with the study summarized here.

 

 

 

STUDY SUMMARY: Both broad and focused CBT delivered results

Conducted at 2 eating disorder centers in the United Kingdom, this RCT included 154 patients, 18 to 65 years of age, who met DSM-IV criteria for either bulimia or eating disorder NOS. Exclusion criteria included prior treatment with CBT or other evidence-based treatment for the same eating disorder, and a BMI ≤17.5.

Most of the patients were female (95.5%) and white (90.3%), with a median age of 26 years and a median duration of eating disorder of 8.6 years. Sixty-two percent of the patients had a diagnosis of eating disorder NOS, and 38% were diagnosed with bulimia. Half of the patients had another current psychiatric diagnosis—a major depressive disorder, an anxiety disorder, or substance abuse.

The patients were randomized into 4 groups: Two received immediate treatment, and the other 2, referred to as waiting list controls, waited 8 weeks before beginning treatment. Treatment consisted of 1 of 2 forms of CBT-E, an enhanced form of CBT used to treat adult outpatients with eating disorders. Patients either received CBT-Ef, a focused form of CBT that exclusively targets eating disorder psychopathology, or CBT-Eb, a broader form of therapy that also addresses other problems that are common in patients with eating disorders, such as perfectionism and low self-esteem.

Both types of CBT-E featured a 90-minute preparatory session, 20 50-minute sessions, and 1 review session 20 weeks after completion of treatment. In the first 4 sessions, CBT-Ef and CBT-Eb were identical—addressing the eating disorder exclusively. CBT-Ef continued to focus on the eating disorder for the rest of the sessions, while subsequent CBT-Eb sessions also dealt with mood intolerance, interpersonal difficulties, and related issues. Five therapists—4 psychologists and 1 nurse-therapist—conducted the treatments.

Patients were weaned from ongoing psychiatric therapy during the study, but those who were on antidepressant therapy (n=76) were able to continue it. Patients were assessed before treatment, at the end of the waiting period for those in the control groups, after 8 weeks of treatment, at the end of treatment, and 20, 40, and 60 weeks after completion of treatment. (Twenty-two percent of the enrollees did not complete treatment.)

Primary outcomes were based on the Eating Disorder Examination (EDE), administered by assessors who were not involved in the treatment and were blinded to the patients’ group assignment. Change in severity of eating disorder features was measured by the global EDE score (0-6) and attaining a global EDE score <1.74 (<1 standard deviation above the community mean).

No treatment vs CBT. The waiting period left little doubt of the short-term efficacy of CBT: After 8 weeks, there was significant improvement in eating disorder behaviors and overall severity in both the CBT-Ef and CBT-Eb groups (EDE scores fell from 4.15 at baseline to 3.26 and from 4.04 to 2.89, respectively). In the same time period, scores for the waiting list control groups remained flat (from 4.08 at baseline to 3.99).

At the end of treatment and at the 60-week follow-up, patients in both forms of CBT-E showed significant improvement across all measures, with no significant difference between treatments. By the end of treatment, 66.4% of those who completed all of the CBT sessions had global EDE scores <1.74 (considered clinically significant).

Subgroup analysis offers opportunity for fine-tuning

When analyzed separately, the patients with bulimia and those with eating disorder NOS did equally well at the end of treatment: 52.7% of those with bulimia and 53.3% of those with eating disorder NOS had global EDE scores <1.74. At the 60-week follow-up, the patients with bulimia maintained their improvement slightly more: 61.4% had global EDE scores <1.74, compared with 45.7% of the patients with eating disorder NOS.

The researchers also compared the outcomes of patients with the most complex additional psychopathology with those of patients with less complex problems. Greater complexity was defined as moderate ratings in at least 2 of the following domains: mood intolerance, clinical perfectionism, low self-esteem, and interpersonal difficulties.

Broad focus more effective for high complexity. Overall, those in the more complex subgroup did not respond as well; 48% had global EDE scores <1.74, vs 60% of those in the less complex group. However, those in the more complex subgroup did better with the broad form of CBT (at 60-week follow up, 60% had scores <1.74 with CBT-Eb, compared with 40% in the CBT-Ef treatment arm), while the less complex subgroup did better with the more tightly focused CBT-Ef. 2

WHAT’S NEW: Evidence supports CBT for NOS diagnosis

The most recent (2004) Cochrane review of “psychotherapy for bulimia nervosa and binging” included 40 RCTs of patients with bulimia, binge-eating disorder, and eating disorder NOS with recurrent binge-eating episodes (included in 7 studies). While the review confirmed that CBT is effective for bulimia and “similar syndromes,” it identified a need for larger and higher quality trials of CBT, particularly in patients with eating disorder NOS.1 The study reviewed in this PURL—the first large, high-quality trial to include a number of patients with eating disorder NOS—provides strong evidence that CBT is effective for this group of patients.2

 

 

 

CAVEATS: Limited wait time leaves unanswered questions

One limitation of this study is the lack of a control group beyond the 8-week waiting period. Prior studies of CBT for bulimia that delayed therapy for those in the control groups for a longer duration have consistently shown that patients receiving CBT did significantly better than those in the control group.9 While a “no treatment” group would have made the results more robust in this case, it would not have been ethical to withhold treatment for the entire length of the study.

It is noteworthy, too, that this study only included patients with a BMI >17.5. Patients with a diagnosis of anorexia nervosa, who by definition have a lower BMI, will need other treatments, including hospitalization in some cases.

CHALLENGES TO IMPLEMENTATION: Identifying patients and therapists

The primary challenge is to determine which of your patients have eating disorders. When discussing diet, adding a simple question such as, “Are you happy with your current weight?” can help you identify those who meet the criteria for an eating disorder or are at high risk.3

Identifying local mental health providers who are trained to provide CBT for patients with eating disorders is another concern. Insurance coverage for this intensive treatment may also be a limiting factor in some cases.

Many studies support the use of fluoxetine for patients with bulimia, and combined treatment with SSRIs and CBT has been shown to be superior to either treatment alone.8,10,14 Consider starting the patient on an antidepressant while she (or he) awaits the start of CBT.

Acknowledgements

The PURLs Surveillance System is supported in part by Grant Number UL1RR024999 from the National Center for Research Resources, a Clinical Translational Science Award to the University of Chicago. The content is solely the responsibility of the authors and does not necessarily represent the official views of the National Center for Research Resources or the National Institutes of Health.

PURLs methodology

This study was selected and evaluated using FPIN’s Priority Updates from the Research Literature (PURLs) Surveillance System methodology. The criteria and findings leading to the selection of this study as a PURL can be accessed at www.jfponline.com/purls.

References

1. Hay PJ, Bacaltchuk J, Stefano S. Psychotherapy for bulimia nervosa and binging. Cochrane Database Syst Rev. 2004;(3):CD000562.

2. Fairburn CG, Cooper Z, Doll HA, et al. Transdiagnostic cognitive-behavioral therapy for patients with eating disorders: a two-site trial with 60-week follow-up. Am J Psychiatry. 2009;166:311-319.

3. Pritts SD, Susman J. Diagnosis of eating disorders in primary care. Am Fam Physician. 2003;67:297-304.

4. Hudson JI, Hiripi E, Pope HG Jr, et al. The prevalence and correlates of eating disorders in the National Comorbidity Survey Replication. Biol Psychiatry. 2007;61:348-358.

5. Button EJ, Benson E, Nollett C, et al. Don’t forget EDNOS (eating disorder not otherwise specified): patterns of service use in an eating disorders service. Psychiatr Bull. 2005;29:134-136.

6. Fairburn CG, Cooper Z, Bohn K, et al. The severity and status of eating disorder NOS: implications for DSM-V. Behav Res Ther. 2007;45:1705-1715.

7. American Psychiatric Association. Diagnostic and Statistical Manual of Mental Disorders. 4th ed. rev. Washington, DC: American Psychiatric Association; 2000:583-595,787.

8. Berkman ND, Bulik CM, Brownley KA, et al. Management of eating disorders. Evidence Report/Technology Assessment No.135. AHRQ Publication No. 06-E010. Rockville, MD: Agency for Healthcare Research and Quality; April 2006.

9. Shapiro JR, Berkman ND, Brownley KA, et al. Bulimia nervosa treatment: a systematic review of randomized controlled trials. Int J Eat Disord. 2007;40:321-336.

10. American Psychiatric Association Practice Guideline. Treatment of patients with eating disorders. 3rd ed Available at: http://www.psychiatryonline.com/pracGuide/pracGuideTopic_12.aspx. Accessed April 9, 2009.

11. Eckert ED, Halmi KA, Marchi P, et al. Ten-year follow-up of anorexia nervosa: clinical course and outcome. Psychol Med. 1995;25:143-156.

12. Hay PJ, Bacaltchuk J, Byrnes RT, et al. Individual psychotherapy in the outpatient treatment of adults with anorexia nervosa. Cochrane Database Syst Rev. 2009;(1):CD003909.

13. Morris J, Twaddle S. Anorexia Nervosa. BMJ. 2007;334:894-898.

14. National Collaborating Centre for Mental Health. Eating disorders: core interventions in the treatment and management of anorexia nervosa, bulimia nervosa and related eating disorders. Clinical guideline 9. Available at: http://www.nice.org.uk/guidance/CG9/niceguidance/pdf/English. Accessed June 28, 2007.

References

1. Hay PJ, Bacaltchuk J, Stefano S. Psychotherapy for bulimia nervosa and binging. Cochrane Database Syst Rev. 2004;(3):CD000562.

2. Fairburn CG, Cooper Z, Doll HA, et al. Transdiagnostic cognitive-behavioral therapy for patients with eating disorders: a two-site trial with 60-week follow-up. Am J Psychiatry. 2009;166:311-319.

3. Pritts SD, Susman J. Diagnosis of eating disorders in primary care. Am Fam Physician. 2003;67:297-304.

4. Hudson JI, Hiripi E, Pope HG Jr, et al. The prevalence and correlates of eating disorders in the National Comorbidity Survey Replication. Biol Psychiatry. 2007;61:348-358.

5. Button EJ, Benson E, Nollett C, et al. Don’t forget EDNOS (eating disorder not otherwise specified): patterns of service use in an eating disorders service. Psychiatr Bull. 2005;29:134-136.

6. Fairburn CG, Cooper Z, Bohn K, et al. The severity and status of eating disorder NOS: implications for DSM-V. Behav Res Ther. 2007;45:1705-1715.

7. American Psychiatric Association. Diagnostic and Statistical Manual of Mental Disorders. 4th ed. rev. Washington, DC: American Psychiatric Association; 2000:583-595,787.

8. Berkman ND, Bulik CM, Brownley KA, et al. Management of eating disorders. Evidence Report/Technology Assessment No.135. AHRQ Publication No. 06-E010. Rockville, MD: Agency for Healthcare Research and Quality; April 2006.

9. Shapiro JR, Berkman ND, Brownley KA, et al. Bulimia nervosa treatment: a systematic review of randomized controlled trials. Int J Eat Disord. 2007;40:321-336.

10. American Psychiatric Association Practice Guideline. Treatment of patients with eating disorders. 3rd ed Available at: http://www.psychiatryonline.com/pracGuide/pracGuideTopic_12.aspx. Accessed April 9, 2009.

11. Eckert ED, Halmi KA, Marchi P, et al. Ten-year follow-up of anorexia nervosa: clinical course and outcome. Psychol Med. 1995;25:143-156.

12. Hay PJ, Bacaltchuk J, Byrnes RT, et al. Individual psychotherapy in the outpatient treatment of adults with anorexia nervosa. Cochrane Database Syst Rev. 2009;(1):CD003909.

13. Morris J, Twaddle S. Anorexia Nervosa. BMJ. 2007;334:894-898.

14. National Collaborating Centre for Mental Health. Eating disorders: core interventions in the treatment and management of anorexia nervosa, bulimia nervosa and related eating disorders. Clinical guideline 9. Available at: http://www.nice.org.uk/guidance/CG9/niceguidance/pdf/English. Accessed June 28, 2007.

Issue
The Journal of Family Practice - 58(5)
Issue
The Journal of Family Practice - 58(5)
Page Number
265-268
Page Number
265-268
Publications
Publications
Topics
Article Type
Display Headline
Suspect an eating disorder? Suggest CBT
Display Headline
Suspect an eating disorder? Suggest CBT
Legacy Keywords
Sarah-Anne Schumann; eating disorder not otherwise specified (NOS); cognitive behavioral therapy (CBT); bulimia; binge-eating disorder; selective serotonin reuptake inhibitors (SSRIs)
Legacy Keywords
Sarah-Anne Schumann; eating disorder not otherwise specified (NOS); cognitive behavioral therapy (CBT); bulimia; binge-eating disorder; selective serotonin reuptake inhibitors (SSRIs)
Sections
PURLs Copyright

Copyright © 2009 The Family Physicians Inquiries Network.
All rights reserved.

Disallow All Ads
Alternative CME
Use ProPublica
Hide sidebar & use full width
render the right sidebar.
Conference Recap Checkbox
Not Conference Recap
Clinical Edge
Article PDF Media
Media Files

Dust mite control measures don’t help asthma patients

Article Type
Changed
Fri, 06/19/2020 - 14:04
Display Headline
Dust mite control measures don’t help asthma patients
 

ILLUSTRATIVE CASE

The parents of a 10-year-old patient whom you recently diagnosed with asthma want to do everything they can to reduce his asthma symptoms. They are considering buying hypoallergenic mattress covers and an expensive air filtration system to decrease the levels of dust mite allergens in their home and want to know if you think that will help their son. What do you tell them?

We want to do everything we can to help our patients control their asthma symptoms, but when it comes to household dust mite control measures, this extensive Cochrane review confirms that interventions like mattress covers and air filtration don’t work, despite recent reviews and guidelines recommending them.

Dust mites (Dermatophagoides pteronyssinus) are one of the most common allergens that provoke asthma symptoms in children and adults.2 Dust mites live in warm, humid places and feed on human skin scales. The areas with the highest levels of household infestation are carpets, mattresses, pillows, drapes, upholstered furniture, and clothing.

Guidelines still encourage mattress cover use

The National Asthma Education and Prevention Program (NAEPP) 2007 guidelines recommend using allergen-impermeable mattress and pillow covers and washing sheets and blankets in hot water. They also recommend “considering” reducing indoor humidity, removing bedroom carpets, and washing stuffed toys weekly. The NAEPP Expert Panel cites many studies to support these recommendations.3

The National Environmental Education and Training Foundation (NEETF) 2005 guidelines recommend additional measures to reduce dust mite exposure including vacuuming using a high-efficiency particulate air (HEPA) filter, removing draperies, and considering using a portable air cleaner with a HEPA filter.4

STUDY SUMMARY: 54 trials, but no support for dust mite measures

This Cochrane systematic review included 54 randomized trials that assessed the effects of physical and/or chemical interventions to reduce exposure to house dust mite antigens in the homes of patients with mite-sensitive asthma. These studies included a total of 3002 pediatric and adult asthma patients (9 - 628 patients analyzed per trial) with mite sensitization confirmed by skin testing or IgE serum assays.

Thirty-six studies tested physical interventions, including mattress covers, vacuum cleaning, heating, ventilation, freezing, washing, air filtration, and ionizers. Ten used chemical interventions to kill dust mites; 8 used a combination of physical and chemical methods. Control groups received either placebo or no treatment.

Outcomes studied. The authors extracted data for the following outcomes: subjective well-being, asthma symptom scores, use of medication, days of sick leave from school or work, number of unscheduled visits to a physician or hospital, forced expiratory volume in 1 second (FEV1), peak expiratory flow rate (PEFR), and provocative concentration that causes a 20% fall in FEV1 (PC20). Length of the intervention and follow-up ranged from 2 weeks to 2 years.

Quality of studies. According to modern standards for randomized trials, the quality of many of the 54 studies was not optimal, especially in the descriptions of randomization and the reporting of outcomes. The method of randomization and concealment of allocation was rarely described. Eleven trial reports did not contain any usable data for the meta-analysis because of the way data were reported, and there was significant potential for reporting bias in favor of a treatment effect in the studies included. Mite reduction was successful in 17 trials, unsuccessful in 24 trials, and not reported in 13 trials.

Interventions didn’t help. There were no differences between the intervention and control groups for any of the outcomes. The percentage of patients who improved after the experimental interventions was not significantly different from the percentage of patients in the control groups (relative risk [RR]=1.01; 95% confidence interval [CI], 0.80-1.27; data based on 7 trials). There was no difference in medication usage (data from 10 trials), FEV1 (data from 14 trials), morning PEFR (data from 23 trials), or PC 20 (data from 14 trials) between the intervention and control groups ( TABLE ).1

TABLE
Dust mite control measures didn’t improve these outcomes

OUTCOMESTANDARDIZED MEAN DIFFERENCE* (95% CI)
Medication usage-0.06 (-0.18 to 0.07)
FEV10.11 (-0.05 to 0.28)
Morning PEFR0.00 (-1.0 to 0.10)
PC 200.05 (-0.13 to 0.22)
CI, confidence interval; FEV1, forced expiratory volume in 1 second; PC20, provocative concentration that causes a 20% fall in FEV1; PEFR, peak expiratory flow rate.
*Standardized mean difference is a common way to combine results of different studies for comparison purposes. If the 95% CI crosses 0, there is no effect of the intervention compared with the control.
 

 

 

WHAT’S NEW?: Nothing is new, yet this will be “news” to many

This Cochrane review includes 5 additional trials that have been conducted since the last Cochrane review of this topic in 2004. However, the 2004 review reported the same conclusion—that interventions to reduce house dust mite exposure in asthma patients are ineffective—as did 3 other Cochrane reviews on the same topic beginning in 1998.5-8

So why are the guidelines out of step? Schmidt and Gøtzsche (one of the authors of the Cochrane review) conducted a systematic review of narrative review articles in 2005 to answer this question. They found 70 review articles, 90% of which recommended physical methods to reduce exposure to house dust mites. They discovered that although these review articles included references to support their recommendations of dust mite control measures, the reviews showed significant bias in favor of positive studies and highlighted the results of low-quality studies, including non-randomized studies that had been excluded from the Cochrane reviews.9

CAVEATS: Duration of studies not long enough?

We know that extreme measures to reduce exposure to dust mite allergen, such as relocating to a high altitude or prolonged hospitalization, can reduce asthma symptoms,10,11 but these are clearly not practical solutions for most patients with dust mite-sensitive asthma. When it comes to this Cochrane review, some might argue that many of the interventions included were not of sufficient duration and did not sufficiently reduce the level of house mite allergen to improve asthma symptoms.

However, the subgroups of trials with long treatment duration (1-2 years) and successful mite reduction (determined by different methods, including mite counts and measured antigen levels in dust samples) also failed to show a significant difference between intervention and control groups.1

Tweak the approach? Most dust mite-sensitive asthma patients are sensitive to other allergens, so perhaps multifaceted interventions that target multiple allergens would be more effective.12 But until these potential interventions are supported by stronger evidence, we should not recommend them to our patients.

CHALLENGES TO IMPLEMENTATION: Swimming against the tide is never easy

Although the evidence to date indicates that interventions to reduce home dust mite exposure are ineffective, there are hundreds of products—including mattress and pillow covers ($10-$100), ionizers ($100-$200), and air filtration systems ($500-$800)—that are being marketed to patients with asthma. In addition, patient education handouts from sources such as the American Academy of Family Physicians, the American Academy of Pediatrics, and UpToDate recommend implementing dust mite control measures to reduce dust mite allergen exposure.13-15

We need to start educating our asthma patients properly so they can spend their time, energy, and money on interventions, such as medications, that work—and not on interventions that make no difference.

Acknowledgements

The PURLs Surveillance System is supported in part by Grant Number UL1RR024999 from the National Center for Research Resources, a Clinical Translational Science Award to the University of Chicago. The content is solely the responsibility of the authors and does not necessarily represent the official views of the National Center for Research Resources or the National Institutes of Health.

This study was selected and evaluated using FPIN’s Priority Updates from the Research Literature (PURL) Surveillance System methodology. The criteria and findings leading to the selection of this study as a PURL can be accessed at www.jfponline.com/purls.

Files
References

1. Gotzsche PC, Johansen HK. House dust mite control measures for asthma. Cochrane Database Syst Rev. 2008;(2):CD001187.-

2. German JA, Harper MB. Environmental control of allergic diseases. Am Fam Physician. 2002;66:421-426.

3. National Asthma Education and Prevention Program (NAEPP). Control of environmental factors and comorbid conditions that affect asthma. In: Expert panel report 3: guidelines for the diagnosis and management of asthma. Bethesda, Md: National Heart, Lung, and Blood Institute; 2007.

4. National Environmental Education & Training Foundation (NEETF). Environmental management of pediatric asthma. Guidelines for health care providers. Washington, DC: National Environmental Education & Training Foundation (NEETF); 2005.

5. Gøtzsche PC, Hammarquist C, Burr M. House dust mite control measures in the management of asthma: meta-analysis. BMJ. 1998;317:1105-1110.

6. Hammarquist C, Burr ML, Gotzsche PC. House dust mite control measures for asthma. Cochrane Database Syst Rev. 2000;(2):CD001187.-

7. Gøtzsche PC, Johansen HK, Burr ML, Hammarquist C. House dust mite control measures for asthma. Cochrane Database Syst Rev. 2001;(3):CD001187.-

8. Gøtzsche PC, Johansen HK, Schmidt LM, Burr ML. House dust mite control measures for asthma. Cochrane Database Syst Rev. 2004;(4):CD001187.-

9. Schmidt LM, Gøtzsche PC. Of mites and men: reference bias in narrative review articles: a systematic review. J Fam Pract. 2005;54:334-338.

10. Platts-Mills TA, Tovey ER, Mitchell EB, Moszoro H, Nock P, Wilkins SR. Reduction of bronchial hyperreactivity during prolonged allergen avoidance. Lancet 1982;2:675-678.

11. Grootendorst DC, Dahlen SE. Benefits of high altitude allergen avoidance in atopic adolescents with moderate to severe asthma, over and above treatment with high dose inhaled steroids. Clin Exp Allergy. 2001;31:400-408.

12. Morgan WJ, Crain EF, Gruchalla RS, et al. Results of a home-based environmental intervention among urban children with asthma. N Engl J Med. 2004;351:1068-1080.

13. American Academy of Family Physicians. Dust mites in the home [patient handout]. Available at: http://familydoctor.org/online/famdocen/home/common/asthma/triggers/683.html. Accessed October 23, 2008.

14. American Academy of Pediatrics. Non-pharmacologic approaches to asthma management [patient handout]. Available at: http://www.aap.org/sections/allergy/nonrxchild.pdf. Accessed October 23, 2008.

15. Bailey W. Patient information: Trigger avoidance in asthma. UpToDate [online database]. Version 16.2. Waltham, Mass: UpToDate; 2008.

Article PDF
Author and Disclosure Information

Sarah-Anne Schumann, MD
John Hickner, MD, MSc
Department of Family Medicine, The University of Chicago

PURLs EDITOR
Bernard Ewigman, MD, MSPH
Department of Family Medicine, The University of Chicago

Issue
The Journal of Family Practice - 57(12)
Publications
Topics
Page Number
789-792
Legacy Keywords
Sarah-Anne Schumann; mattress covers; filtration system; asthma symptoms; allergen-impermeable mattresses
Sections
Files
Files
Author and Disclosure Information

Sarah-Anne Schumann, MD
John Hickner, MD, MSc
Department of Family Medicine, The University of Chicago

PURLs EDITOR
Bernard Ewigman, MD, MSPH
Department of Family Medicine, The University of Chicago

Author and Disclosure Information

Sarah-Anne Schumann, MD
John Hickner, MD, MSc
Department of Family Medicine, The University of Chicago

PURLs EDITOR
Bernard Ewigman, MD, MSPH
Department of Family Medicine, The University of Chicago

Article PDF
Article PDF
 

ILLUSTRATIVE CASE

The parents of a 10-year-old patient whom you recently diagnosed with asthma want to do everything they can to reduce his asthma symptoms. They are considering buying hypoallergenic mattress covers and an expensive air filtration system to decrease the levels of dust mite allergens in their home and want to know if you think that will help their son. What do you tell them?

We want to do everything we can to help our patients control their asthma symptoms, but when it comes to household dust mite control measures, this extensive Cochrane review confirms that interventions like mattress covers and air filtration don’t work, despite recent reviews and guidelines recommending them.

Dust mites (Dermatophagoides pteronyssinus) are one of the most common allergens that provoke asthma symptoms in children and adults.2 Dust mites live in warm, humid places and feed on human skin scales. The areas with the highest levels of household infestation are carpets, mattresses, pillows, drapes, upholstered furniture, and clothing.

Guidelines still encourage mattress cover use

The National Asthma Education and Prevention Program (NAEPP) 2007 guidelines recommend using allergen-impermeable mattress and pillow covers and washing sheets and blankets in hot water. They also recommend “considering” reducing indoor humidity, removing bedroom carpets, and washing stuffed toys weekly. The NAEPP Expert Panel cites many studies to support these recommendations.3

The National Environmental Education and Training Foundation (NEETF) 2005 guidelines recommend additional measures to reduce dust mite exposure including vacuuming using a high-efficiency particulate air (HEPA) filter, removing draperies, and considering using a portable air cleaner with a HEPA filter.4

STUDY SUMMARY: 54 trials, but no support for dust mite measures

This Cochrane systematic review included 54 randomized trials that assessed the effects of physical and/or chemical interventions to reduce exposure to house dust mite antigens in the homes of patients with mite-sensitive asthma. These studies included a total of 3002 pediatric and adult asthma patients (9 - 628 patients analyzed per trial) with mite sensitization confirmed by skin testing or IgE serum assays.

Thirty-six studies tested physical interventions, including mattress covers, vacuum cleaning, heating, ventilation, freezing, washing, air filtration, and ionizers. Ten used chemical interventions to kill dust mites; 8 used a combination of physical and chemical methods. Control groups received either placebo or no treatment.

Outcomes studied. The authors extracted data for the following outcomes: subjective well-being, asthma symptom scores, use of medication, days of sick leave from school or work, number of unscheduled visits to a physician or hospital, forced expiratory volume in 1 second (FEV1), peak expiratory flow rate (PEFR), and provocative concentration that causes a 20% fall in FEV1 (PC20). Length of the intervention and follow-up ranged from 2 weeks to 2 years.

Quality of studies. According to modern standards for randomized trials, the quality of many of the 54 studies was not optimal, especially in the descriptions of randomization and the reporting of outcomes. The method of randomization and concealment of allocation was rarely described. Eleven trial reports did not contain any usable data for the meta-analysis because of the way data were reported, and there was significant potential for reporting bias in favor of a treatment effect in the studies included. Mite reduction was successful in 17 trials, unsuccessful in 24 trials, and not reported in 13 trials.

Interventions didn’t help. There were no differences between the intervention and control groups for any of the outcomes. The percentage of patients who improved after the experimental interventions was not significantly different from the percentage of patients in the control groups (relative risk [RR]=1.01; 95% confidence interval [CI], 0.80-1.27; data based on 7 trials). There was no difference in medication usage (data from 10 trials), FEV1 (data from 14 trials), morning PEFR (data from 23 trials), or PC 20 (data from 14 trials) between the intervention and control groups ( TABLE ).1

TABLE
Dust mite control measures didn’t improve these outcomes

OUTCOMESTANDARDIZED MEAN DIFFERENCE* (95% CI)
Medication usage-0.06 (-0.18 to 0.07)
FEV10.11 (-0.05 to 0.28)
Morning PEFR0.00 (-1.0 to 0.10)
PC 200.05 (-0.13 to 0.22)
CI, confidence interval; FEV1, forced expiratory volume in 1 second; PC20, provocative concentration that causes a 20% fall in FEV1; PEFR, peak expiratory flow rate.
*Standardized mean difference is a common way to combine results of different studies for comparison purposes. If the 95% CI crosses 0, there is no effect of the intervention compared with the control.
 

 

 

WHAT’S NEW?: Nothing is new, yet this will be “news” to many

This Cochrane review includes 5 additional trials that have been conducted since the last Cochrane review of this topic in 2004. However, the 2004 review reported the same conclusion—that interventions to reduce house dust mite exposure in asthma patients are ineffective—as did 3 other Cochrane reviews on the same topic beginning in 1998.5-8

So why are the guidelines out of step? Schmidt and Gøtzsche (one of the authors of the Cochrane review) conducted a systematic review of narrative review articles in 2005 to answer this question. They found 70 review articles, 90% of which recommended physical methods to reduce exposure to house dust mites. They discovered that although these review articles included references to support their recommendations of dust mite control measures, the reviews showed significant bias in favor of positive studies and highlighted the results of low-quality studies, including non-randomized studies that had been excluded from the Cochrane reviews.9

CAVEATS: Duration of studies not long enough?

We know that extreme measures to reduce exposure to dust mite allergen, such as relocating to a high altitude or prolonged hospitalization, can reduce asthma symptoms,10,11 but these are clearly not practical solutions for most patients with dust mite-sensitive asthma. When it comes to this Cochrane review, some might argue that many of the interventions included were not of sufficient duration and did not sufficiently reduce the level of house mite allergen to improve asthma symptoms.

However, the subgroups of trials with long treatment duration (1-2 years) and successful mite reduction (determined by different methods, including mite counts and measured antigen levels in dust samples) also failed to show a significant difference between intervention and control groups.1

Tweak the approach? Most dust mite-sensitive asthma patients are sensitive to other allergens, so perhaps multifaceted interventions that target multiple allergens would be more effective.12 But until these potential interventions are supported by stronger evidence, we should not recommend them to our patients.

CHALLENGES TO IMPLEMENTATION: Swimming against the tide is never easy

Although the evidence to date indicates that interventions to reduce home dust mite exposure are ineffective, there are hundreds of products—including mattress and pillow covers ($10-$100), ionizers ($100-$200), and air filtration systems ($500-$800)—that are being marketed to patients with asthma. In addition, patient education handouts from sources such as the American Academy of Family Physicians, the American Academy of Pediatrics, and UpToDate recommend implementing dust mite control measures to reduce dust mite allergen exposure.13-15

We need to start educating our asthma patients properly so they can spend their time, energy, and money on interventions, such as medications, that work—and not on interventions that make no difference.

Acknowledgements

The PURLs Surveillance System is supported in part by Grant Number UL1RR024999 from the National Center for Research Resources, a Clinical Translational Science Award to the University of Chicago. The content is solely the responsibility of the authors and does not necessarily represent the official views of the National Center for Research Resources or the National Institutes of Health.

This study was selected and evaluated using FPIN’s Priority Updates from the Research Literature (PURL) Surveillance System methodology. The criteria and findings leading to the selection of this study as a PURL can be accessed at www.jfponline.com/purls.

 

ILLUSTRATIVE CASE

The parents of a 10-year-old patient whom you recently diagnosed with asthma want to do everything they can to reduce his asthma symptoms. They are considering buying hypoallergenic mattress covers and an expensive air filtration system to decrease the levels of dust mite allergens in their home and want to know if you think that will help their son. What do you tell them?

We want to do everything we can to help our patients control their asthma symptoms, but when it comes to household dust mite control measures, this extensive Cochrane review confirms that interventions like mattress covers and air filtration don’t work, despite recent reviews and guidelines recommending them.

Dust mites (Dermatophagoides pteronyssinus) are one of the most common allergens that provoke asthma symptoms in children and adults.2 Dust mites live in warm, humid places and feed on human skin scales. The areas with the highest levels of household infestation are carpets, mattresses, pillows, drapes, upholstered furniture, and clothing.

Guidelines still encourage mattress cover use

The National Asthma Education and Prevention Program (NAEPP) 2007 guidelines recommend using allergen-impermeable mattress and pillow covers and washing sheets and blankets in hot water. They also recommend “considering” reducing indoor humidity, removing bedroom carpets, and washing stuffed toys weekly. The NAEPP Expert Panel cites many studies to support these recommendations.3

The National Environmental Education and Training Foundation (NEETF) 2005 guidelines recommend additional measures to reduce dust mite exposure including vacuuming using a high-efficiency particulate air (HEPA) filter, removing draperies, and considering using a portable air cleaner with a HEPA filter.4

STUDY SUMMARY: 54 trials, but no support for dust mite measures

This Cochrane systematic review included 54 randomized trials that assessed the effects of physical and/or chemical interventions to reduce exposure to house dust mite antigens in the homes of patients with mite-sensitive asthma. These studies included a total of 3002 pediatric and adult asthma patients (9 - 628 patients analyzed per trial) with mite sensitization confirmed by skin testing or IgE serum assays.

Thirty-six studies tested physical interventions, including mattress covers, vacuum cleaning, heating, ventilation, freezing, washing, air filtration, and ionizers. Ten used chemical interventions to kill dust mites; 8 used a combination of physical and chemical methods. Control groups received either placebo or no treatment.

Outcomes studied. The authors extracted data for the following outcomes: subjective well-being, asthma symptom scores, use of medication, days of sick leave from school or work, number of unscheduled visits to a physician or hospital, forced expiratory volume in 1 second (FEV1), peak expiratory flow rate (PEFR), and provocative concentration that causes a 20% fall in FEV1 (PC20). Length of the intervention and follow-up ranged from 2 weeks to 2 years.

Quality of studies. According to modern standards for randomized trials, the quality of many of the 54 studies was not optimal, especially in the descriptions of randomization and the reporting of outcomes. The method of randomization and concealment of allocation was rarely described. Eleven trial reports did not contain any usable data for the meta-analysis because of the way data were reported, and there was significant potential for reporting bias in favor of a treatment effect in the studies included. Mite reduction was successful in 17 trials, unsuccessful in 24 trials, and not reported in 13 trials.

Interventions didn’t help. There were no differences between the intervention and control groups for any of the outcomes. The percentage of patients who improved after the experimental interventions was not significantly different from the percentage of patients in the control groups (relative risk [RR]=1.01; 95% confidence interval [CI], 0.80-1.27; data based on 7 trials). There was no difference in medication usage (data from 10 trials), FEV1 (data from 14 trials), morning PEFR (data from 23 trials), or PC 20 (data from 14 trials) between the intervention and control groups ( TABLE ).1

TABLE
Dust mite control measures didn’t improve these outcomes

OUTCOMESTANDARDIZED MEAN DIFFERENCE* (95% CI)
Medication usage-0.06 (-0.18 to 0.07)
FEV10.11 (-0.05 to 0.28)
Morning PEFR0.00 (-1.0 to 0.10)
PC 200.05 (-0.13 to 0.22)
CI, confidence interval; FEV1, forced expiratory volume in 1 second; PC20, provocative concentration that causes a 20% fall in FEV1; PEFR, peak expiratory flow rate.
*Standardized mean difference is a common way to combine results of different studies for comparison purposes. If the 95% CI crosses 0, there is no effect of the intervention compared with the control.
 

 

 

WHAT’S NEW?: Nothing is new, yet this will be “news” to many

This Cochrane review includes 5 additional trials that have been conducted since the last Cochrane review of this topic in 2004. However, the 2004 review reported the same conclusion—that interventions to reduce house dust mite exposure in asthma patients are ineffective—as did 3 other Cochrane reviews on the same topic beginning in 1998.5-8

So why are the guidelines out of step? Schmidt and Gøtzsche (one of the authors of the Cochrane review) conducted a systematic review of narrative review articles in 2005 to answer this question. They found 70 review articles, 90% of which recommended physical methods to reduce exposure to house dust mites. They discovered that although these review articles included references to support their recommendations of dust mite control measures, the reviews showed significant bias in favor of positive studies and highlighted the results of low-quality studies, including non-randomized studies that had been excluded from the Cochrane reviews.9

CAVEATS: Duration of studies not long enough?

We know that extreme measures to reduce exposure to dust mite allergen, such as relocating to a high altitude or prolonged hospitalization, can reduce asthma symptoms,10,11 but these are clearly not practical solutions for most patients with dust mite-sensitive asthma. When it comes to this Cochrane review, some might argue that many of the interventions included were not of sufficient duration and did not sufficiently reduce the level of house mite allergen to improve asthma symptoms.

However, the subgroups of trials with long treatment duration (1-2 years) and successful mite reduction (determined by different methods, including mite counts and measured antigen levels in dust samples) also failed to show a significant difference between intervention and control groups.1

Tweak the approach? Most dust mite-sensitive asthma patients are sensitive to other allergens, so perhaps multifaceted interventions that target multiple allergens would be more effective.12 But until these potential interventions are supported by stronger evidence, we should not recommend them to our patients.

CHALLENGES TO IMPLEMENTATION: Swimming against the tide is never easy

Although the evidence to date indicates that interventions to reduce home dust mite exposure are ineffective, there are hundreds of products—including mattress and pillow covers ($10-$100), ionizers ($100-$200), and air filtration systems ($500-$800)—that are being marketed to patients with asthma. In addition, patient education handouts from sources such as the American Academy of Family Physicians, the American Academy of Pediatrics, and UpToDate recommend implementing dust mite control measures to reduce dust mite allergen exposure.13-15

We need to start educating our asthma patients properly so they can spend their time, energy, and money on interventions, such as medications, that work—and not on interventions that make no difference.

Acknowledgements

The PURLs Surveillance System is supported in part by Grant Number UL1RR024999 from the National Center for Research Resources, a Clinical Translational Science Award to the University of Chicago. The content is solely the responsibility of the authors and does not necessarily represent the official views of the National Center for Research Resources or the National Institutes of Health.

This study was selected and evaluated using FPIN’s Priority Updates from the Research Literature (PURL) Surveillance System methodology. The criteria and findings leading to the selection of this study as a PURL can be accessed at www.jfponline.com/purls.

References

1. Gotzsche PC, Johansen HK. House dust mite control measures for asthma. Cochrane Database Syst Rev. 2008;(2):CD001187.-

2. German JA, Harper MB. Environmental control of allergic diseases. Am Fam Physician. 2002;66:421-426.

3. National Asthma Education and Prevention Program (NAEPP). Control of environmental factors and comorbid conditions that affect asthma. In: Expert panel report 3: guidelines for the diagnosis and management of asthma. Bethesda, Md: National Heart, Lung, and Blood Institute; 2007.

4. National Environmental Education & Training Foundation (NEETF). Environmental management of pediatric asthma. Guidelines for health care providers. Washington, DC: National Environmental Education & Training Foundation (NEETF); 2005.

5. Gøtzsche PC, Hammarquist C, Burr M. House dust mite control measures in the management of asthma: meta-analysis. BMJ. 1998;317:1105-1110.

6. Hammarquist C, Burr ML, Gotzsche PC. House dust mite control measures for asthma. Cochrane Database Syst Rev. 2000;(2):CD001187.-

7. Gøtzsche PC, Johansen HK, Burr ML, Hammarquist C. House dust mite control measures for asthma. Cochrane Database Syst Rev. 2001;(3):CD001187.-

8. Gøtzsche PC, Johansen HK, Schmidt LM, Burr ML. House dust mite control measures for asthma. Cochrane Database Syst Rev. 2004;(4):CD001187.-

9. Schmidt LM, Gøtzsche PC. Of mites and men: reference bias in narrative review articles: a systematic review. J Fam Pract. 2005;54:334-338.

10. Platts-Mills TA, Tovey ER, Mitchell EB, Moszoro H, Nock P, Wilkins SR. Reduction of bronchial hyperreactivity during prolonged allergen avoidance. Lancet 1982;2:675-678.

11. Grootendorst DC, Dahlen SE. Benefits of high altitude allergen avoidance in atopic adolescents with moderate to severe asthma, over and above treatment with high dose inhaled steroids. Clin Exp Allergy. 2001;31:400-408.

12. Morgan WJ, Crain EF, Gruchalla RS, et al. Results of a home-based environmental intervention among urban children with asthma. N Engl J Med. 2004;351:1068-1080.

13. American Academy of Family Physicians. Dust mites in the home [patient handout]. Available at: http://familydoctor.org/online/famdocen/home/common/asthma/triggers/683.html. Accessed October 23, 2008.

14. American Academy of Pediatrics. Non-pharmacologic approaches to asthma management [patient handout]. Available at: http://www.aap.org/sections/allergy/nonrxchild.pdf. Accessed October 23, 2008.

15. Bailey W. Patient information: Trigger avoidance in asthma. UpToDate [online database]. Version 16.2. Waltham, Mass: UpToDate; 2008.

References

1. Gotzsche PC, Johansen HK. House dust mite control measures for asthma. Cochrane Database Syst Rev. 2008;(2):CD001187.-

2. German JA, Harper MB. Environmental control of allergic diseases. Am Fam Physician. 2002;66:421-426.

3. National Asthma Education and Prevention Program (NAEPP). Control of environmental factors and comorbid conditions that affect asthma. In: Expert panel report 3: guidelines for the diagnosis and management of asthma. Bethesda, Md: National Heart, Lung, and Blood Institute; 2007.

4. National Environmental Education & Training Foundation (NEETF). Environmental management of pediatric asthma. Guidelines for health care providers. Washington, DC: National Environmental Education & Training Foundation (NEETF); 2005.

5. Gøtzsche PC, Hammarquist C, Burr M. House dust mite control measures in the management of asthma: meta-analysis. BMJ. 1998;317:1105-1110.

6. Hammarquist C, Burr ML, Gotzsche PC. House dust mite control measures for asthma. Cochrane Database Syst Rev. 2000;(2):CD001187.-

7. Gøtzsche PC, Johansen HK, Burr ML, Hammarquist C. House dust mite control measures for asthma. Cochrane Database Syst Rev. 2001;(3):CD001187.-

8. Gøtzsche PC, Johansen HK, Schmidt LM, Burr ML. House dust mite control measures for asthma. Cochrane Database Syst Rev. 2004;(4):CD001187.-

9. Schmidt LM, Gøtzsche PC. Of mites and men: reference bias in narrative review articles: a systematic review. J Fam Pract. 2005;54:334-338.

10. Platts-Mills TA, Tovey ER, Mitchell EB, Moszoro H, Nock P, Wilkins SR. Reduction of bronchial hyperreactivity during prolonged allergen avoidance. Lancet 1982;2:675-678.

11. Grootendorst DC, Dahlen SE. Benefits of high altitude allergen avoidance in atopic adolescents with moderate to severe asthma, over and above treatment with high dose inhaled steroids. Clin Exp Allergy. 2001;31:400-408.

12. Morgan WJ, Crain EF, Gruchalla RS, et al. Results of a home-based environmental intervention among urban children with asthma. N Engl J Med. 2004;351:1068-1080.

13. American Academy of Family Physicians. Dust mites in the home [patient handout]. Available at: http://familydoctor.org/online/famdocen/home/common/asthma/triggers/683.html. Accessed October 23, 2008.

14. American Academy of Pediatrics. Non-pharmacologic approaches to asthma management [patient handout]. Available at: http://www.aap.org/sections/allergy/nonrxchild.pdf. Accessed October 23, 2008.

15. Bailey W. Patient information: Trigger avoidance in asthma. UpToDate [online database]. Version 16.2. Waltham, Mass: UpToDate; 2008.

Issue
The Journal of Family Practice - 57(12)
Issue
The Journal of Family Practice - 57(12)
Page Number
789-792
Page Number
789-792
Publications
Publications
Topics
Article Type
Display Headline
Dust mite control measures don’t help asthma patients
Display Headline
Dust mite control measures don’t help asthma patients
Legacy Keywords
Sarah-Anne Schumann; mattress covers; filtration system; asthma symptoms; allergen-impermeable mattresses
Legacy Keywords
Sarah-Anne Schumann; mattress covers; filtration system; asthma symptoms; allergen-impermeable mattresses
Sections
PURLs Copyright

Copyright © 2008 The Family Physicians Inquiries Network.
All rights reserved.

Disallow All Ads
Alternative CME
Use ProPublica
Hide sidebar & use full width
render the right sidebar.
Conference Recap Checkbox
Not Conference Recap
Clinical Edge
Article PDF Media
Media Files

Can metformin undo weight gain induced by antipsychotics?

Article Type
Changed
Fri, 06/19/2020 - 10:41
Display Headline
Can metformin undo weight gain induced by antipsychotics?
 

Lack of evidence for weight loss drugs

The most recent guideline on this topic does not recommend any medication, citing a lack of evidence. In its 2003 consensus statement, a panel representing the American Diabetes Association, American Psychiatric Association, American Association of Clinical Endocrinologists, and the North American Association for the Study of Obesity3 recommends:

  • That patients taking second-generation antipsychotics have the following assessments at baseline and regular intervals: weight, height, waist circumference, blood pressure, fasting plasma glucose, and fasting lipids.
  • Providing nutrition and exercise counseling to all patients who are over-weight or obese at baseline.
  • Initiating treatment with one of the second-generation antipsychotics with a lower risk of weight gain for patients at high risk of diabetes (ie, family history) and for patients who gain 5% or more of their initial weight or develop worsening hyperglycemia or dyslipidemia during treatment.

This guideline does not recommend metformin to reduce weight gain.

A 2007 Cochrane review of interventions to reduce weight gain in patients with schizophrenia included 23 randomized controlled trials of a variety of weight loss interventions, including cognitive/behavioral interventions and a variety of medications, including sibutramine, orlistat, fluoxetine, topiramate, and metformin. The authors highlighted the limited number of studies of short duration and with small sample sizes and concluded that the evidence was insufficient for the use of pharmacologic interventions to prevent or treat weight gain.5

STUDY SUMMARY: Lifestyle changes and metformin compared

This randomized controlled trial was conducted in China and included 128 adults aged 18 to 45 with a first psychotic episode of schizophrenia. All patients had to have gained more than 10% of their pretreatment body weight during the first year of treatment with an antipsychotic medication (clozapine, olanzapine, risperidone, or sulpiride [not approved for use in the United States]). All study participants had to be under the care of an adult caregiver who monitored and recorded food intake, exercise, and medication intake. Patients with diabetes, cardiovascular disease, liver or renal dysfunction, substance abuse, or psychiatric diagnoses other than schizophrenia were excluded.

Patients were randomized to 1 of 4 groups for the 12 weeks of the study:

  • Metformin alone, 250 mg 3 times daily
  • Placebo alone
  • Lifestyle intervention plus metformin
  • Lifestyle intervention plus placebo

The lifestyle intervention included 3 components: (1) education: monthly programs on nutrition and physical activity; (2) diet: the American Heart Association step 2 diet (<30% calories from fat, 55% carbohydrates, >15% protein, with at least 15 g fiber per 1000 kcal); and (3) exercise: 1 week of sessions with an exercise physiologist followed by an individualized home-based exercise program.

Primary outcomes included changes in weight, body mass index (BMI), waist circumference, and fasting glucose ( TABLE 2 ). Ten of the 128 randomized patients either discontinued the study or were lost to follow up, but all 128 patients were included in the analysis.

TABLE 2
Mean difference between baseline and endpoint (week 12) of treatment outcomes
(95% confidence intervals)1

 LIFESTYLE + METFORMINMETFORMINLIFESTYLEPLACEBO
Weight, kg-4.7 (-5.7 to -3.4)-3.2 (-3.9 to -2.5)-1.4 (-2.0 to -0.7)3.1 (2.4 to 3.8)
BMI, kg/m2-1.8 (-2.3 to -1.3)-1.2 (-1.5 to -0.9)-0.5 (-0.8 to -0.3)1.2 (0.9 to 1.5)
Waist circumference, cm-2.0 (-2.4 to -1.5)-1.3 (-1.5 to -1.1)0.1 (-0.5 to 0.7)2.2 (1.7 to 2.8)
Fasting glucose, mg/dL-7.2 (-10.8 to -5.4)-10.8 (-16.2 to -7.2)-7.2 (-9.0 to -3.6)1.8 (-1.8 to 3.6)

Best result: Lifestyle changes plus metformin

Compared with baseline, weight decreased by 7.3% in the lifestyle plus metformin group, by 4.9% in the metformin-only group, and by 2.2% in the lifestyle-only group; in the placebo group, weight increased by 4.8%.

Participants in all 3 intervention groups also showed significant decreases in the mean fasting glucose, insulin levels, and insulin resistance index (IRI). The insulin levels and the IRI increased in the placebo group.

No significant differences in adverse effects were noted among the 4 treatment groups.1

 

 

 

WHAT’S NEW: Convincing evidence

This is the first randomized controlled trial to show convincingly that metformin alone or in combination with lifestyle changes is superior to lifestyle changes alone or placebo for reducing weight gain and other adverse metabolic outcomes induced by second-generation antipsychotics.

Intensive lifestyle interventions

Prior studies found that intensive lifestyle interventions can help reduce antipsychotic-related weight gain. A 3-month randomized controlled trial compared an early behavioral intervention (dietary counseling, an exercise program, and behavior therapy) with routine care in 61 patients with first-episode psychosis who were taking risperidone, olanzapine, or haloperidol;6 significantly fewer patients assigned to behavioral intervention had an increased initial body weight of more than 7%: 39% in the behavioral intervention group vs 79% in the routine care group (P<.002).

Small samples, small effect sizes

Past studies of metformin for antipsychotic-associated weight gain have generally shown a small benefit, though small sample sizes and small effect sizes prohibited definitive conclusions. Unlike the study by Wu and colleagues,1 none of these past studies were designed to compare the combination of metformin and lifestyle intervention with metformin alone, lifestyle intervention alone, or placebo alone.

Klein et al conducted a randomized placebo-controlled trial of metformin in 39 children ages 10 to 17 whose weight had increased more than 10% on atypical antipsychotic therapy.7 The children treated with placebo gained a mean of 4 kg and increased their mean BMI by 1.12 kg/m2 during 16 weeks of treatment, while those in the metformin group did not gain weight and decreased their mean BMI by 0.43 kg/m2.

Baptista et al randomized 40 in-patients with schizophrenia, who were being switched from conventional antipsychotics to olanzapine, to either metformin (850-1750 mg/d) or placebo. Both groups gained a similar amount of weight after the 14-week study (5.5 vs 6.3 kg, metformin vs placebo). Three patients who started with high fasting glucose had decreases while taking metformin, and 3 patients given placebo developed elevated fasting glucose during the study.8

In another randomized controlled trial of metformin vs placebo in 80 patients who had been taking olanzapine for at least 4 months, Baptista et al found only a small, insignificant difference in weight loss after 12 weeks of treatment (metformin group lost 1.4 kg, placebo group lost 0.18 kg, P=.09). They reported that both groups were highly motivated to lose weight and were compliant with the healthy lifestyle recommendations.9

An adequately powered study

The trial1 highlighted in this PURL had an adequate sample size to compare metformin plus a lifestyle intervention with either treatment alone or placebo. It showed a clinically important effect of metformin both by itself and in conjunction with the lifestyle intervention.

CAVEATS: Consider switching drugs

Before adding metformin to help with weight loss, primary care clinicians should contact the patient’s psychiatrist to discuss the option of switching antipsychotic medications. Switching from a medication with a higher risk for weight gain, such as olanzapine, to one with a lower risk, such as aripiprazole or ziprasidone, can lead to significant weight loss.10

Not an option for some

However, some patients, especially those taking clozapine, may have already tried multiple antipsychotic agents without success, and switching is not an option for them.

Prescribing metformin

CHALLENGES TO IMPLEMENTATION: Adherence

These study participants were under the care of an adult caregiver who monitored and recorded food and medication intake and exercise level. The lifestyle intervention was thorough and structured and this kind of program is often not available to us for our patients. As a consequence, we may not obtain the same results as in this study. However, even the metformin-alone group showed improvements, and if our patients can reliably take their second-generation antipsychotic, they should also be able to take metformin reliably.

Patient resistance

Some patients may resist taking an additional medication to treat the side effects of their antipsychotic medication. Taking the time to educate them about the increased risk of diabetes and cardiovascular disease related to weight gain may help convince them to do so. Warn them about possible gastrointestinal adverse effects of metformin, which tend to lessen or disappear with time.

The PURLs Surveillance System is supported in part by Grant Number UL1RR024999 from the National Center For Research Resources, a Clinical Translational Science Award to the University of Chicago. The content is solely the responsibility of the authors and does not necessarily represent the official views of the National Center For Research Resources or the National Institutes of Health.

PURLs methodology

This study was selected and evaluated using FPIN’s Priority Updates from the Research Literature (PURL) Surveillance System methodology. The criteria and findings leading to the selection of this study as a PURL can be accessed at www.jfponline.com/purls.

Files
References

1. Wu R-R, Zhao J-P, Jin H, et al. Lifestyle intervention and metformin for treatment of antipsychotic-induced weight gain: a randomized controlled trial. JAMA. 2008;299:185-193.

2. Rowland K, Schumann SA. Have pedometer, will travel. J Fam Pract. 2008;57:90-93.

3. American Diabetes Association; American Psychiatric Association; American Association of Clinical Endocrinologists; North American Association for the Study of Obesity. Consensus development conference on antipsychotic drugs and obesity and diabetes. Diabetes Care. 2004;27:596-601.

4. Newcomer JW. Metabolic considerations in the use of antipsychotic medications: a review of recent evidence. J Clin Psychiatry. 2007;68(suppl 1):20-27.

5. Faulkner G, Cohn T, Remington G. Interventions to reduce weight gain in schizophrenia. Cochrane Database Syst Rev. 2007;(1):CD005148.-

6. Alvarez-Jiménez M, González-Blanch C, Vázquez-Barquero JL, et al. Attenuation of antipsychotic-induced weight gain with early behavioral intervention in drug-naïve first-episode psychosis patients: a randomized controlled trial. J Clin Psychiatry. 2006;67:1253-1260.

7. Klein DJ, Cottingham EM, Sorter M, Barton BA, Morrison JA. A randomized, double-blind, placebo-controlled trial of metformin treatment of weight gain associated with initiation of atypical antipsychotic therapy in children and adolescents. Am J Psychiatry. 2006;163:2072-2079.

8. Baptista T, Martínez J, Lacruz A, et al. Metformin for prevention of weight gain and insulin resistance with olanzapine: a double-blind placebo-controlled trial. Can J Psychiatry. 2006;51:192-196.

9. Baptista T, Rangel N, Fernández V, et al. Metformin as an adjunctive treatment to control body weight and metabolic dysfunction during olanzapine administration: a multicentric, double-blind, placebo-controlled trial. Schizophrenia Res. 2007;93:99-108.

10. Weiden PJ. Switching antipsychotics as a treatment strategy for antipsychotic-induced weight gain and dyslipidemia. J Clin Psychiatry. 2007;68(suppl 4):34-39.

Article PDF
Author and Disclosure Information

Sarah-Anne Schumann, MD
Bernard Ewigman, MD, MSPH
Department of Family Medicine, The University of Chicago

PURLs EDITOR
John Hickner, MD, MSc
Department of Family Medicine, The University of Chicago

Issue
The Journal of Family Practice - 57(8)
Publications
Topics
Page Number
515-525
Legacy Keywords
metformin; antipsychotic; weight gain; obesity and schizophrenia; diabetes and schizophrenia; olanzapine; clozapine; risperidone; quetiapine; aripiprazole; ziprasidone; Sarah-Anne Schumann MD; Bernard Ewigman MD; John Hickner MD
Sections
Files
Files
Author and Disclosure Information

Sarah-Anne Schumann, MD
Bernard Ewigman, MD, MSPH
Department of Family Medicine, The University of Chicago

PURLs EDITOR
John Hickner, MD, MSc
Department of Family Medicine, The University of Chicago

Author and Disclosure Information

Sarah-Anne Schumann, MD
Bernard Ewigman, MD, MSPH
Department of Family Medicine, The University of Chicago

PURLs EDITOR
John Hickner, MD, MSc
Department of Family Medicine, The University of Chicago

Article PDF
Article PDF
 

Lack of evidence for weight loss drugs

The most recent guideline on this topic does not recommend any medication, citing a lack of evidence. In its 2003 consensus statement, a panel representing the American Diabetes Association, American Psychiatric Association, American Association of Clinical Endocrinologists, and the North American Association for the Study of Obesity3 recommends:

  • That patients taking second-generation antipsychotics have the following assessments at baseline and regular intervals: weight, height, waist circumference, blood pressure, fasting plasma glucose, and fasting lipids.
  • Providing nutrition and exercise counseling to all patients who are over-weight or obese at baseline.
  • Initiating treatment with one of the second-generation antipsychotics with a lower risk of weight gain for patients at high risk of diabetes (ie, family history) and for patients who gain 5% or more of their initial weight or develop worsening hyperglycemia or dyslipidemia during treatment.

This guideline does not recommend metformin to reduce weight gain.

A 2007 Cochrane review of interventions to reduce weight gain in patients with schizophrenia included 23 randomized controlled trials of a variety of weight loss interventions, including cognitive/behavioral interventions and a variety of medications, including sibutramine, orlistat, fluoxetine, topiramate, and metformin. The authors highlighted the limited number of studies of short duration and with small sample sizes and concluded that the evidence was insufficient for the use of pharmacologic interventions to prevent or treat weight gain.5

STUDY SUMMARY: Lifestyle changes and metformin compared

This randomized controlled trial was conducted in China and included 128 adults aged 18 to 45 with a first psychotic episode of schizophrenia. All patients had to have gained more than 10% of their pretreatment body weight during the first year of treatment with an antipsychotic medication (clozapine, olanzapine, risperidone, or sulpiride [not approved for use in the United States]). All study participants had to be under the care of an adult caregiver who monitored and recorded food intake, exercise, and medication intake. Patients with diabetes, cardiovascular disease, liver or renal dysfunction, substance abuse, or psychiatric diagnoses other than schizophrenia were excluded.

Patients were randomized to 1 of 4 groups for the 12 weeks of the study:

  • Metformin alone, 250 mg 3 times daily
  • Placebo alone
  • Lifestyle intervention plus metformin
  • Lifestyle intervention plus placebo

The lifestyle intervention included 3 components: (1) education: monthly programs on nutrition and physical activity; (2) diet: the American Heart Association step 2 diet (<30% calories from fat, 55% carbohydrates, >15% protein, with at least 15 g fiber per 1000 kcal); and (3) exercise: 1 week of sessions with an exercise physiologist followed by an individualized home-based exercise program.

Primary outcomes included changes in weight, body mass index (BMI), waist circumference, and fasting glucose ( TABLE 2 ). Ten of the 128 randomized patients either discontinued the study or were lost to follow up, but all 128 patients were included in the analysis.

TABLE 2
Mean difference between baseline and endpoint (week 12) of treatment outcomes
(95% confidence intervals)1

 LIFESTYLE + METFORMINMETFORMINLIFESTYLEPLACEBO
Weight, kg-4.7 (-5.7 to -3.4)-3.2 (-3.9 to -2.5)-1.4 (-2.0 to -0.7)3.1 (2.4 to 3.8)
BMI, kg/m2-1.8 (-2.3 to -1.3)-1.2 (-1.5 to -0.9)-0.5 (-0.8 to -0.3)1.2 (0.9 to 1.5)
Waist circumference, cm-2.0 (-2.4 to -1.5)-1.3 (-1.5 to -1.1)0.1 (-0.5 to 0.7)2.2 (1.7 to 2.8)
Fasting glucose, mg/dL-7.2 (-10.8 to -5.4)-10.8 (-16.2 to -7.2)-7.2 (-9.0 to -3.6)1.8 (-1.8 to 3.6)

Best result: Lifestyle changes plus metformin

Compared with baseline, weight decreased by 7.3% in the lifestyle plus metformin group, by 4.9% in the metformin-only group, and by 2.2% in the lifestyle-only group; in the placebo group, weight increased by 4.8%.

Participants in all 3 intervention groups also showed significant decreases in the mean fasting glucose, insulin levels, and insulin resistance index (IRI). The insulin levels and the IRI increased in the placebo group.

No significant differences in adverse effects were noted among the 4 treatment groups.1

 

 

 

WHAT’S NEW: Convincing evidence

This is the first randomized controlled trial to show convincingly that metformin alone or in combination with lifestyle changes is superior to lifestyle changes alone or placebo for reducing weight gain and other adverse metabolic outcomes induced by second-generation antipsychotics.

Intensive lifestyle interventions

Prior studies found that intensive lifestyle interventions can help reduce antipsychotic-related weight gain. A 3-month randomized controlled trial compared an early behavioral intervention (dietary counseling, an exercise program, and behavior therapy) with routine care in 61 patients with first-episode psychosis who were taking risperidone, olanzapine, or haloperidol;6 significantly fewer patients assigned to behavioral intervention had an increased initial body weight of more than 7%: 39% in the behavioral intervention group vs 79% in the routine care group (P<.002).

Small samples, small effect sizes

Past studies of metformin for antipsychotic-associated weight gain have generally shown a small benefit, though small sample sizes and small effect sizes prohibited definitive conclusions. Unlike the study by Wu and colleagues,1 none of these past studies were designed to compare the combination of metformin and lifestyle intervention with metformin alone, lifestyle intervention alone, or placebo alone.

Klein et al conducted a randomized placebo-controlled trial of metformin in 39 children ages 10 to 17 whose weight had increased more than 10% on atypical antipsychotic therapy.7 The children treated with placebo gained a mean of 4 kg and increased their mean BMI by 1.12 kg/m2 during 16 weeks of treatment, while those in the metformin group did not gain weight and decreased their mean BMI by 0.43 kg/m2.

Baptista et al randomized 40 in-patients with schizophrenia, who were being switched from conventional antipsychotics to olanzapine, to either metformin (850-1750 mg/d) or placebo. Both groups gained a similar amount of weight after the 14-week study (5.5 vs 6.3 kg, metformin vs placebo). Three patients who started with high fasting glucose had decreases while taking metformin, and 3 patients given placebo developed elevated fasting glucose during the study.8

In another randomized controlled trial of metformin vs placebo in 80 patients who had been taking olanzapine for at least 4 months, Baptista et al found only a small, insignificant difference in weight loss after 12 weeks of treatment (metformin group lost 1.4 kg, placebo group lost 0.18 kg, P=.09). They reported that both groups were highly motivated to lose weight and were compliant with the healthy lifestyle recommendations.9

An adequately powered study

The trial1 highlighted in this PURL had an adequate sample size to compare metformin plus a lifestyle intervention with either treatment alone or placebo. It showed a clinically important effect of metformin both by itself and in conjunction with the lifestyle intervention.

CAVEATS: Consider switching drugs

Before adding metformin to help with weight loss, primary care clinicians should contact the patient’s psychiatrist to discuss the option of switching antipsychotic medications. Switching from a medication with a higher risk for weight gain, such as olanzapine, to one with a lower risk, such as aripiprazole or ziprasidone, can lead to significant weight loss.10

Not an option for some

However, some patients, especially those taking clozapine, may have already tried multiple antipsychotic agents without success, and switching is not an option for them.

Prescribing metformin

CHALLENGES TO IMPLEMENTATION: Adherence

These study participants were under the care of an adult caregiver who monitored and recorded food and medication intake and exercise level. The lifestyle intervention was thorough and structured and this kind of program is often not available to us for our patients. As a consequence, we may not obtain the same results as in this study. However, even the metformin-alone group showed improvements, and if our patients can reliably take their second-generation antipsychotic, they should also be able to take metformin reliably.

Patient resistance

Some patients may resist taking an additional medication to treat the side effects of their antipsychotic medication. Taking the time to educate them about the increased risk of diabetes and cardiovascular disease related to weight gain may help convince them to do so. Warn them about possible gastrointestinal adverse effects of metformin, which tend to lessen or disappear with time.

The PURLs Surveillance System is supported in part by Grant Number UL1RR024999 from the National Center For Research Resources, a Clinical Translational Science Award to the University of Chicago. The content is solely the responsibility of the authors and does not necessarily represent the official views of the National Center For Research Resources or the National Institutes of Health.

PURLs methodology

This study was selected and evaluated using FPIN’s Priority Updates from the Research Literature (PURL) Surveillance System methodology. The criteria and findings leading to the selection of this study as a PURL can be accessed at www.jfponline.com/purls.

 

Lack of evidence for weight loss drugs

The most recent guideline on this topic does not recommend any medication, citing a lack of evidence. In its 2003 consensus statement, a panel representing the American Diabetes Association, American Psychiatric Association, American Association of Clinical Endocrinologists, and the North American Association for the Study of Obesity3 recommends:

  • That patients taking second-generation antipsychotics have the following assessments at baseline and regular intervals: weight, height, waist circumference, blood pressure, fasting plasma glucose, and fasting lipids.
  • Providing nutrition and exercise counseling to all patients who are over-weight or obese at baseline.
  • Initiating treatment with one of the second-generation antipsychotics with a lower risk of weight gain for patients at high risk of diabetes (ie, family history) and for patients who gain 5% or more of their initial weight or develop worsening hyperglycemia or dyslipidemia during treatment.

This guideline does not recommend metformin to reduce weight gain.

A 2007 Cochrane review of interventions to reduce weight gain in patients with schizophrenia included 23 randomized controlled trials of a variety of weight loss interventions, including cognitive/behavioral interventions and a variety of medications, including sibutramine, orlistat, fluoxetine, topiramate, and metformin. The authors highlighted the limited number of studies of short duration and with small sample sizes and concluded that the evidence was insufficient for the use of pharmacologic interventions to prevent or treat weight gain.5

STUDY SUMMARY: Lifestyle changes and metformin compared

This randomized controlled trial was conducted in China and included 128 adults aged 18 to 45 with a first psychotic episode of schizophrenia. All patients had to have gained more than 10% of their pretreatment body weight during the first year of treatment with an antipsychotic medication (clozapine, olanzapine, risperidone, or sulpiride [not approved for use in the United States]). All study participants had to be under the care of an adult caregiver who monitored and recorded food intake, exercise, and medication intake. Patients with diabetes, cardiovascular disease, liver or renal dysfunction, substance abuse, or psychiatric diagnoses other than schizophrenia were excluded.

Patients were randomized to 1 of 4 groups for the 12 weeks of the study:

  • Metformin alone, 250 mg 3 times daily
  • Placebo alone
  • Lifestyle intervention plus metformin
  • Lifestyle intervention plus placebo

The lifestyle intervention included 3 components: (1) education: monthly programs on nutrition and physical activity; (2) diet: the American Heart Association step 2 diet (<30% calories from fat, 55% carbohydrates, >15% protein, with at least 15 g fiber per 1000 kcal); and (3) exercise: 1 week of sessions with an exercise physiologist followed by an individualized home-based exercise program.

Primary outcomes included changes in weight, body mass index (BMI), waist circumference, and fasting glucose ( TABLE 2 ). Ten of the 128 randomized patients either discontinued the study or were lost to follow up, but all 128 patients were included in the analysis.

TABLE 2
Mean difference between baseline and endpoint (week 12) of treatment outcomes
(95% confidence intervals)1

 LIFESTYLE + METFORMINMETFORMINLIFESTYLEPLACEBO
Weight, kg-4.7 (-5.7 to -3.4)-3.2 (-3.9 to -2.5)-1.4 (-2.0 to -0.7)3.1 (2.4 to 3.8)
BMI, kg/m2-1.8 (-2.3 to -1.3)-1.2 (-1.5 to -0.9)-0.5 (-0.8 to -0.3)1.2 (0.9 to 1.5)
Waist circumference, cm-2.0 (-2.4 to -1.5)-1.3 (-1.5 to -1.1)0.1 (-0.5 to 0.7)2.2 (1.7 to 2.8)
Fasting glucose, mg/dL-7.2 (-10.8 to -5.4)-10.8 (-16.2 to -7.2)-7.2 (-9.0 to -3.6)1.8 (-1.8 to 3.6)

Best result: Lifestyle changes plus metformin

Compared with baseline, weight decreased by 7.3% in the lifestyle plus metformin group, by 4.9% in the metformin-only group, and by 2.2% in the lifestyle-only group; in the placebo group, weight increased by 4.8%.

Participants in all 3 intervention groups also showed significant decreases in the mean fasting glucose, insulin levels, and insulin resistance index (IRI). The insulin levels and the IRI increased in the placebo group.

No significant differences in adverse effects were noted among the 4 treatment groups.1

 

 

 

WHAT’S NEW: Convincing evidence

This is the first randomized controlled trial to show convincingly that metformin alone or in combination with lifestyle changes is superior to lifestyle changes alone or placebo for reducing weight gain and other adverse metabolic outcomes induced by second-generation antipsychotics.

Intensive lifestyle interventions

Prior studies found that intensive lifestyle interventions can help reduce antipsychotic-related weight gain. A 3-month randomized controlled trial compared an early behavioral intervention (dietary counseling, an exercise program, and behavior therapy) with routine care in 61 patients with first-episode psychosis who were taking risperidone, olanzapine, or haloperidol;6 significantly fewer patients assigned to behavioral intervention had an increased initial body weight of more than 7%: 39% in the behavioral intervention group vs 79% in the routine care group (P<.002).

Small samples, small effect sizes

Past studies of metformin for antipsychotic-associated weight gain have generally shown a small benefit, though small sample sizes and small effect sizes prohibited definitive conclusions. Unlike the study by Wu and colleagues,1 none of these past studies were designed to compare the combination of metformin and lifestyle intervention with metformin alone, lifestyle intervention alone, or placebo alone.

Klein et al conducted a randomized placebo-controlled trial of metformin in 39 children ages 10 to 17 whose weight had increased more than 10% on atypical antipsychotic therapy.7 The children treated with placebo gained a mean of 4 kg and increased their mean BMI by 1.12 kg/m2 during 16 weeks of treatment, while those in the metformin group did not gain weight and decreased their mean BMI by 0.43 kg/m2.

Baptista et al randomized 40 in-patients with schizophrenia, who were being switched from conventional antipsychotics to olanzapine, to either metformin (850-1750 mg/d) or placebo. Both groups gained a similar amount of weight after the 14-week study (5.5 vs 6.3 kg, metformin vs placebo). Three patients who started with high fasting glucose had decreases while taking metformin, and 3 patients given placebo developed elevated fasting glucose during the study.8

In another randomized controlled trial of metformin vs placebo in 80 patients who had been taking olanzapine for at least 4 months, Baptista et al found only a small, insignificant difference in weight loss after 12 weeks of treatment (metformin group lost 1.4 kg, placebo group lost 0.18 kg, P=.09). They reported that both groups were highly motivated to lose weight and were compliant with the healthy lifestyle recommendations.9

An adequately powered study

The trial1 highlighted in this PURL had an adequate sample size to compare metformin plus a lifestyle intervention with either treatment alone or placebo. It showed a clinically important effect of metformin both by itself and in conjunction with the lifestyle intervention.

CAVEATS: Consider switching drugs

Before adding metformin to help with weight loss, primary care clinicians should contact the patient’s psychiatrist to discuss the option of switching antipsychotic medications. Switching from a medication with a higher risk for weight gain, such as olanzapine, to one with a lower risk, such as aripiprazole or ziprasidone, can lead to significant weight loss.10

Not an option for some

However, some patients, especially those taking clozapine, may have already tried multiple antipsychotic agents without success, and switching is not an option for them.

Prescribing metformin

CHALLENGES TO IMPLEMENTATION: Adherence

These study participants were under the care of an adult caregiver who monitored and recorded food and medication intake and exercise level. The lifestyle intervention was thorough and structured and this kind of program is often not available to us for our patients. As a consequence, we may not obtain the same results as in this study. However, even the metformin-alone group showed improvements, and if our patients can reliably take their second-generation antipsychotic, they should also be able to take metformin reliably.

Patient resistance

Some patients may resist taking an additional medication to treat the side effects of their antipsychotic medication. Taking the time to educate them about the increased risk of diabetes and cardiovascular disease related to weight gain may help convince them to do so. Warn them about possible gastrointestinal adverse effects of metformin, which tend to lessen or disappear with time.

The PURLs Surveillance System is supported in part by Grant Number UL1RR024999 from the National Center For Research Resources, a Clinical Translational Science Award to the University of Chicago. The content is solely the responsibility of the authors and does not necessarily represent the official views of the National Center For Research Resources or the National Institutes of Health.

PURLs methodology

This study was selected and evaluated using FPIN’s Priority Updates from the Research Literature (PURL) Surveillance System methodology. The criteria and findings leading to the selection of this study as a PURL can be accessed at www.jfponline.com/purls.

References

1. Wu R-R, Zhao J-P, Jin H, et al. Lifestyle intervention and metformin for treatment of antipsychotic-induced weight gain: a randomized controlled trial. JAMA. 2008;299:185-193.

2. Rowland K, Schumann SA. Have pedometer, will travel. J Fam Pract. 2008;57:90-93.

3. American Diabetes Association; American Psychiatric Association; American Association of Clinical Endocrinologists; North American Association for the Study of Obesity. Consensus development conference on antipsychotic drugs and obesity and diabetes. Diabetes Care. 2004;27:596-601.

4. Newcomer JW. Metabolic considerations in the use of antipsychotic medications: a review of recent evidence. J Clin Psychiatry. 2007;68(suppl 1):20-27.

5. Faulkner G, Cohn T, Remington G. Interventions to reduce weight gain in schizophrenia. Cochrane Database Syst Rev. 2007;(1):CD005148.-

6. Alvarez-Jiménez M, González-Blanch C, Vázquez-Barquero JL, et al. Attenuation of antipsychotic-induced weight gain with early behavioral intervention in drug-naïve first-episode psychosis patients: a randomized controlled trial. J Clin Psychiatry. 2006;67:1253-1260.

7. Klein DJ, Cottingham EM, Sorter M, Barton BA, Morrison JA. A randomized, double-blind, placebo-controlled trial of metformin treatment of weight gain associated with initiation of atypical antipsychotic therapy in children and adolescents. Am J Psychiatry. 2006;163:2072-2079.

8. Baptista T, Martínez J, Lacruz A, et al. Metformin for prevention of weight gain and insulin resistance with olanzapine: a double-blind placebo-controlled trial. Can J Psychiatry. 2006;51:192-196.

9. Baptista T, Rangel N, Fernández V, et al. Metformin as an adjunctive treatment to control body weight and metabolic dysfunction during olanzapine administration: a multicentric, double-blind, placebo-controlled trial. Schizophrenia Res. 2007;93:99-108.

10. Weiden PJ. Switching antipsychotics as a treatment strategy for antipsychotic-induced weight gain and dyslipidemia. J Clin Psychiatry. 2007;68(suppl 4):34-39.

References

1. Wu R-R, Zhao J-P, Jin H, et al. Lifestyle intervention and metformin for treatment of antipsychotic-induced weight gain: a randomized controlled trial. JAMA. 2008;299:185-193.

2. Rowland K, Schumann SA. Have pedometer, will travel. J Fam Pract. 2008;57:90-93.

3. American Diabetes Association; American Psychiatric Association; American Association of Clinical Endocrinologists; North American Association for the Study of Obesity. Consensus development conference on antipsychotic drugs and obesity and diabetes. Diabetes Care. 2004;27:596-601.

4. Newcomer JW. Metabolic considerations in the use of antipsychotic medications: a review of recent evidence. J Clin Psychiatry. 2007;68(suppl 1):20-27.

5. Faulkner G, Cohn T, Remington G. Interventions to reduce weight gain in schizophrenia. Cochrane Database Syst Rev. 2007;(1):CD005148.-

6. Alvarez-Jiménez M, González-Blanch C, Vázquez-Barquero JL, et al. Attenuation of antipsychotic-induced weight gain with early behavioral intervention in drug-naïve first-episode psychosis patients: a randomized controlled trial. J Clin Psychiatry. 2006;67:1253-1260.

7. Klein DJ, Cottingham EM, Sorter M, Barton BA, Morrison JA. A randomized, double-blind, placebo-controlled trial of metformin treatment of weight gain associated with initiation of atypical antipsychotic therapy in children and adolescents. Am J Psychiatry. 2006;163:2072-2079.

8. Baptista T, Martínez J, Lacruz A, et al. Metformin for prevention of weight gain and insulin resistance with olanzapine: a double-blind placebo-controlled trial. Can J Psychiatry. 2006;51:192-196.

9. Baptista T, Rangel N, Fernández V, et al. Metformin as an adjunctive treatment to control body weight and metabolic dysfunction during olanzapine administration: a multicentric, double-blind, placebo-controlled trial. Schizophrenia Res. 2007;93:99-108.

10. Weiden PJ. Switching antipsychotics as a treatment strategy for antipsychotic-induced weight gain and dyslipidemia. J Clin Psychiatry. 2007;68(suppl 4):34-39.

Issue
The Journal of Family Practice - 57(8)
Issue
The Journal of Family Practice - 57(8)
Page Number
515-525
Page Number
515-525
Publications
Publications
Topics
Article Type
Display Headline
Can metformin undo weight gain induced by antipsychotics?
Display Headline
Can metformin undo weight gain induced by antipsychotics?
Legacy Keywords
metformin; antipsychotic; weight gain; obesity and schizophrenia; diabetes and schizophrenia; olanzapine; clozapine; risperidone; quetiapine; aripiprazole; ziprasidone; Sarah-Anne Schumann MD; Bernard Ewigman MD; John Hickner MD
Legacy Keywords
metformin; antipsychotic; weight gain; obesity and schizophrenia; diabetes and schizophrenia; olanzapine; clozapine; risperidone; quetiapine; aripiprazole; ziprasidone; Sarah-Anne Schumann MD; Bernard Ewigman MD; John Hickner MD
Sections
PURLs Copyright

Copyright © 2008 The Family Physicians Inquiries Network.
All rights reserved.

Disallow All Ads
Alternative CME
Use ProPublica
Hide sidebar & use full width
render the right sidebar.
Conference Recap Checkbox
Not Conference Recap
Clinical Edge
Article PDF Media
Media Files

Patients insist on antibiotics for sinusitis? Here is a good reason to say “no”

Article Type
Changed
Fri, 06/19/2020 - 10:58
Display Headline
Patients insist on antibiotics for sinusitis? Here is a good reason to say “no”
 

ILLUSTRATIVE CASE

A 23-year-old woman presents to your office with a 1-week history of cough, purulent nasal discharge, and unilateral facial pain. You diagnose acute sinusitis.

Should you prescribe an antibiotic?

No. Yet it’s no wonder that most adults treated for acute sinusitis leave the doctor’s office with a prescription for antibiotics. Until the publication of the meta-analysis by Young and colleagues1 featured in this PURL, we have lacked A-level evidence from studies conducted in realistic settings—like your practice and ours.

Review of serial data from the National Ambulatory Medical Care Surveys (NAMCS) from 1999 through 2005 does show a slight downward trend in antibiotic prescribing for acute sinusitis: 1999-2002 data showed that 83% of cases of acute sinusitis were treated with an antibiotic.2 Data from the 2004 and 2005 NAMCS reveal that family physicians prescribed antibiotics for 80% of patients with acute sinusitis in 2004 and 76% of patients in 2005 (S. Medvedev, unpublished data, NAMCS database, March 2008).

Is this continued high rate of antibiotic prescribing justified?

Do antibiotics improve symptoms and shorten the duration of illness or not?

These questions are important, obviously, not only because of the high rate of prescribing but also because sinusitis is one of the most common diagnoses: approximately 20 million cases annually in the United States, or about 21% of all outpatient antibiotic prescriptions for adults.2

Which patients might benefit from antibiotics?

Common clinical signs and symptoms cannot identify patients with rhinosinusitis for whom treatment is clearly justified, given the cost, adverse events, and bacterial resistance associated with antibiotic use

  • Severity of symptoms is important only in that signs suggestive of a serious complication are the sole reason for immediate antibiotic treatment
  • Purulent discharge noted in the pharynx on exam was associated with a higher likelihood of benefit from antibiotics, but NNT was 8
  • Antibiotics are not justified even if a patient reports having symptoms for longer than 7-10 days

Source: Young et al.1

A diagnostic dilemma

Before we discuss the evidence that is summarized in the excellent meta-analysis by Young and colleagues,1 let’s acknowledge that acute sinusitis is undeniably a diagnostic dilemma. Distinguishing bacterial from viral infection is nearly impossible on clinical grounds because the symptoms are so similar. A litany of identical upper respiratory symptoms accompanies both viral and bacterial sinus infections. Purulent nasal secretions, maxillary facial pain (especially with unilateral predominance), maxillary tooth pain (which is uncommon with sinus infection), altered sense of smell, and worsening illness after improvement constitute the short list of signs and symptoms that has some predictive value, but even the presence of all of these is not a terrific predictor of bacterial sinus infection. Plain x-rays have low accuracy in distinguishing viral from bacterial infection. Computed tomography (CT) sinus scans are better but far from perfect, are not readily available in practice, and are expensive.

Sinusitis in the real world

How effective are antibiotics for patients diagnosed not by sinus x-rays or CTs, but by signs and symptoms—as we typically do in daily practice?

A meta-analysis3 of 13 randomized controlled trials (RCTs) found that sinusitis improved without antibiotics, but it included trials in which patients were recruited based on results of imaging studies and cultures, which are not normally used in primary care clinical practice. That study compared antibiotic treatment to placebo for acute uncomplicated sinusitis; 35% of placebo-treated patients were clinically cured by 7 to 12 days and 73% were improved after 7 days. Antibiotic therapy increased cure rates by 15% and improvement rates by 14%, yielding a number needed to treat of 7 to achieve 1 additional positive outcome at 7 days.

 

 

 

STUDY SUMMARY: Meta-analysis of primary care trials

Young and colleagues1 aggregated and analyzed individual patient-level data from all known placebo-controlled, randomized, antibiotic treatment trials of adults with clinical symptoms of acute sinusitis that were conducted in primary care settings. They excluded trials that used imaging or bacterial culture as part of patient recruitment.

Studies were included that allowed the use of concomitant medication such as nonsteroidal anti-inflammatory drugs, decongestants, or nasal steroids, as long as patients in both groups had access to the same medications. All trials excluded patients with severe symptoms such as high fever, periorbital swelling or erythema, or intense facial pain, important exclusions that we will discuss below.

They identified 10 such studies and completed an intent-to-treat analysis of the 9 double-blind trials for which patient level data were available. Using individual data from 2547 patients, the odds ratio for an overall antibiotic treatment effect was 1.37 (95% confidence interval, 1.13-1.66), with a number needed to treat (NNT) of 15.

This finding means that 15 patients needed to be given an antibiotic for 1 additional patient to be cured at 8 to 15 days after treatment commenced. Using statistical modeling, they determined that 64% of patients treated with placebo were cured at 14 days compared with 70% given an antibiotic. One patient out of 1381 treated with placebo experienced a serious complication, a brain abscess.

Do antibiotics benefit any subgroups?

The investigators also analyzed the prognostic value of specific signs and symptoms to answer the question: Is there any subgroup of patients who might benefit more from antibiotic treatment?

Duration. Patients with a longer duration of symptoms, more severe symptoms, or increased age took longer to cure, but were no more likely to benefit from antibiotic treatment than other patients.

Symptoms, such as a previous common cold, pain on bending, unilateral facial pain, tooth pain, and purulent nasal discharge did not have any prognostic value.

Only one sign—purulent discharge noted in the pharynx on examination—was associated with a higher likelihood of benefit from treatment with antibiotics, but the NNT was still 8 in this group. Patients with symptoms for 7 days or longer were no more likely to respond to antibiotics than those with symptoms for fewer than 7 days.1

WHAT’S NEW: Realistic evidence from realistic settings

We believe this meta-analysis provides a high level of evidence against routine treatment of sinusitis with antibiotics in primary care practice. Treating 15 patients with an antibiotic to possibly benefit 1 patient 2 weeks after treatment commences does not seem like a good idea when one considers the cost and complications of antibiotic use. Diarrhea and other adverse outcomes are 80% more common among patients with sinusitis who are treated with an antibiotic compared with placebo.3 As noted above, prior meta-analyses of antibiotic treatment for acute sinusitis have been more encouraging than this meta-analysis, with a number needed to treat of 7, but those meta-analyses are clearly overly optimistic for the results one will achieve in primary care practice using clinical signs and symptoms to diagnose acute sinusitis.3,4 Unlike the Young study, they included trials in specialty clinics with CT scans and sinus puncture and culture used for the diagnostic standard.

Symptoms >1 week are not a reason to prescribe

One very important new finding in this meta-analysis that should change practice is that the duration of illness did not predict a positive response to antibiotics.

Current national recommendations are to use an antibiotic for patients with a duration of illness longer than 1 week, as these patients are presumably more likely to have a bacterial infection.5-7 However, that recommendation had been based on expert opinion, not on data from clinical trials. A longer duration of symptoms should not be a reason to prescribe an antibiotic for sinusitis symptoms.

How can you help your patient?

What to do, then, for patients with acute sinusitis? Treat the symptoms, which means recommending pain medication for facial pain or headache and saline nasal spray for the nasal discharge, not antibiotics or nasal corticosteroids. Side effects will be fewer and costs will be lower.

  • Saline irrigation. A 2007 Cochrane review of 8 chronic and recurrent sinusitis trials showed that nasal saline irrigation is effective for reducing symptoms of chronic and recurrent sinusitis.8 Although we do not have high-quality RCT data on saline nasal irrigation for treatment of acute sinusitis, nasal saline irrigation is harmless and inexpensive.
  • What about nasal steroids? The evidence is equivocal, and the most recent high-quality RCT of nasal steroids showed no effect.9
 

 

 

CAVEATS: Refer seriously ill patients and complicated cases

A very important caveat to our recommendation is that seriously ill patients must be managed differently. Very infrequently a patient develops a serious complication of acute sinusitis such as brain abscess, periorbital cellulitis, or meningitis. Therefore, seriously ill patients with signs and symptoms of acute bacterial sinusitis, such as high fever, periorbital erythema or edema, severe headache, or intense facial pain must be carefully evaluated and treated with great caution and close follow-up. These patients should be referred immediately for consultation with an otolaryngologist.

Of course, mildly ill patients today may become quite ill tomorrow, so always provide advice to patients to return if they are getting worse, a good clinical practice for any condition that is usually benign but occasionally serious.

Patients who have prolonged or recurrent sinusitis symptoms need further evaluation for other diagnoses such as allergies, cystic fibrosis, fungal sinus infection, and other illnesses associated with immune compromise. These complicated patients benefit from consultation with an otolaryngologist who has a specific interest in chronic and recurrent sinusitis, and perhaps consultation from an infectious disease specialist as well.

CHALLENGES TO IMPLEMENTATION: The patient who wants a pill

Some patients may be accustomed to receiving an antibiotic prescription for their “sinus infections” and may resist conservative management. It may be difficult to convince them that antibiotics won’t make a difference when they attribute past resolution of symptoms to antibiotics.

Take enough time to educate your patients on the natural course of illness, the positive benefits of nasal saline, and the reasons not to use unnecessary antibiotics (eg, they are not effective, have potential adverse effects, and can contribute to future antibiotic resistance); this effort will save you time in future visits.10 A “just in case you don’t get better” prescription to be filled only if the patient is not improving in the next few days is about 50% effective in reducing antibiotic usage for upper respiratory infections.11

Acknowledgement

We acknowledge Sofia Medvedev of the University HealthSystem Consortium (UHC) in Oak Brook, IL for analysis of the National Ambulatory Medical Care Survey data.

PURLs methodology

This study was selected and evaluated using FPIN’s Priority Updates from the Research Literature (PURL) Surveillance System methodology. The criteria and findings leading to the selection of this study as a PURL can be accessed at www.jfponline.com/purls.

Files
References

1. Young J, De Sutter A, Merenstein D, et al. Antibiotics for adults with clinically diagnosed acute rhinosinusitis: a meta-analysis of individual patient data. Lancet. 2008;371:908-914.

2. Sharp HJ, Denman D, Puumala S, Leopold DA. Treatment of acute and chronic rhinosinusitis in the United States, 1999-2002. Arch Otolaryngol Head Neck Surg. 2007;133:260-265.

3. Rosenfeld RM, Singer M, Jones S. Systematic review of antimicrobial therapy in patients with acute rhinosinusitis. Otolaryngol Head Neck Surg. 2007;137(3 suppl):S32-S45.

4. Williams JW, Jr, Aguilar C, Cornell J, et al. Antibiotics for acute maxillary sinusitis. Cochrane Database Syst Rev. 2003;(2):CD000243.-

5. Hickner JM, Bartlett JG, Besser RE, Gonzales R, Hoffman JR, Sande MA. For the American Academy of Family Physicians; American College of Physicians-American Society of Internal Medicine; Centers for Disease Control; Infectious Diseases Society of America. Principles of appropriate antibiotic use for acute rhinosinusitis in adults: background. Ann Intern Med. 2001;134:498-505.

6. Rosenfeld RM, Andes D, Bhattacharyya N, et al. Clinical practice guidelines: adult sinusitis. Otolaryngol Head Neck Surg. 2007;137(3 suppl):S1-S31.

7. Anon JB, Jacobs MR, Poole MD, et al. For the Sinus and Allergy Health Partnership. Antimicrobial treatment guidelines for acute bacterial rhinosinusitis. Otolaryngol Head Neck Surg. 2004;130(1 suppl):S1-S45.

8. Harvey R, Hannan SA, Badia L, Scadding G. Nasal saline irrigations for the symptoms of chronic rhinosinusitis. Cochrane Database Syst Rev. 2007;(3):CD006394.-

9. Williamson IG, Rumsby K, Benge S, et al. Antibiotics and topical nasal steroid for treatment of acute maxillary sinusitis: a randomized controlled trial. JAMA. 2007;298:2487-2496.

10. Little P, Gould C, Williamson I, Warner G, Gantley M, Kinmonth AL. Reattendance and complications in a randomised trial of prescribing strategies for sore throat: the medicalising effect of prescribing antibiotics. BMJ. 1997;315:350-352.

11. Arroll B, Kenealy T, Kerse N. Do delayed prescriptions reduce antibiotic use in respiratory tract infections? A systematic review. Br J Gen Pract. 2003;53:871-877.

The PURLs Surveillance System is supported in part by Grant Number UL1RR024999 from the National Center For Research Resources, a Clinical Translational Science Award to the University of Chicago. The content is solely the responsibility of the authors and does not necessarily represent the official views of the National Center For Research Resources or the National Institutes of Health.

Article PDF
Author and Disclosure Information

Sarah-Anne Schumann, MD;
John Hickner, MD, MSc
Department of Family Medicine, The University of Chicago

PURLs EDITOR
Bernard Ewigman, MD, MSPH
Department of Family Medicine, The University of Chicago

Issue
The Journal of Family Practice - 57(7)
Publications
Topics
Page Number
464-468
Sections
Files
Files
Author and Disclosure Information

Sarah-Anne Schumann, MD;
John Hickner, MD, MSc
Department of Family Medicine, The University of Chicago

PURLs EDITOR
Bernard Ewigman, MD, MSPH
Department of Family Medicine, The University of Chicago

Author and Disclosure Information

Sarah-Anne Schumann, MD;
John Hickner, MD, MSc
Department of Family Medicine, The University of Chicago

PURLs EDITOR
Bernard Ewigman, MD, MSPH
Department of Family Medicine, The University of Chicago

Article PDF
Article PDF
 

ILLUSTRATIVE CASE

A 23-year-old woman presents to your office with a 1-week history of cough, purulent nasal discharge, and unilateral facial pain. You diagnose acute sinusitis.

Should you prescribe an antibiotic?

No. Yet it’s no wonder that most adults treated for acute sinusitis leave the doctor’s office with a prescription for antibiotics. Until the publication of the meta-analysis by Young and colleagues1 featured in this PURL, we have lacked A-level evidence from studies conducted in realistic settings—like your practice and ours.

Review of serial data from the National Ambulatory Medical Care Surveys (NAMCS) from 1999 through 2005 does show a slight downward trend in antibiotic prescribing for acute sinusitis: 1999-2002 data showed that 83% of cases of acute sinusitis were treated with an antibiotic.2 Data from the 2004 and 2005 NAMCS reveal that family physicians prescribed antibiotics for 80% of patients with acute sinusitis in 2004 and 76% of patients in 2005 (S. Medvedev, unpublished data, NAMCS database, March 2008).

Is this continued high rate of antibiotic prescribing justified?

Do antibiotics improve symptoms and shorten the duration of illness or not?

These questions are important, obviously, not only because of the high rate of prescribing but also because sinusitis is one of the most common diagnoses: approximately 20 million cases annually in the United States, or about 21% of all outpatient antibiotic prescriptions for adults.2

Which patients might benefit from antibiotics?

Common clinical signs and symptoms cannot identify patients with rhinosinusitis for whom treatment is clearly justified, given the cost, adverse events, and bacterial resistance associated with antibiotic use

  • Severity of symptoms is important only in that signs suggestive of a serious complication are the sole reason for immediate antibiotic treatment
  • Purulent discharge noted in the pharynx on exam was associated with a higher likelihood of benefit from antibiotics, but NNT was 8
  • Antibiotics are not justified even if a patient reports having symptoms for longer than 7-10 days

Source: Young et al.1

A diagnostic dilemma

Before we discuss the evidence that is summarized in the excellent meta-analysis by Young and colleagues,1 let’s acknowledge that acute sinusitis is undeniably a diagnostic dilemma. Distinguishing bacterial from viral infection is nearly impossible on clinical grounds because the symptoms are so similar. A litany of identical upper respiratory symptoms accompanies both viral and bacterial sinus infections. Purulent nasal secretions, maxillary facial pain (especially with unilateral predominance), maxillary tooth pain (which is uncommon with sinus infection), altered sense of smell, and worsening illness after improvement constitute the short list of signs and symptoms that has some predictive value, but even the presence of all of these is not a terrific predictor of bacterial sinus infection. Plain x-rays have low accuracy in distinguishing viral from bacterial infection. Computed tomography (CT) sinus scans are better but far from perfect, are not readily available in practice, and are expensive.

Sinusitis in the real world

How effective are antibiotics for patients diagnosed not by sinus x-rays or CTs, but by signs and symptoms—as we typically do in daily practice?

A meta-analysis3 of 13 randomized controlled trials (RCTs) found that sinusitis improved without antibiotics, but it included trials in which patients were recruited based on results of imaging studies and cultures, which are not normally used in primary care clinical practice. That study compared antibiotic treatment to placebo for acute uncomplicated sinusitis; 35% of placebo-treated patients were clinically cured by 7 to 12 days and 73% were improved after 7 days. Antibiotic therapy increased cure rates by 15% and improvement rates by 14%, yielding a number needed to treat of 7 to achieve 1 additional positive outcome at 7 days.

 

 

 

STUDY SUMMARY: Meta-analysis of primary care trials

Young and colleagues1 aggregated and analyzed individual patient-level data from all known placebo-controlled, randomized, antibiotic treatment trials of adults with clinical symptoms of acute sinusitis that were conducted in primary care settings. They excluded trials that used imaging or bacterial culture as part of patient recruitment.

Studies were included that allowed the use of concomitant medication such as nonsteroidal anti-inflammatory drugs, decongestants, or nasal steroids, as long as patients in both groups had access to the same medications. All trials excluded patients with severe symptoms such as high fever, periorbital swelling or erythema, or intense facial pain, important exclusions that we will discuss below.

They identified 10 such studies and completed an intent-to-treat analysis of the 9 double-blind trials for which patient level data were available. Using individual data from 2547 patients, the odds ratio for an overall antibiotic treatment effect was 1.37 (95% confidence interval, 1.13-1.66), with a number needed to treat (NNT) of 15.

This finding means that 15 patients needed to be given an antibiotic for 1 additional patient to be cured at 8 to 15 days after treatment commenced. Using statistical modeling, they determined that 64% of patients treated with placebo were cured at 14 days compared with 70% given an antibiotic. One patient out of 1381 treated with placebo experienced a serious complication, a brain abscess.

Do antibiotics benefit any subgroups?

The investigators also analyzed the prognostic value of specific signs and symptoms to answer the question: Is there any subgroup of patients who might benefit more from antibiotic treatment?

Duration. Patients with a longer duration of symptoms, more severe symptoms, or increased age took longer to cure, but were no more likely to benefit from antibiotic treatment than other patients.

Symptoms, such as a previous common cold, pain on bending, unilateral facial pain, tooth pain, and purulent nasal discharge did not have any prognostic value.

Only one sign—purulent discharge noted in the pharynx on examination—was associated with a higher likelihood of benefit from treatment with antibiotics, but the NNT was still 8 in this group. Patients with symptoms for 7 days or longer were no more likely to respond to antibiotics than those with symptoms for fewer than 7 days.1

WHAT’S NEW: Realistic evidence from realistic settings

We believe this meta-analysis provides a high level of evidence against routine treatment of sinusitis with antibiotics in primary care practice. Treating 15 patients with an antibiotic to possibly benefit 1 patient 2 weeks after treatment commences does not seem like a good idea when one considers the cost and complications of antibiotic use. Diarrhea and other adverse outcomes are 80% more common among patients with sinusitis who are treated with an antibiotic compared with placebo.3 As noted above, prior meta-analyses of antibiotic treatment for acute sinusitis have been more encouraging than this meta-analysis, with a number needed to treat of 7, but those meta-analyses are clearly overly optimistic for the results one will achieve in primary care practice using clinical signs and symptoms to diagnose acute sinusitis.3,4 Unlike the Young study, they included trials in specialty clinics with CT scans and sinus puncture and culture used for the diagnostic standard.

Symptoms >1 week are not a reason to prescribe

One very important new finding in this meta-analysis that should change practice is that the duration of illness did not predict a positive response to antibiotics.

Current national recommendations are to use an antibiotic for patients with a duration of illness longer than 1 week, as these patients are presumably more likely to have a bacterial infection.5-7 However, that recommendation had been based on expert opinion, not on data from clinical trials. A longer duration of symptoms should not be a reason to prescribe an antibiotic for sinusitis symptoms.

How can you help your patient?

What to do, then, for patients with acute sinusitis? Treat the symptoms, which means recommending pain medication for facial pain or headache and saline nasal spray for the nasal discharge, not antibiotics or nasal corticosteroids. Side effects will be fewer and costs will be lower.

  • Saline irrigation. A 2007 Cochrane review of 8 chronic and recurrent sinusitis trials showed that nasal saline irrigation is effective for reducing symptoms of chronic and recurrent sinusitis.8 Although we do not have high-quality RCT data on saline nasal irrigation for treatment of acute sinusitis, nasal saline irrigation is harmless and inexpensive.
  • What about nasal steroids? The evidence is equivocal, and the most recent high-quality RCT of nasal steroids showed no effect.9
 

 

 

CAVEATS: Refer seriously ill patients and complicated cases

A very important caveat to our recommendation is that seriously ill patients must be managed differently. Very infrequently a patient develops a serious complication of acute sinusitis such as brain abscess, periorbital cellulitis, or meningitis. Therefore, seriously ill patients with signs and symptoms of acute bacterial sinusitis, such as high fever, periorbital erythema or edema, severe headache, or intense facial pain must be carefully evaluated and treated with great caution and close follow-up. These patients should be referred immediately for consultation with an otolaryngologist.

Of course, mildly ill patients today may become quite ill tomorrow, so always provide advice to patients to return if they are getting worse, a good clinical practice for any condition that is usually benign but occasionally serious.

Patients who have prolonged or recurrent sinusitis symptoms need further evaluation for other diagnoses such as allergies, cystic fibrosis, fungal sinus infection, and other illnesses associated with immune compromise. These complicated patients benefit from consultation with an otolaryngologist who has a specific interest in chronic and recurrent sinusitis, and perhaps consultation from an infectious disease specialist as well.

CHALLENGES TO IMPLEMENTATION: The patient who wants a pill

Some patients may be accustomed to receiving an antibiotic prescription for their “sinus infections” and may resist conservative management. It may be difficult to convince them that antibiotics won’t make a difference when they attribute past resolution of symptoms to antibiotics.

Take enough time to educate your patients on the natural course of illness, the positive benefits of nasal saline, and the reasons not to use unnecessary antibiotics (eg, they are not effective, have potential adverse effects, and can contribute to future antibiotic resistance); this effort will save you time in future visits.10 A “just in case you don’t get better” prescription to be filled only if the patient is not improving in the next few days is about 50% effective in reducing antibiotic usage for upper respiratory infections.11

Acknowledgement

We acknowledge Sofia Medvedev of the University HealthSystem Consortium (UHC) in Oak Brook, IL for analysis of the National Ambulatory Medical Care Survey data.

PURLs methodology

This study was selected and evaluated using FPIN’s Priority Updates from the Research Literature (PURL) Surveillance System methodology. The criteria and findings leading to the selection of this study as a PURL can be accessed at www.jfponline.com/purls.

 

ILLUSTRATIVE CASE

A 23-year-old woman presents to your office with a 1-week history of cough, purulent nasal discharge, and unilateral facial pain. You diagnose acute sinusitis.

Should you prescribe an antibiotic?

No. Yet it’s no wonder that most adults treated for acute sinusitis leave the doctor’s office with a prescription for antibiotics. Until the publication of the meta-analysis by Young and colleagues1 featured in this PURL, we have lacked A-level evidence from studies conducted in realistic settings—like your practice and ours.

Review of serial data from the National Ambulatory Medical Care Surveys (NAMCS) from 1999 through 2005 does show a slight downward trend in antibiotic prescribing for acute sinusitis: 1999-2002 data showed that 83% of cases of acute sinusitis were treated with an antibiotic.2 Data from the 2004 and 2005 NAMCS reveal that family physicians prescribed antibiotics for 80% of patients with acute sinusitis in 2004 and 76% of patients in 2005 (S. Medvedev, unpublished data, NAMCS database, March 2008).

Is this continued high rate of antibiotic prescribing justified?

Do antibiotics improve symptoms and shorten the duration of illness or not?

These questions are important, obviously, not only because of the high rate of prescribing but also because sinusitis is one of the most common diagnoses: approximately 20 million cases annually in the United States, or about 21% of all outpatient antibiotic prescriptions for adults.2

Which patients might benefit from antibiotics?

Common clinical signs and symptoms cannot identify patients with rhinosinusitis for whom treatment is clearly justified, given the cost, adverse events, and bacterial resistance associated with antibiotic use

  • Severity of symptoms is important only in that signs suggestive of a serious complication are the sole reason for immediate antibiotic treatment
  • Purulent discharge noted in the pharynx on exam was associated with a higher likelihood of benefit from antibiotics, but NNT was 8
  • Antibiotics are not justified even if a patient reports having symptoms for longer than 7-10 days

Source: Young et al.1

A diagnostic dilemma

Before we discuss the evidence that is summarized in the excellent meta-analysis by Young and colleagues,1 let’s acknowledge that acute sinusitis is undeniably a diagnostic dilemma. Distinguishing bacterial from viral infection is nearly impossible on clinical grounds because the symptoms are so similar. A litany of identical upper respiratory symptoms accompanies both viral and bacterial sinus infections. Purulent nasal secretions, maxillary facial pain (especially with unilateral predominance), maxillary tooth pain (which is uncommon with sinus infection), altered sense of smell, and worsening illness after improvement constitute the short list of signs and symptoms that has some predictive value, but even the presence of all of these is not a terrific predictor of bacterial sinus infection. Plain x-rays have low accuracy in distinguishing viral from bacterial infection. Computed tomography (CT) sinus scans are better but far from perfect, are not readily available in practice, and are expensive.

Sinusitis in the real world

How effective are antibiotics for patients diagnosed not by sinus x-rays or CTs, but by signs and symptoms—as we typically do in daily practice?

A meta-analysis3 of 13 randomized controlled trials (RCTs) found that sinusitis improved without antibiotics, but it included trials in which patients were recruited based on results of imaging studies and cultures, which are not normally used in primary care clinical practice. That study compared antibiotic treatment to placebo for acute uncomplicated sinusitis; 35% of placebo-treated patients were clinically cured by 7 to 12 days and 73% were improved after 7 days. Antibiotic therapy increased cure rates by 15% and improvement rates by 14%, yielding a number needed to treat of 7 to achieve 1 additional positive outcome at 7 days.

 

 

 

STUDY SUMMARY: Meta-analysis of primary care trials

Young and colleagues1 aggregated and analyzed individual patient-level data from all known placebo-controlled, randomized, antibiotic treatment trials of adults with clinical symptoms of acute sinusitis that were conducted in primary care settings. They excluded trials that used imaging or bacterial culture as part of patient recruitment.

Studies were included that allowed the use of concomitant medication such as nonsteroidal anti-inflammatory drugs, decongestants, or nasal steroids, as long as patients in both groups had access to the same medications. All trials excluded patients with severe symptoms such as high fever, periorbital swelling or erythema, or intense facial pain, important exclusions that we will discuss below.

They identified 10 such studies and completed an intent-to-treat analysis of the 9 double-blind trials for which patient level data were available. Using individual data from 2547 patients, the odds ratio for an overall antibiotic treatment effect was 1.37 (95% confidence interval, 1.13-1.66), with a number needed to treat (NNT) of 15.

This finding means that 15 patients needed to be given an antibiotic for 1 additional patient to be cured at 8 to 15 days after treatment commenced. Using statistical modeling, they determined that 64% of patients treated with placebo were cured at 14 days compared with 70% given an antibiotic. One patient out of 1381 treated with placebo experienced a serious complication, a brain abscess.

Do antibiotics benefit any subgroups?

The investigators also analyzed the prognostic value of specific signs and symptoms to answer the question: Is there any subgroup of patients who might benefit more from antibiotic treatment?

Duration. Patients with a longer duration of symptoms, more severe symptoms, or increased age took longer to cure, but were no more likely to benefit from antibiotic treatment than other patients.

Symptoms, such as a previous common cold, pain on bending, unilateral facial pain, tooth pain, and purulent nasal discharge did not have any prognostic value.

Only one sign—purulent discharge noted in the pharynx on examination—was associated with a higher likelihood of benefit from treatment with antibiotics, but the NNT was still 8 in this group. Patients with symptoms for 7 days or longer were no more likely to respond to antibiotics than those with symptoms for fewer than 7 days.1

WHAT’S NEW: Realistic evidence from realistic settings

We believe this meta-analysis provides a high level of evidence against routine treatment of sinusitis with antibiotics in primary care practice. Treating 15 patients with an antibiotic to possibly benefit 1 patient 2 weeks after treatment commences does not seem like a good idea when one considers the cost and complications of antibiotic use. Diarrhea and other adverse outcomes are 80% more common among patients with sinusitis who are treated with an antibiotic compared with placebo.3 As noted above, prior meta-analyses of antibiotic treatment for acute sinusitis have been more encouraging than this meta-analysis, with a number needed to treat of 7, but those meta-analyses are clearly overly optimistic for the results one will achieve in primary care practice using clinical signs and symptoms to diagnose acute sinusitis.3,4 Unlike the Young study, they included trials in specialty clinics with CT scans and sinus puncture and culture used for the diagnostic standard.

Symptoms >1 week are not a reason to prescribe

One very important new finding in this meta-analysis that should change practice is that the duration of illness did not predict a positive response to antibiotics.

Current national recommendations are to use an antibiotic for patients with a duration of illness longer than 1 week, as these patients are presumably more likely to have a bacterial infection.5-7 However, that recommendation had been based on expert opinion, not on data from clinical trials. A longer duration of symptoms should not be a reason to prescribe an antibiotic for sinusitis symptoms.

How can you help your patient?

What to do, then, for patients with acute sinusitis? Treat the symptoms, which means recommending pain medication for facial pain or headache and saline nasal spray for the nasal discharge, not antibiotics or nasal corticosteroids. Side effects will be fewer and costs will be lower.

  • Saline irrigation. A 2007 Cochrane review of 8 chronic and recurrent sinusitis trials showed that nasal saline irrigation is effective for reducing symptoms of chronic and recurrent sinusitis.8 Although we do not have high-quality RCT data on saline nasal irrigation for treatment of acute sinusitis, nasal saline irrigation is harmless and inexpensive.
  • What about nasal steroids? The evidence is equivocal, and the most recent high-quality RCT of nasal steroids showed no effect.9
 

 

 

CAVEATS: Refer seriously ill patients and complicated cases

A very important caveat to our recommendation is that seriously ill patients must be managed differently. Very infrequently a patient develops a serious complication of acute sinusitis such as brain abscess, periorbital cellulitis, or meningitis. Therefore, seriously ill patients with signs and symptoms of acute bacterial sinusitis, such as high fever, periorbital erythema or edema, severe headache, or intense facial pain must be carefully evaluated and treated with great caution and close follow-up. These patients should be referred immediately for consultation with an otolaryngologist.

Of course, mildly ill patients today may become quite ill tomorrow, so always provide advice to patients to return if they are getting worse, a good clinical practice for any condition that is usually benign but occasionally serious.

Patients who have prolonged or recurrent sinusitis symptoms need further evaluation for other diagnoses such as allergies, cystic fibrosis, fungal sinus infection, and other illnesses associated with immune compromise. These complicated patients benefit from consultation with an otolaryngologist who has a specific interest in chronic and recurrent sinusitis, and perhaps consultation from an infectious disease specialist as well.

CHALLENGES TO IMPLEMENTATION: The patient who wants a pill

Some patients may be accustomed to receiving an antibiotic prescription for their “sinus infections” and may resist conservative management. It may be difficult to convince them that antibiotics won’t make a difference when they attribute past resolution of symptoms to antibiotics.

Take enough time to educate your patients on the natural course of illness, the positive benefits of nasal saline, and the reasons not to use unnecessary antibiotics (eg, they are not effective, have potential adverse effects, and can contribute to future antibiotic resistance); this effort will save you time in future visits.10 A “just in case you don’t get better” prescription to be filled only if the patient is not improving in the next few days is about 50% effective in reducing antibiotic usage for upper respiratory infections.11

Acknowledgement

We acknowledge Sofia Medvedev of the University HealthSystem Consortium (UHC) in Oak Brook, IL for analysis of the National Ambulatory Medical Care Survey data.

PURLs methodology

This study was selected and evaluated using FPIN’s Priority Updates from the Research Literature (PURL) Surveillance System methodology. The criteria and findings leading to the selection of this study as a PURL can be accessed at www.jfponline.com/purls.

References

1. Young J, De Sutter A, Merenstein D, et al. Antibiotics for adults with clinically diagnosed acute rhinosinusitis: a meta-analysis of individual patient data. Lancet. 2008;371:908-914.

2. Sharp HJ, Denman D, Puumala S, Leopold DA. Treatment of acute and chronic rhinosinusitis in the United States, 1999-2002. Arch Otolaryngol Head Neck Surg. 2007;133:260-265.

3. Rosenfeld RM, Singer M, Jones S. Systematic review of antimicrobial therapy in patients with acute rhinosinusitis. Otolaryngol Head Neck Surg. 2007;137(3 suppl):S32-S45.

4. Williams JW, Jr, Aguilar C, Cornell J, et al. Antibiotics for acute maxillary sinusitis. Cochrane Database Syst Rev. 2003;(2):CD000243.-

5. Hickner JM, Bartlett JG, Besser RE, Gonzales R, Hoffman JR, Sande MA. For the American Academy of Family Physicians; American College of Physicians-American Society of Internal Medicine; Centers for Disease Control; Infectious Diseases Society of America. Principles of appropriate antibiotic use for acute rhinosinusitis in adults: background. Ann Intern Med. 2001;134:498-505.

6. Rosenfeld RM, Andes D, Bhattacharyya N, et al. Clinical practice guidelines: adult sinusitis. Otolaryngol Head Neck Surg. 2007;137(3 suppl):S1-S31.

7. Anon JB, Jacobs MR, Poole MD, et al. For the Sinus and Allergy Health Partnership. Antimicrobial treatment guidelines for acute bacterial rhinosinusitis. Otolaryngol Head Neck Surg. 2004;130(1 suppl):S1-S45.

8. Harvey R, Hannan SA, Badia L, Scadding G. Nasal saline irrigations for the symptoms of chronic rhinosinusitis. Cochrane Database Syst Rev. 2007;(3):CD006394.-

9. Williamson IG, Rumsby K, Benge S, et al. Antibiotics and topical nasal steroid for treatment of acute maxillary sinusitis: a randomized controlled trial. JAMA. 2007;298:2487-2496.

10. Little P, Gould C, Williamson I, Warner G, Gantley M, Kinmonth AL. Reattendance and complications in a randomised trial of prescribing strategies for sore throat: the medicalising effect of prescribing antibiotics. BMJ. 1997;315:350-352.

11. Arroll B, Kenealy T, Kerse N. Do delayed prescriptions reduce antibiotic use in respiratory tract infections? A systematic review. Br J Gen Pract. 2003;53:871-877.

The PURLs Surveillance System is supported in part by Grant Number UL1RR024999 from the National Center For Research Resources, a Clinical Translational Science Award to the University of Chicago. The content is solely the responsibility of the authors and does not necessarily represent the official views of the National Center For Research Resources or the National Institutes of Health.

References

1. Young J, De Sutter A, Merenstein D, et al. Antibiotics for adults with clinically diagnosed acute rhinosinusitis: a meta-analysis of individual patient data. Lancet. 2008;371:908-914.

2. Sharp HJ, Denman D, Puumala S, Leopold DA. Treatment of acute and chronic rhinosinusitis in the United States, 1999-2002. Arch Otolaryngol Head Neck Surg. 2007;133:260-265.

3. Rosenfeld RM, Singer M, Jones S. Systematic review of antimicrobial therapy in patients with acute rhinosinusitis. Otolaryngol Head Neck Surg. 2007;137(3 suppl):S32-S45.

4. Williams JW, Jr, Aguilar C, Cornell J, et al. Antibiotics for acute maxillary sinusitis. Cochrane Database Syst Rev. 2003;(2):CD000243.-

5. Hickner JM, Bartlett JG, Besser RE, Gonzales R, Hoffman JR, Sande MA. For the American Academy of Family Physicians; American College of Physicians-American Society of Internal Medicine; Centers for Disease Control; Infectious Diseases Society of America. Principles of appropriate antibiotic use for acute rhinosinusitis in adults: background. Ann Intern Med. 2001;134:498-505.

6. Rosenfeld RM, Andes D, Bhattacharyya N, et al. Clinical practice guidelines: adult sinusitis. Otolaryngol Head Neck Surg. 2007;137(3 suppl):S1-S31.

7. Anon JB, Jacobs MR, Poole MD, et al. For the Sinus and Allergy Health Partnership. Antimicrobial treatment guidelines for acute bacterial rhinosinusitis. Otolaryngol Head Neck Surg. 2004;130(1 suppl):S1-S45.

8. Harvey R, Hannan SA, Badia L, Scadding G. Nasal saline irrigations for the symptoms of chronic rhinosinusitis. Cochrane Database Syst Rev. 2007;(3):CD006394.-

9. Williamson IG, Rumsby K, Benge S, et al. Antibiotics and topical nasal steroid for treatment of acute maxillary sinusitis: a randomized controlled trial. JAMA. 2007;298:2487-2496.

10. Little P, Gould C, Williamson I, Warner G, Gantley M, Kinmonth AL. Reattendance and complications in a randomised trial of prescribing strategies for sore throat: the medicalising effect of prescribing antibiotics. BMJ. 1997;315:350-352.

11. Arroll B, Kenealy T, Kerse N. Do delayed prescriptions reduce antibiotic use in respiratory tract infections? A systematic review. Br J Gen Pract. 2003;53:871-877.

The PURLs Surveillance System is supported in part by Grant Number UL1RR024999 from the National Center For Research Resources, a Clinical Translational Science Award to the University of Chicago. The content is solely the responsibility of the authors and does not necessarily represent the official views of the National Center For Research Resources or the National Institutes of Health.

Issue
The Journal of Family Practice - 57(7)
Issue
The Journal of Family Practice - 57(7)
Page Number
464-468
Page Number
464-468
Publications
Publications
Topics
Article Type
Display Headline
Patients insist on antibiotics for sinusitis? Here is a good reason to say “no”
Display Headline
Patients insist on antibiotics for sinusitis? Here is a good reason to say “no”
Sections
PURLs Copyright

Copyright © 2008 The Family Physicians Inquiries Network.
All rights reserved.

Disallow All Ads
Alternative CME
Use ProPublica
Hide sidebar & use full width
render the right sidebar.
Conference Recap Checkbox
Not Conference Recap
Clinical Edge
Article PDF Media
Media Files

Have pedometer, will travel

Article Type
Changed
Fri, 06/19/2020 - 11:35
Display Headline
Have pedometer, will travel
Practice changer

Advise your patients to use a pedometer, set a step goal, and keep a step diary. This simple intervention takes only a few moments and is effective in increasing patients’ physical activity and decreasing both body-mass index (BMI) and systolic blood pressure.1

Strength of recommendation

A: Based on a meta-analysis of randomized controlled trials (RCTs) and observational studies

Bravata DM, Smith-Spangler C, Sundaram V et al. Using pedometers to increase physical activity and improve health: a systematic review. JAMA 2007; 298:2296–2304.

 

Illustrative case

Your first 4 patients this morning were a 50-year-old woman with metabolic syndrome, a 62-year-old obese man with high blood pressure, a 44-year-old woman with depression, and a 75-year-old man with a recent admission for myocardial infarction. In addition to managing their medications and reviewing lab results, you have already spent a lot of time discussing the benefits of exercise with each of these patients.

As you prepare to talk with your next patient—a 28-year-old woman with a BMI of 29 whose chief complaint is “wants to lose weight”—you wonder if there are any simple, brief, effective interventions to help your patients increase their physical activity.

BACKGROUND: A long way to go

Although there is no evidence that simply advising patients to walk has any effect, primary care physicians frequently recommend walking as a form of exercise—it is free, requires no special equipment, and is readily accessible to most motivated patients.

The Centers for Disease Control and Prevention recommends that adults engage in moderate physical activity for at least 30 minutes a day, at least 5 days per week.2 Yet 40% of adults do not engage in any leisure-time physical activity. This percentage is higher in women (43%), African-Americans (52%), and Hispanics (54%).3

The health benefits of exercise are clear. Regular physical activity has been shown to decrease overweight and obesity.4 It has also been shown to improve control of type 2 diabetes5 and hypertension.6 Frequent exercise is associated with a decreased mortality rate.7 Walking has been shown to decrease the risk of cardiovascular events in women, regardless of BMI.8

Walking has similarly been shown to decrease overall mortality among men.9 Cardiovascular fitness has also been shown to decrease mortality in adults over 60, even in the absence of weight loss.10

CLINICAL CONTEXT: USPSTF: Advice alone won’t kick-start exercise

We realize, of course, that most of our adult patients could benefit from regular exercise. Exercise is included in the treatment guidelines for overweight/obesity, hypertension, type 2 diabetes, metabolic syndrome, cardiovascular disease, chronic pain, peripheral vascular disease, and depression.11

Eureka! a simple, practical intervention


PURLs EDITOR
Bernard Ewigman. MD, MSPH

Department of Family Medicine
The University of Chicago
be.editor@gmail.com

At last, the humble pedometer gives us a brief intervention for physical exercise that works. yes, we need more research for lots of reasons (always), but this Purl gives us a practical tool that can be recommended in a few minutes, consistent with the realities of daily practice.

The outcomes from this intervention are not dramatic. No lives were saved, no catastrophic diseases averted. yet regular exercise is so fundamentally important to just feeling good and having energy for daily life, not to mention lowering blood pressure and weight.

My guess is that this could become a handy recommendation used daily in family medicine and other primary care practices.

I am interested to know whether you already recommend pedometers to your patients. If not, does this seem like a worthwhile change in your practice?

On a personal note, I made a New year’s resolution to increase my physical activity. as soon as I finish this commentary, I am ordering a pedometer.

However, few office-based interventions have been shown to lead to increased physical activity. Patients sometimes resist making lifestyle changes, and providers are uncertain how to effectively promote physical activity. Furthermore, counseling patients to exercise without a specific intervention has not been shown to lead to long-term increases in physical activity. The US Preventive Services Task Force (USPSTF) finds there is insufficient evidence to recommend behavioral counseling alone for exercise, citing the lack of evidence for long-term efficacy.12,13

STUDY SUMMARY: Pedometer users walked 2491 additional steps

This meta-analysis included 26 RCTs and observational studies of pedometer use in adult outpatients that reported a change in the number of steps walked per day. The 2767 participants in these studies were 85% women, with a mean age of 49. In the 7 studies that reported race, 93% of patients were white. At baseline, most participants were overweight, with normal blood pressure (mean 129/79 mm Hg) and relatively well-controlled lipid levels (mean total cholesterol 198 mg/dL, HDL 52 mg/dL, LDL 113 mg/dL). The mean baseline activity level was 7473 steps per day (range 2140–12,371). Duration of interventions ranged from 3 to 104 weeks, with a mean of 18 weeks. Sixteen of the studies used the Yamax pedometer, which has been validated for accuracy and reliability.

Participants in the RCTs who used pedometers increased their physical activity by 2491 steps per day more than controls. After excluding 1 study with a much higher increase in physical activity than the others, the increase was 2004 steps per day (95% confidence interval [CI], 878–3129; P<.001). In the observational studies, participants walked 2183 steps per day more than they had at baseline (95% CI, 1571–2796; P<.001). Overall, pedometer users increased their number of steps by 27% over baseline.

 

 

 

Step goal and step diary

Only studies that included a step goal and required participants to keep a step diary showed a significant increase in physical activity with pedometer use. There were no differences in outcomes based on duration of the intervention, inclusion of physical activity counseling, or the brand of pedometer used.

BMI and BP improved; lipids, glucose did not

Intervention participants had a statistically significant decrease in BMI of 0.38, which was associated with older age (P=.001), having a step goal (P=.04), and longer duration of the intervention (P=.07, trend). Intervention participants also had a significant decrease in systolic blood pressure of 3.8 mm Hg and diastolic blood pressure of 0.3 mm Hg (TABLE 1), which was associated with greater systolic blood pressure at baseline (P=.009).

There were no significant differences in serum lipids or fasting serum glucose in the studies that reported these variables.1

TABLE 1
Pre- and post-intervention body mass index and blood pressure

body-mass index18 (562)30 (3.4)–0.38 (–0.05 to –0.72).03
systolic blood pressure12 (468)129 (7.5)–3.8 (–1.7 to –5.9)<.001
diastolic blood pressure12 (468)79 (4.5)–0.3 (0.02 to–0.46).001

WHAT’S NEW?: Weight loss without dieting

This study is the first large meta-analysis to show that pedometer use is an effective intervention for promoting physical activity. Another recent meta-analysis shows that pedometer use is also effective for short-term weight loss, even in the absence of dietary changes.14

Pedometers and goal-setting are simple, relatively inexpensive ways to help patients become physically active. According to systematic reviews,15,16 telephone-based programs, encouraging stair use, and creating exercise space are other effective interventions to promote physical activity. Some of these interventions are at least as effective as pedometers; however, only encouraging stair walking and pedometer use are practical office-based interventions.

CAVEATS: Price and quality

A 2004 Consumer Reports article ranked pedometers by accuracy, ease of use, and features.17 Accurate step counts allow patients and physicians to assess whether step goals are being met. Pedometers are more accurate when recording fast walking (2.5–3.0 mph), compared with slow walking. Pedometers may therefore be less accurate in the elderly, very obese, or those who walk slowly.18

TABLE 2
Consumer Reports top-rated pedometers17

Omron healthcare HJ-112$28.45*
Freestyle Tracer$15.99*
New lifestyles NL-2000$59.95
* Price from www.pedometersusa.com, accessed December 12, 2007.
† Price from newlifestyles.com, accessed December 12, 2007.
Omron, Freestyle, Yamax, Walk4Life, and New Lifestyles have been shown to be reliable brands.19,20

Negotiate the goal, patient keeps diary

Remember that patients must be counseled to set a step goal and keep a step diary. Most patients will have an initial step goal between 6000 and 10,000 steps per day. The step goal should be individualized to each patient’s current level of activity and gradually increased as activity level increases.

Schedule monthly or semi-monthly follow-up visits to evaluate progress towards activity or weight loss goals and to re-evaluate the step goal. Before beginning an exercise regimen, including walking, patients must be healthy enough for physical activity. In some cases, patients will need stress testing or other evaluation before using a pedometer to increase activity.

CHALLENGES TO IMPLEMENTATION: Time-wise

Counseling patients on the use of pedometers, and coaching them to set an appropriate step goal and keep a step diary, will take up time during the office visit, but it should be a brief intervention and therefore feasible.21


Omron Healthcare HJ-112


FreeStyle Tracer


New Lifestyles NL-2000

Organizing your office staff to assist you, and using a patient handout containing the basic information on pedometers, could reduce the demands on your time. Including information from the 2004 Consumer Reports article and Web sites with pedometers prices (such as www.pedometersusa.com and newlifestyles.com) should provide a good start for those patients who want more information.

PURLs methodology

This study was selected and evaluated using FPIN’s Priority Updates from the Research Literature (PURL) Surveillance System methodology. The criteria and findings leading to the selection of this study as a PURL can be accessed at www.jfponline.com/purls.

Files
References

1. Bravata DM, Smith-Spangler C, Sundaram V, et al. Using pedometers to increase physical activity and improve health: a systematic review. JAMA 2007;298:2296-2304.

2. Centers for Disease Control and Prevention/National Center for Health Statistics website. FASTATS: Exercise/physical activity. Available at www.cdc.gov/nchs/fastats/exercise.htm. Accessed January 22, 2008.

3. US Department of Health and Human Services. Office of Disease Prevention and Health Promotion. Healthy People 2010, Available at www.health.gov/healthypeople. Accessed January 22, 2008.

4. Miller WC, Koceja DM, Hamilton EJ. A meta-analysis of the past 25 years of weight loss research using diet, exercise or diet plus exercise intervention. Int J Obes Rel Metabolic Disord 1997;21:941-947.

5. Sigal RJ, Kenny GP, Boulé NG, et al. Effects of aerobic training, resistance training, or both on glycemic control in type 2 diabetes: a randomized trial. Ann Intern Med 2007;147:357-369.

6. Stewart KJ, Bacher AC, Turner KL, et al. Effect of exercise on blood pressure in older persons: a randomized controlled trial. Arch Intern Med 2005;165:756-762.

7. Paffenbarger RS, Hyde RT, Wing AL, et al. The association of changes in physical-activity level and other lifestyle characteristics with mortality among men. N Engl J Med 1993;328:538-545.

8. Manson JE, Greenland P, LaCroix AZ, et al. Walking compared with vigorous exercise for the prevention of cardiovascular events in women. N Engl J Med 2002;347:716-725.

9. Hakim AA, Petrovitch H, Burchfiel CM, et al. Effects of walking on mortality among nonsmoking retired men. N Engl J Med 1998;338:94-99.

10. Sui X, LaMonte MJ, Laditka JN, et al. Cardiorespiratory fitness and adiposity as mortality predictors in older adults. JAMA 2007;298:2507-2516.

11. National Guideline Clearinghouse. Available at www.guideline.gov. Accessed January 22, 2008.

12. US Preventive Services Task Force. Behavioral counseling in primary care to promote physical activity: recommendations and rationale. July 2002. Agency for Healthcare Research and Quality. Available at www.ahrq.gov/clinic/3rduspstf/physactivity/physactrr.htm. Accessed January 22, 2008.

13. Eden KB, Orleans CT, Mulrow CD, et al. Does counseling by clinicians improve physical activity? A summary of the evidence for the US Preventive Services Task Force. Ann Intern Med 2002;137:208-215.

14. Richardson CR, Newton TL, Abraham JJ, Sen A, Jimbo M, Swartz AM. Meta-analysis of pedometer-based walking interventions and weight loss. Ann Fam Med 2008;6:69-77.

15. Eakin EG, Lawler SP, Vandelanotte C, Owen N. Telephone interventions for physical activity and dietary behavior change: a systematic review. Am J Prev Med 2007;32:419-434.

16. Kahn EB, Ramsey LT, Brownson RC, et al. Effectiveness of interventions to increase physical activity: a systematic review. Am J Prev Med 2002;22(Suppl):73-107.

17. Pedometers: walking by the numbers Consumer Reports2004; Oct.

18. Melanson EL, Knoll JR, Bell ML, et al. Commercially available pedometers: considerations for accurate step counting. Prev Med 2004;39:361-368.

19. Bassett Dr, Jr, Ainsworth BE, Leggett SR, et al. Accuracy of five electronic pedometers for measuring distance walked. Med Sci Sports Exerc 1996;28:1071-1077.

20. Schneider PL, Crouter SE, Lukajic O, et al. Accuracy and reliability of 10 pedometers for measuring steps over a 400-m walk. Med Sci Sports Exerc 2003;35:1779-1784.

21. Ogilvie D, Foster CE, Rothnie H, et al. Interventions to promote walking: systematic review. BMJ 2007;334:1204.-

Author and Disclosure Information

Kathleen Rowland, MD
Sarah-Anne Schumann, MD
Department of Family Medicine, The University of Chicago

PURLs EDITOR
John Hickner, MD, MSc
Pritzker School of Medicine, The University of Chicago

Issue
The Journal of Family Practice - 57(2)
Publications
Topics
Page Number
90-93
Legacy Keywords
pedometer;walking;exercise;intervention;step;diary;physician;recommendation;BMI;activity;hypertension
Sections
Files
Files
Author and Disclosure Information

Kathleen Rowland, MD
Sarah-Anne Schumann, MD
Department of Family Medicine, The University of Chicago

PURLs EDITOR
John Hickner, MD, MSc
Pritzker School of Medicine, The University of Chicago

Author and Disclosure Information

Kathleen Rowland, MD
Sarah-Anne Schumann, MD
Department of Family Medicine, The University of Chicago

PURLs EDITOR
John Hickner, MD, MSc
Pritzker School of Medicine, The University of Chicago

Practice changer

Advise your patients to use a pedometer, set a step goal, and keep a step diary. This simple intervention takes only a few moments and is effective in increasing patients’ physical activity and decreasing both body-mass index (BMI) and systolic blood pressure.1

Strength of recommendation

A: Based on a meta-analysis of randomized controlled trials (RCTs) and observational studies

Bravata DM, Smith-Spangler C, Sundaram V et al. Using pedometers to increase physical activity and improve health: a systematic review. JAMA 2007; 298:2296–2304.

 

Illustrative case

Your first 4 patients this morning were a 50-year-old woman with metabolic syndrome, a 62-year-old obese man with high blood pressure, a 44-year-old woman with depression, and a 75-year-old man with a recent admission for myocardial infarction. In addition to managing their medications and reviewing lab results, you have already spent a lot of time discussing the benefits of exercise with each of these patients.

As you prepare to talk with your next patient—a 28-year-old woman with a BMI of 29 whose chief complaint is “wants to lose weight”—you wonder if there are any simple, brief, effective interventions to help your patients increase their physical activity.

BACKGROUND: A long way to go

Although there is no evidence that simply advising patients to walk has any effect, primary care physicians frequently recommend walking as a form of exercise—it is free, requires no special equipment, and is readily accessible to most motivated patients.

The Centers for Disease Control and Prevention recommends that adults engage in moderate physical activity for at least 30 minutes a day, at least 5 days per week.2 Yet 40% of adults do not engage in any leisure-time physical activity. This percentage is higher in women (43%), African-Americans (52%), and Hispanics (54%).3

The health benefits of exercise are clear. Regular physical activity has been shown to decrease overweight and obesity.4 It has also been shown to improve control of type 2 diabetes5 and hypertension.6 Frequent exercise is associated with a decreased mortality rate.7 Walking has been shown to decrease the risk of cardiovascular events in women, regardless of BMI.8

Walking has similarly been shown to decrease overall mortality among men.9 Cardiovascular fitness has also been shown to decrease mortality in adults over 60, even in the absence of weight loss.10

CLINICAL CONTEXT: USPSTF: Advice alone won’t kick-start exercise

We realize, of course, that most of our adult patients could benefit from regular exercise. Exercise is included in the treatment guidelines for overweight/obesity, hypertension, type 2 diabetes, metabolic syndrome, cardiovascular disease, chronic pain, peripheral vascular disease, and depression.11

Eureka! a simple, practical intervention


PURLs EDITOR
Bernard Ewigman. MD, MSPH

Department of Family Medicine
The University of Chicago
be.editor@gmail.com

At last, the humble pedometer gives us a brief intervention for physical exercise that works. yes, we need more research for lots of reasons (always), but this Purl gives us a practical tool that can be recommended in a few minutes, consistent with the realities of daily practice.

The outcomes from this intervention are not dramatic. No lives were saved, no catastrophic diseases averted. yet regular exercise is so fundamentally important to just feeling good and having energy for daily life, not to mention lowering blood pressure and weight.

My guess is that this could become a handy recommendation used daily in family medicine and other primary care practices.

I am interested to know whether you already recommend pedometers to your patients. If not, does this seem like a worthwhile change in your practice?

On a personal note, I made a New year’s resolution to increase my physical activity. as soon as I finish this commentary, I am ordering a pedometer.

However, few office-based interventions have been shown to lead to increased physical activity. Patients sometimes resist making lifestyle changes, and providers are uncertain how to effectively promote physical activity. Furthermore, counseling patients to exercise without a specific intervention has not been shown to lead to long-term increases in physical activity. The US Preventive Services Task Force (USPSTF) finds there is insufficient evidence to recommend behavioral counseling alone for exercise, citing the lack of evidence for long-term efficacy.12,13

STUDY SUMMARY: Pedometer users walked 2491 additional steps

This meta-analysis included 26 RCTs and observational studies of pedometer use in adult outpatients that reported a change in the number of steps walked per day. The 2767 participants in these studies were 85% women, with a mean age of 49. In the 7 studies that reported race, 93% of patients were white. At baseline, most participants were overweight, with normal blood pressure (mean 129/79 mm Hg) and relatively well-controlled lipid levels (mean total cholesterol 198 mg/dL, HDL 52 mg/dL, LDL 113 mg/dL). The mean baseline activity level was 7473 steps per day (range 2140–12,371). Duration of interventions ranged from 3 to 104 weeks, with a mean of 18 weeks. Sixteen of the studies used the Yamax pedometer, which has been validated for accuracy and reliability.

Participants in the RCTs who used pedometers increased their physical activity by 2491 steps per day more than controls. After excluding 1 study with a much higher increase in physical activity than the others, the increase was 2004 steps per day (95% confidence interval [CI], 878–3129; P<.001). In the observational studies, participants walked 2183 steps per day more than they had at baseline (95% CI, 1571–2796; P<.001). Overall, pedometer users increased their number of steps by 27% over baseline.

 

 

 

Step goal and step diary

Only studies that included a step goal and required participants to keep a step diary showed a significant increase in physical activity with pedometer use. There were no differences in outcomes based on duration of the intervention, inclusion of physical activity counseling, or the brand of pedometer used.

BMI and BP improved; lipids, glucose did not

Intervention participants had a statistically significant decrease in BMI of 0.38, which was associated with older age (P=.001), having a step goal (P=.04), and longer duration of the intervention (P=.07, trend). Intervention participants also had a significant decrease in systolic blood pressure of 3.8 mm Hg and diastolic blood pressure of 0.3 mm Hg (TABLE 1), which was associated with greater systolic blood pressure at baseline (P=.009).

There were no significant differences in serum lipids or fasting serum glucose in the studies that reported these variables.1

TABLE 1
Pre- and post-intervention body mass index and blood pressure

body-mass index18 (562)30 (3.4)–0.38 (–0.05 to –0.72).03
systolic blood pressure12 (468)129 (7.5)–3.8 (–1.7 to –5.9)<.001
diastolic blood pressure12 (468)79 (4.5)–0.3 (0.02 to–0.46).001

WHAT’S NEW?: Weight loss without dieting

This study is the first large meta-analysis to show that pedometer use is an effective intervention for promoting physical activity. Another recent meta-analysis shows that pedometer use is also effective for short-term weight loss, even in the absence of dietary changes.14

Pedometers and goal-setting are simple, relatively inexpensive ways to help patients become physically active. According to systematic reviews,15,16 telephone-based programs, encouraging stair use, and creating exercise space are other effective interventions to promote physical activity. Some of these interventions are at least as effective as pedometers; however, only encouraging stair walking and pedometer use are practical office-based interventions.

CAVEATS: Price and quality

A 2004 Consumer Reports article ranked pedometers by accuracy, ease of use, and features.17 Accurate step counts allow patients and physicians to assess whether step goals are being met. Pedometers are more accurate when recording fast walking (2.5–3.0 mph), compared with slow walking. Pedometers may therefore be less accurate in the elderly, very obese, or those who walk slowly.18

TABLE 2
Consumer Reports top-rated pedometers17

Omron healthcare HJ-112$28.45*
Freestyle Tracer$15.99*
New lifestyles NL-2000$59.95
* Price from www.pedometersusa.com, accessed December 12, 2007.
† Price from newlifestyles.com, accessed December 12, 2007.
Omron, Freestyle, Yamax, Walk4Life, and New Lifestyles have been shown to be reliable brands.19,20

Negotiate the goal, patient keeps diary

Remember that patients must be counseled to set a step goal and keep a step diary. Most patients will have an initial step goal between 6000 and 10,000 steps per day. The step goal should be individualized to each patient’s current level of activity and gradually increased as activity level increases.

Schedule monthly or semi-monthly follow-up visits to evaluate progress towards activity or weight loss goals and to re-evaluate the step goal. Before beginning an exercise regimen, including walking, patients must be healthy enough for physical activity. In some cases, patients will need stress testing or other evaluation before using a pedometer to increase activity.

CHALLENGES TO IMPLEMENTATION: Time-wise

Counseling patients on the use of pedometers, and coaching them to set an appropriate step goal and keep a step diary, will take up time during the office visit, but it should be a brief intervention and therefore feasible.21


Omron Healthcare HJ-112


FreeStyle Tracer


New Lifestyles NL-2000

Organizing your office staff to assist you, and using a patient handout containing the basic information on pedometers, could reduce the demands on your time. Including information from the 2004 Consumer Reports article and Web sites with pedometers prices (such as www.pedometersusa.com and newlifestyles.com) should provide a good start for those patients who want more information.

PURLs methodology

This study was selected and evaluated using FPIN’s Priority Updates from the Research Literature (PURL) Surveillance System methodology. The criteria and findings leading to the selection of this study as a PURL can be accessed at www.jfponline.com/purls.

Practice changer

Advise your patients to use a pedometer, set a step goal, and keep a step diary. This simple intervention takes only a few moments and is effective in increasing patients’ physical activity and decreasing both body-mass index (BMI) and systolic blood pressure.1

Strength of recommendation

A: Based on a meta-analysis of randomized controlled trials (RCTs) and observational studies

Bravata DM, Smith-Spangler C, Sundaram V et al. Using pedometers to increase physical activity and improve health: a systematic review. JAMA 2007; 298:2296–2304.

 

Illustrative case

Your first 4 patients this morning were a 50-year-old woman with metabolic syndrome, a 62-year-old obese man with high blood pressure, a 44-year-old woman with depression, and a 75-year-old man with a recent admission for myocardial infarction. In addition to managing their medications and reviewing lab results, you have already spent a lot of time discussing the benefits of exercise with each of these patients.

As you prepare to talk with your next patient—a 28-year-old woman with a BMI of 29 whose chief complaint is “wants to lose weight”—you wonder if there are any simple, brief, effective interventions to help your patients increase their physical activity.

BACKGROUND: A long way to go

Although there is no evidence that simply advising patients to walk has any effect, primary care physicians frequently recommend walking as a form of exercise—it is free, requires no special equipment, and is readily accessible to most motivated patients.

The Centers for Disease Control and Prevention recommends that adults engage in moderate physical activity for at least 30 minutes a day, at least 5 days per week.2 Yet 40% of adults do not engage in any leisure-time physical activity. This percentage is higher in women (43%), African-Americans (52%), and Hispanics (54%).3

The health benefits of exercise are clear. Regular physical activity has been shown to decrease overweight and obesity.4 It has also been shown to improve control of type 2 diabetes5 and hypertension.6 Frequent exercise is associated with a decreased mortality rate.7 Walking has been shown to decrease the risk of cardiovascular events in women, regardless of BMI.8

Walking has similarly been shown to decrease overall mortality among men.9 Cardiovascular fitness has also been shown to decrease mortality in adults over 60, even in the absence of weight loss.10

CLINICAL CONTEXT: USPSTF: Advice alone won’t kick-start exercise

We realize, of course, that most of our adult patients could benefit from regular exercise. Exercise is included in the treatment guidelines for overweight/obesity, hypertension, type 2 diabetes, metabolic syndrome, cardiovascular disease, chronic pain, peripheral vascular disease, and depression.11

Eureka! a simple, practical intervention


PURLs EDITOR
Bernard Ewigman. MD, MSPH

Department of Family Medicine
The University of Chicago
be.editor@gmail.com

At last, the humble pedometer gives us a brief intervention for physical exercise that works. yes, we need more research for lots of reasons (always), but this Purl gives us a practical tool that can be recommended in a few minutes, consistent with the realities of daily practice.

The outcomes from this intervention are not dramatic. No lives were saved, no catastrophic diseases averted. yet regular exercise is so fundamentally important to just feeling good and having energy for daily life, not to mention lowering blood pressure and weight.

My guess is that this could become a handy recommendation used daily in family medicine and other primary care practices.

I am interested to know whether you already recommend pedometers to your patients. If not, does this seem like a worthwhile change in your practice?

On a personal note, I made a New year’s resolution to increase my physical activity. as soon as I finish this commentary, I am ordering a pedometer.

However, few office-based interventions have been shown to lead to increased physical activity. Patients sometimes resist making lifestyle changes, and providers are uncertain how to effectively promote physical activity. Furthermore, counseling patients to exercise without a specific intervention has not been shown to lead to long-term increases in physical activity. The US Preventive Services Task Force (USPSTF) finds there is insufficient evidence to recommend behavioral counseling alone for exercise, citing the lack of evidence for long-term efficacy.12,13

STUDY SUMMARY: Pedometer users walked 2491 additional steps

This meta-analysis included 26 RCTs and observational studies of pedometer use in adult outpatients that reported a change in the number of steps walked per day. The 2767 participants in these studies were 85% women, with a mean age of 49. In the 7 studies that reported race, 93% of patients were white. At baseline, most participants were overweight, with normal blood pressure (mean 129/79 mm Hg) and relatively well-controlled lipid levels (mean total cholesterol 198 mg/dL, HDL 52 mg/dL, LDL 113 mg/dL). The mean baseline activity level was 7473 steps per day (range 2140–12,371). Duration of interventions ranged from 3 to 104 weeks, with a mean of 18 weeks. Sixteen of the studies used the Yamax pedometer, which has been validated for accuracy and reliability.

Participants in the RCTs who used pedometers increased their physical activity by 2491 steps per day more than controls. After excluding 1 study with a much higher increase in physical activity than the others, the increase was 2004 steps per day (95% confidence interval [CI], 878–3129; P<.001). In the observational studies, participants walked 2183 steps per day more than they had at baseline (95% CI, 1571–2796; P<.001). Overall, pedometer users increased their number of steps by 27% over baseline.

 

 

 

Step goal and step diary

Only studies that included a step goal and required participants to keep a step diary showed a significant increase in physical activity with pedometer use. There were no differences in outcomes based on duration of the intervention, inclusion of physical activity counseling, or the brand of pedometer used.

BMI and BP improved; lipids, glucose did not

Intervention participants had a statistically significant decrease in BMI of 0.38, which was associated with older age (P=.001), having a step goal (P=.04), and longer duration of the intervention (P=.07, trend). Intervention participants also had a significant decrease in systolic blood pressure of 3.8 mm Hg and diastolic blood pressure of 0.3 mm Hg (TABLE 1), which was associated with greater systolic blood pressure at baseline (P=.009).

There were no significant differences in serum lipids or fasting serum glucose in the studies that reported these variables.1

TABLE 1
Pre- and post-intervention body mass index and blood pressure

body-mass index18 (562)30 (3.4)–0.38 (–0.05 to –0.72).03
systolic blood pressure12 (468)129 (7.5)–3.8 (–1.7 to –5.9)<.001
diastolic blood pressure12 (468)79 (4.5)–0.3 (0.02 to–0.46).001

WHAT’S NEW?: Weight loss without dieting

This study is the first large meta-analysis to show that pedometer use is an effective intervention for promoting physical activity. Another recent meta-analysis shows that pedometer use is also effective for short-term weight loss, even in the absence of dietary changes.14

Pedometers and goal-setting are simple, relatively inexpensive ways to help patients become physically active. According to systematic reviews,15,16 telephone-based programs, encouraging stair use, and creating exercise space are other effective interventions to promote physical activity. Some of these interventions are at least as effective as pedometers; however, only encouraging stair walking and pedometer use are practical office-based interventions.

CAVEATS: Price and quality

A 2004 Consumer Reports article ranked pedometers by accuracy, ease of use, and features.17 Accurate step counts allow patients and physicians to assess whether step goals are being met. Pedometers are more accurate when recording fast walking (2.5–3.0 mph), compared with slow walking. Pedometers may therefore be less accurate in the elderly, very obese, or those who walk slowly.18

TABLE 2
Consumer Reports top-rated pedometers17

Omron healthcare HJ-112$28.45*
Freestyle Tracer$15.99*
New lifestyles NL-2000$59.95
* Price from www.pedometersusa.com, accessed December 12, 2007.
† Price from newlifestyles.com, accessed December 12, 2007.
Omron, Freestyle, Yamax, Walk4Life, and New Lifestyles have been shown to be reliable brands.19,20

Negotiate the goal, patient keeps diary

Remember that patients must be counseled to set a step goal and keep a step diary. Most patients will have an initial step goal between 6000 and 10,000 steps per day. The step goal should be individualized to each patient’s current level of activity and gradually increased as activity level increases.

Schedule monthly or semi-monthly follow-up visits to evaluate progress towards activity or weight loss goals and to re-evaluate the step goal. Before beginning an exercise regimen, including walking, patients must be healthy enough for physical activity. In some cases, patients will need stress testing or other evaluation before using a pedometer to increase activity.

CHALLENGES TO IMPLEMENTATION: Time-wise

Counseling patients on the use of pedometers, and coaching them to set an appropriate step goal and keep a step diary, will take up time during the office visit, but it should be a brief intervention and therefore feasible.21


Omron Healthcare HJ-112


FreeStyle Tracer


New Lifestyles NL-2000

Organizing your office staff to assist you, and using a patient handout containing the basic information on pedometers, could reduce the demands on your time. Including information from the 2004 Consumer Reports article and Web sites with pedometers prices (such as www.pedometersusa.com and newlifestyles.com) should provide a good start for those patients who want more information.

PURLs methodology

This study was selected and evaluated using FPIN’s Priority Updates from the Research Literature (PURL) Surveillance System methodology. The criteria and findings leading to the selection of this study as a PURL can be accessed at www.jfponline.com/purls.

References

1. Bravata DM, Smith-Spangler C, Sundaram V, et al. Using pedometers to increase physical activity and improve health: a systematic review. JAMA 2007;298:2296-2304.

2. Centers for Disease Control and Prevention/National Center for Health Statistics website. FASTATS: Exercise/physical activity. Available at www.cdc.gov/nchs/fastats/exercise.htm. Accessed January 22, 2008.

3. US Department of Health and Human Services. Office of Disease Prevention and Health Promotion. Healthy People 2010, Available at www.health.gov/healthypeople. Accessed January 22, 2008.

4. Miller WC, Koceja DM, Hamilton EJ. A meta-analysis of the past 25 years of weight loss research using diet, exercise or diet plus exercise intervention. Int J Obes Rel Metabolic Disord 1997;21:941-947.

5. Sigal RJ, Kenny GP, Boulé NG, et al. Effects of aerobic training, resistance training, or both on glycemic control in type 2 diabetes: a randomized trial. Ann Intern Med 2007;147:357-369.

6. Stewart KJ, Bacher AC, Turner KL, et al. Effect of exercise on blood pressure in older persons: a randomized controlled trial. Arch Intern Med 2005;165:756-762.

7. Paffenbarger RS, Hyde RT, Wing AL, et al. The association of changes in physical-activity level and other lifestyle characteristics with mortality among men. N Engl J Med 1993;328:538-545.

8. Manson JE, Greenland P, LaCroix AZ, et al. Walking compared with vigorous exercise for the prevention of cardiovascular events in women. N Engl J Med 2002;347:716-725.

9. Hakim AA, Petrovitch H, Burchfiel CM, et al. Effects of walking on mortality among nonsmoking retired men. N Engl J Med 1998;338:94-99.

10. Sui X, LaMonte MJ, Laditka JN, et al. Cardiorespiratory fitness and adiposity as mortality predictors in older adults. JAMA 2007;298:2507-2516.

11. National Guideline Clearinghouse. Available at www.guideline.gov. Accessed January 22, 2008.

12. US Preventive Services Task Force. Behavioral counseling in primary care to promote physical activity: recommendations and rationale. July 2002. Agency for Healthcare Research and Quality. Available at www.ahrq.gov/clinic/3rduspstf/physactivity/physactrr.htm. Accessed January 22, 2008.

13. Eden KB, Orleans CT, Mulrow CD, et al. Does counseling by clinicians improve physical activity? A summary of the evidence for the US Preventive Services Task Force. Ann Intern Med 2002;137:208-215.

14. Richardson CR, Newton TL, Abraham JJ, Sen A, Jimbo M, Swartz AM. Meta-analysis of pedometer-based walking interventions and weight loss. Ann Fam Med 2008;6:69-77.

15. Eakin EG, Lawler SP, Vandelanotte C, Owen N. Telephone interventions for physical activity and dietary behavior change: a systematic review. Am J Prev Med 2007;32:419-434.

16. Kahn EB, Ramsey LT, Brownson RC, et al. Effectiveness of interventions to increase physical activity: a systematic review. Am J Prev Med 2002;22(Suppl):73-107.

17. Pedometers: walking by the numbers Consumer Reports2004; Oct.

18. Melanson EL, Knoll JR, Bell ML, et al. Commercially available pedometers: considerations for accurate step counting. Prev Med 2004;39:361-368.

19. Bassett Dr, Jr, Ainsworth BE, Leggett SR, et al. Accuracy of five electronic pedometers for measuring distance walked. Med Sci Sports Exerc 1996;28:1071-1077.

20. Schneider PL, Crouter SE, Lukajic O, et al. Accuracy and reliability of 10 pedometers for measuring steps over a 400-m walk. Med Sci Sports Exerc 2003;35:1779-1784.

21. Ogilvie D, Foster CE, Rothnie H, et al. Interventions to promote walking: systematic review. BMJ 2007;334:1204.-

References

1. Bravata DM, Smith-Spangler C, Sundaram V, et al. Using pedometers to increase physical activity and improve health: a systematic review. JAMA 2007;298:2296-2304.

2. Centers for Disease Control and Prevention/National Center for Health Statistics website. FASTATS: Exercise/physical activity. Available at www.cdc.gov/nchs/fastats/exercise.htm. Accessed January 22, 2008.

3. US Department of Health and Human Services. Office of Disease Prevention and Health Promotion. Healthy People 2010, Available at www.health.gov/healthypeople. Accessed January 22, 2008.

4. Miller WC, Koceja DM, Hamilton EJ. A meta-analysis of the past 25 years of weight loss research using diet, exercise or diet plus exercise intervention. Int J Obes Rel Metabolic Disord 1997;21:941-947.

5. Sigal RJ, Kenny GP, Boulé NG, et al. Effects of aerobic training, resistance training, or both on glycemic control in type 2 diabetes: a randomized trial. Ann Intern Med 2007;147:357-369.

6. Stewart KJ, Bacher AC, Turner KL, et al. Effect of exercise on blood pressure in older persons: a randomized controlled trial. Arch Intern Med 2005;165:756-762.

7. Paffenbarger RS, Hyde RT, Wing AL, et al. The association of changes in physical-activity level and other lifestyle characteristics with mortality among men. N Engl J Med 1993;328:538-545.

8. Manson JE, Greenland P, LaCroix AZ, et al. Walking compared with vigorous exercise for the prevention of cardiovascular events in women. N Engl J Med 2002;347:716-725.

9. Hakim AA, Petrovitch H, Burchfiel CM, et al. Effects of walking on mortality among nonsmoking retired men. N Engl J Med 1998;338:94-99.

10. Sui X, LaMonte MJ, Laditka JN, et al. Cardiorespiratory fitness and adiposity as mortality predictors in older adults. JAMA 2007;298:2507-2516.

11. National Guideline Clearinghouse. Available at www.guideline.gov. Accessed January 22, 2008.

12. US Preventive Services Task Force. Behavioral counseling in primary care to promote physical activity: recommendations and rationale. July 2002. Agency for Healthcare Research and Quality. Available at www.ahrq.gov/clinic/3rduspstf/physactivity/physactrr.htm. Accessed January 22, 2008.

13. Eden KB, Orleans CT, Mulrow CD, et al. Does counseling by clinicians improve physical activity? A summary of the evidence for the US Preventive Services Task Force. Ann Intern Med 2002;137:208-215.

14. Richardson CR, Newton TL, Abraham JJ, Sen A, Jimbo M, Swartz AM. Meta-analysis of pedometer-based walking interventions and weight loss. Ann Fam Med 2008;6:69-77.

15. Eakin EG, Lawler SP, Vandelanotte C, Owen N. Telephone interventions for physical activity and dietary behavior change: a systematic review. Am J Prev Med 2007;32:419-434.

16. Kahn EB, Ramsey LT, Brownson RC, et al. Effectiveness of interventions to increase physical activity: a systematic review. Am J Prev Med 2002;22(Suppl):73-107.

17. Pedometers: walking by the numbers Consumer Reports2004; Oct.

18. Melanson EL, Knoll JR, Bell ML, et al. Commercially available pedometers: considerations for accurate step counting. Prev Med 2004;39:361-368.

19. Bassett Dr, Jr, Ainsworth BE, Leggett SR, et al. Accuracy of five electronic pedometers for measuring distance walked. Med Sci Sports Exerc 1996;28:1071-1077.

20. Schneider PL, Crouter SE, Lukajic O, et al. Accuracy and reliability of 10 pedometers for measuring steps over a 400-m walk. Med Sci Sports Exerc 2003;35:1779-1784.

21. Ogilvie D, Foster CE, Rothnie H, et al. Interventions to promote walking: systematic review. BMJ 2007;334:1204.-

Issue
The Journal of Family Practice - 57(2)
Issue
The Journal of Family Practice - 57(2)
Page Number
90-93
Page Number
90-93
Publications
Publications
Topics
Article Type
Display Headline
Have pedometer, will travel
Display Headline
Have pedometer, will travel
Legacy Keywords
pedometer;walking;exercise;intervention;step;diary;physician;recommendation;BMI;activity;hypertension
Legacy Keywords
pedometer;walking;exercise;intervention;step;diary;physician;recommendation;BMI;activity;hypertension
Sections
PURLs Copyright

Copyright © 2008 The Family Physicians Inquiries Network.
All rights reserved.

Disallow All Ads
Alternative CME
Use ProPublica
Hide sidebar & use full width
render the right sidebar.
Conference Recap Checkbox
Not Conference Recap
Clinical Edge
Media Files

For Bell’s palsy, start steroids early; no need for an antiviral

Article Type
Changed
Fri, 06/19/2020 - 12:42
Display Headline
For Bell’s palsy, start steroids early; no need for an antiviral
Practice changer

A 10-day course of corticosteroids (prednisolone 25 mg twice daily) started within 72 hours significantly improves the chances of complete recovery. There is no added benefit from acyclovir.1

Strength of recommendation

A: Based on a large, well-designed randomized controlled trial

Sullivan FM, Swan IR, Donnan PT, et al. Early treatment with prednisolone or acyclovir in Bell’s palsy. N Engl J Med 2007;357:1598–1607.

 

Illustrative case

A 45-year-old man presents to your outpatient clinic within 24 hours after onset of left-sided facial nerve weakness and inability to smile on one side of his face. He asks for a therapy to help improve his symptoms quickly, as his daughter is getting married in a few months, and he will be in the wedding pictures.

Is there a treatment that will hasten his complete recovery?

Background: Insufficient statistical power until now

Many of us treat patients with Bell’s palsy with both corticosteroids and antiviral medications, such as acyclovir or valacyclovir, largely on the basis of pathophysiologic reasoning, because we’ve had no clear guidance from outcome studies. Until now, outcome studies have had mixed findings, and have been inconclusive.2 Most outcome studies have lacked the statistical power to either detect or to rule out potential benefits convincingly. The study by Sullivan and colleagues is the first to have a sufficiently large study sample from which to draw more definitive conclusions based on patient-oriented outcomes.

Steroid plus antiviral makes sense, pathophysiologically

Corticosteroids are thought to decrease inflammation of the facial nerve during an episode of facial paralysis. Some have postulated that herpes simplex virus type I may be a cause of facial nerve paralysis, hence the treatment with antivirals.3

Most of our PURL surveillance system clinician reviewers said that they prescribe both corticosteroids and antivirals.

Guidelines: “probably, possibly”

For example, a report of the Quality Standards Subcommittee of the American Academy of Neurology concluded that benefit from both steroids and antivirals has not been well established in patients with Bell’s palsy. However, the report states that evidence suggests that steroids are safe and probably effective, while antivirals are also safe and possibly effective.4

In contrast, UpToDate suggests treating all patients seen within a week of symptom onset with corticosteroids (prednisone 60–80 mg daily) plus valacyclovir (1 g 3 times daily) for 1 week.5

CLINICAL CONTEXT: Quality of life, risk of permanent harm

Bell’s palsy, defined as an acute peripheral facial weakness of unknown cause, has an annual incidence of 20 to 32 per 100,000. Most patients recover completely, with or without treatment, but 20% to 30% can have permanent facial weakness or paralysis.

The time to resolution is a quality of life issue for those in whom disease does not resolve spontaneously.6

We think that this study provides convincing evidence that acyclovir is not indicated for Bell’s palsy and that corticosteroids are.

STUDY SUMMARY: 10-day treatment, starting promptly

This double-blind, placebo-controlled, randomized, multifactorial trial compared recovery of facial nerve function for patients randomized to receive 10 days of treatment with prednisolone (25 mg twice daily), acyclovir (400 mg 5 times daily), both agents, or placebo (lactose).

Inclusion criteria

Patients had to be at least 16 years of age (average age=44), with unilateral facial nerve weakness of no identifiable cause (eg, a diagnosis of Bell’s palsy). They were recruited mostly through their family doctors (75%) but also through emergency rooms and dental offices, and were referred to otolaryngologists at 17 Scottish hospitals within 72 hours.

The degree of initial facial paralysis was moderate to severe, based on the House-Brackmann scale, a widely used system for grading recovery from facial nerve paralysis. After the onset of symptoms, most patients (53.8%) initiated treatment within 24 hours, 32.1% within 48 hours, and 14.1% within 72 hours. Patients were assessed at baseline, 3 months, and 9 months.

Exclusion criteria

Exclusion criteria included pregnancy, breastfeeding, uncontrolled diabetes (Hb A1c >8.0%), peptic ulcer disease, suppurative otitis media, herpes zoster, multiple sclerosis, systemic infection, sarcoid or other rare disorder, and inability to give informed consent.

FIGURE
Facial weakness or paralysis may be permanent


Although most patients with Bell’s palsy recover completely, with or without treatment, 20% to 30% can have permanent facial weakness or paralysis.

Primary outcome: Complete recovery

The study was designed to test the effectiveness of prednisolone and acyclovir’s effects on facial nerve recovery. The House-Brackmann scale was used to score recovery. The scale divides patients into 1 of 6 categories depending on the severity of facial nerve dysfunction, with grade 1 describing normal function and grade 6 indicating total paralysis.

 

 

The scale was applied to photographs of patients taken while smiling, raising eyebrows, at rest, and closing eyes. The photographs were assessed and graded independently by 3 experts: an otolaryngologist, a neurologist, and a plastic surgeon. They were unaware of the study group assignment or stage of assessment.

Of 496 patients who completed the study, 357 recovered fully at 3 months, with no further treatment needed. Of the remaining patients, 80 had fully recovered at 9 months and 59 still had some facial-nerve deficit. At 3 months, there was a significant difference in recovery rates in prednisolone comparison groups: 83% with prednisolone vs 63.6% without prednisolone, a difference of 19.4 percentage points (95% confidence interval [CI], 11.7 to 27.1; P<.001, number needed to treat [NNT]=5). There was no significant difference in recovery rates in acyclovir comparison groups: 71.2% with acyclovir vs 75% not treated with acyclovir, a difference of 4.5 % percentage points (95% CI, –12.4 to 3.3; unadjusted P=.30; adjusted P=.50). At 9 months, the rates of complete recovery were 94.4% in prednisolone treated groups vs 81.6% in no prednisolone treatment groups (NNT=8) (TABLE).

TABLE
Complete recovery was significantly higher in the group that received prednisolone without an antiviral

 ACYCLOVIR ARMPLACEBO ARM
 ACYCLOVIR + PREDNISOLONEACYCLOVIR + PLACEBOPLACEBO + PREDNISOLONEPLACEBO + PLACEBO
Number of patients who completed therapy124123127122
% complete recovery* at 9 months92.7%78.0%96.1%85.2%
* Grade 1 on House-Brackmann scale, indicating normal function.
Source: Adapted from Sullivan et al.1
 

Adverse events

Adverse events included an expected range of minor symptoms associated with use of prednisolone and acyclovir, such as dizziness and vomiting. During the study, 3 patients died under circumstances unrelated to treatment: 2 were receiving double placebo and 1 received only acyclovir.1

WHAT’S NEW: A treatment based on patient-oriented evidence

Neither corticosteroids nor antivirals are new treatments for Bell’s palsy. What is new is that we know what works (corticosteroids) and what does not work (antivirals). This randomized controlled trial finally gives us the evidence on patient-oriented outcomes that we need to make confident recommendations, primarily because it enrolled twice as many patients as all trials compiled for the Cochrane systematic reviews on this topic.7,8

As an interesting side note, this is a good case study of how pathophysiologic reasoning sometimes leads us to good medical practice (corticosteroids in this case) and sometimes does not (antivirals in this case).

Isn’t it good to know that we can actually help patients with Bell’s palsy with corticosteroids and that antivirals are not necessary?

 

 

 

CAVEATS: Valacyclovir

Hato et al,9 in a Japanese study, showed that valacyclovir reaches a level of bioavailability that is 3 to 5 times more than acyclovir and may add some benefit to recovery when used in conjunction with prednisolone, particularly in more severe cases of Bell’s palsy.

The Hato study was a prospective, multicenter, randomized, placebo-controlled study that investigated the effects of valacyclovir (1000 mg/d for 5 days) and prednisolone in comparison with the effects of placebo and prednisolone for the treatment of Bell’s palsy.

The study outcomes included complete recovery from palsy; patients were followed until recovery occurred or more than 6 months in cases with severe prognosis. The patients in the Hato study had an average Yanagihara score of 15 when rating their facial palsy (which falls between House–Brackmann grades 4 and 5).

The overall rate of recovery of those treated with valacyclovir and prednisolone (96.5%) was significantly better (P<.05) than the rate among those treated with placebo and prednisolone (89.7%). In cases of complete or severe palsy, the rates of patients treated with both agents vs prednisolone alone who recovered were 95.7% (n=92) and 86.6% (n=82) (P<.05; NNT=11).

One big difference between the Sullivan and Hato studies is that the patients recruited for the Hato study had much more severe facial palsy (rated between 4 and 5) than in the Sullivan study (average=3.6), which suggests that there may be a use for valacyclovir in treating patients with complete facial palsy.6 Patients were all recruited from tertiary care centers as opposed to mainly from primary care settings as in the Sullivan study, consistent with the greater severity of cases in the Hato study.

Outcome assessors were not blinded to treatment assignments or stage of assessment in the Hato study, raising major concerns about the validity of the findings given the nature of facial paralysis as an outcome measure. We find the Sullivan study a more rigorous and convincing study. Nonetheless, future research may verify their findings and support the use of valacyclovir in the most severe cases of Bell’s palsy. For now, we are not convinced.

CHALLENGES TO IMPLEMENTATION: Easy to put into practice

Thankfully, some changes in practice are easy to implement. This is one of them. For those who prefer to prescribe prednisone, the dose of prednisolone used in the study, 25 mg bid, is equivalent to 60 mg of prednisone.

PURL surveillance system methodology
This study was selected and evaluated using FPIN’s Priority Updates from the Research Literature (PURL) surveillance system methodology.

Files
References

1. Sullivan FM, Swan IR, Donnan PT, et al. Early treatment with prednisolone or acyclovir in Bell’s palsy. N Engl J Med 2007;357:1598-1607.

2. Esslen E. Investigations on the localization and pathogenesis of meato-labyrinthine facial palsies. In: Esslen E (ed): The Acute Facial Palsies. Berlin: Springer-Verlag; 1977:41-91.

3. Ramsey MJ, DerSimonian R, Holtel MR, Burgess LP. Corticosteroid treatment for idiopathic facial nerve paralysis: a meta-analysis. Laryngoscope 2000;110:335-341.

4. Grogan PM, Gronseth GS. Practice parameter: Steroids, acyclovir, and surgery for Bell’s palsy (an evidence-based review): report of the Quality Standards Subcommittee of the American Academy of Neurology. Neurology 2001;56:830-836.

5. Ronthal M. Bell’s palsy. UpToDate [online database]. Updated September 7, 2007. Available at: www.uptodate.com. Accessed on November 20, 2007.

6. Gilden DH, Tyler KL. Bell’s palsy—is glucocorticoid treatment enough? N Engl J Med 2007;357:1653-1655.

7. Salinas RA, Alvarez G, Alvarez MI, Ferreira J. Corticosteroids for Bell’s palsy (idiopathic facial paralysis) (Cochrane Review). Cochrane Database Syst Rev 2002;(1):CD001942.-

8. Allen D, Dunn L. Acyclovir or valacyclovir for Bell’s palsy (idiopathic facial paralysis). Cochrane Database Syst Rev 2004;(3):CD001869.-

9. Hato N, Yamada H, Kohno H, et al. Valacyclovir and prednisolone treatment for Bell’s palsy: a multicenter, randomized, placebo-controlled study. Otol Neurotol 2007;28:408-413.

Author and Disclosure Information

Lisa Vargish, MD, MS
Sarah-Anne Schumann, MD
Department of Family Medicine, The University of Chicago

PURLs EDITOR
Bernard Ewigman, MD, MSPH
Department of Family Medicine, The University of Chicago

Issue
The Journal of Family Practice - 57(1)
Publications
Topics
Page Number
22-25
Legacy Keywords
Bell's palsy;Bell;herpetic;facial;paralysis;herpes;viral;neurological;nervous system;steroids;corticosteroids;prednisolone;acyclovir;treatment
Sections
Files
Files
Author and Disclosure Information

Lisa Vargish, MD, MS
Sarah-Anne Schumann, MD
Department of Family Medicine, The University of Chicago

PURLs EDITOR
Bernard Ewigman, MD, MSPH
Department of Family Medicine, The University of Chicago

Author and Disclosure Information

Lisa Vargish, MD, MS
Sarah-Anne Schumann, MD
Department of Family Medicine, The University of Chicago

PURLs EDITOR
Bernard Ewigman, MD, MSPH
Department of Family Medicine, The University of Chicago

Practice changer

A 10-day course of corticosteroids (prednisolone 25 mg twice daily) started within 72 hours significantly improves the chances of complete recovery. There is no added benefit from acyclovir.1

Strength of recommendation

A: Based on a large, well-designed randomized controlled trial

Sullivan FM, Swan IR, Donnan PT, et al. Early treatment with prednisolone or acyclovir in Bell’s palsy. N Engl J Med 2007;357:1598–1607.

 

Illustrative case

A 45-year-old man presents to your outpatient clinic within 24 hours after onset of left-sided facial nerve weakness and inability to smile on one side of his face. He asks for a therapy to help improve his symptoms quickly, as his daughter is getting married in a few months, and he will be in the wedding pictures.

Is there a treatment that will hasten his complete recovery?

Background: Insufficient statistical power until now

Many of us treat patients with Bell’s palsy with both corticosteroids and antiviral medications, such as acyclovir or valacyclovir, largely on the basis of pathophysiologic reasoning, because we’ve had no clear guidance from outcome studies. Until now, outcome studies have had mixed findings, and have been inconclusive.2 Most outcome studies have lacked the statistical power to either detect or to rule out potential benefits convincingly. The study by Sullivan and colleagues is the first to have a sufficiently large study sample from which to draw more definitive conclusions based on patient-oriented outcomes.

Steroid plus antiviral makes sense, pathophysiologically

Corticosteroids are thought to decrease inflammation of the facial nerve during an episode of facial paralysis. Some have postulated that herpes simplex virus type I may be a cause of facial nerve paralysis, hence the treatment with antivirals.3

Most of our PURL surveillance system clinician reviewers said that they prescribe both corticosteroids and antivirals.

Guidelines: “probably, possibly”

For example, a report of the Quality Standards Subcommittee of the American Academy of Neurology concluded that benefit from both steroids and antivirals has not been well established in patients with Bell’s palsy. However, the report states that evidence suggests that steroids are safe and probably effective, while antivirals are also safe and possibly effective.4

In contrast, UpToDate suggests treating all patients seen within a week of symptom onset with corticosteroids (prednisone 60–80 mg daily) plus valacyclovir (1 g 3 times daily) for 1 week.5

CLINICAL CONTEXT: Quality of life, risk of permanent harm

Bell’s palsy, defined as an acute peripheral facial weakness of unknown cause, has an annual incidence of 20 to 32 per 100,000. Most patients recover completely, with or without treatment, but 20% to 30% can have permanent facial weakness or paralysis.

The time to resolution is a quality of life issue for those in whom disease does not resolve spontaneously.6

We think that this study provides convincing evidence that acyclovir is not indicated for Bell’s palsy and that corticosteroids are.

STUDY SUMMARY: 10-day treatment, starting promptly

This double-blind, placebo-controlled, randomized, multifactorial trial compared recovery of facial nerve function for patients randomized to receive 10 days of treatment with prednisolone (25 mg twice daily), acyclovir (400 mg 5 times daily), both agents, or placebo (lactose).

Inclusion criteria

Patients had to be at least 16 years of age (average age=44), with unilateral facial nerve weakness of no identifiable cause (eg, a diagnosis of Bell’s palsy). They were recruited mostly through their family doctors (75%) but also through emergency rooms and dental offices, and were referred to otolaryngologists at 17 Scottish hospitals within 72 hours.

The degree of initial facial paralysis was moderate to severe, based on the House-Brackmann scale, a widely used system for grading recovery from facial nerve paralysis. After the onset of symptoms, most patients (53.8%) initiated treatment within 24 hours, 32.1% within 48 hours, and 14.1% within 72 hours. Patients were assessed at baseline, 3 months, and 9 months.

Exclusion criteria

Exclusion criteria included pregnancy, breastfeeding, uncontrolled diabetes (Hb A1c >8.0%), peptic ulcer disease, suppurative otitis media, herpes zoster, multiple sclerosis, systemic infection, sarcoid or other rare disorder, and inability to give informed consent.

FIGURE
Facial weakness or paralysis may be permanent


Although most patients with Bell’s palsy recover completely, with or without treatment, 20% to 30% can have permanent facial weakness or paralysis.

Primary outcome: Complete recovery

The study was designed to test the effectiveness of prednisolone and acyclovir’s effects on facial nerve recovery. The House-Brackmann scale was used to score recovery. The scale divides patients into 1 of 6 categories depending on the severity of facial nerve dysfunction, with grade 1 describing normal function and grade 6 indicating total paralysis.

 

 

The scale was applied to photographs of patients taken while smiling, raising eyebrows, at rest, and closing eyes. The photographs were assessed and graded independently by 3 experts: an otolaryngologist, a neurologist, and a plastic surgeon. They were unaware of the study group assignment or stage of assessment.

Of 496 patients who completed the study, 357 recovered fully at 3 months, with no further treatment needed. Of the remaining patients, 80 had fully recovered at 9 months and 59 still had some facial-nerve deficit. At 3 months, there was a significant difference in recovery rates in prednisolone comparison groups: 83% with prednisolone vs 63.6% without prednisolone, a difference of 19.4 percentage points (95% confidence interval [CI], 11.7 to 27.1; P<.001, number needed to treat [NNT]=5). There was no significant difference in recovery rates in acyclovir comparison groups: 71.2% with acyclovir vs 75% not treated with acyclovir, a difference of 4.5 % percentage points (95% CI, –12.4 to 3.3; unadjusted P=.30; adjusted P=.50). At 9 months, the rates of complete recovery were 94.4% in prednisolone treated groups vs 81.6% in no prednisolone treatment groups (NNT=8) (TABLE).

TABLE
Complete recovery was significantly higher in the group that received prednisolone without an antiviral

 ACYCLOVIR ARMPLACEBO ARM
 ACYCLOVIR + PREDNISOLONEACYCLOVIR + PLACEBOPLACEBO + PREDNISOLONEPLACEBO + PLACEBO
Number of patients who completed therapy124123127122
% complete recovery* at 9 months92.7%78.0%96.1%85.2%
* Grade 1 on House-Brackmann scale, indicating normal function.
Source: Adapted from Sullivan et al.1
 

Adverse events

Adverse events included an expected range of minor symptoms associated with use of prednisolone and acyclovir, such as dizziness and vomiting. During the study, 3 patients died under circumstances unrelated to treatment: 2 were receiving double placebo and 1 received only acyclovir.1

WHAT’S NEW: A treatment based on patient-oriented evidence

Neither corticosteroids nor antivirals are new treatments for Bell’s palsy. What is new is that we know what works (corticosteroids) and what does not work (antivirals). This randomized controlled trial finally gives us the evidence on patient-oriented outcomes that we need to make confident recommendations, primarily because it enrolled twice as many patients as all trials compiled for the Cochrane systematic reviews on this topic.7,8

As an interesting side note, this is a good case study of how pathophysiologic reasoning sometimes leads us to good medical practice (corticosteroids in this case) and sometimes does not (antivirals in this case).

Isn’t it good to know that we can actually help patients with Bell’s palsy with corticosteroids and that antivirals are not necessary?

 

 

 

CAVEATS: Valacyclovir

Hato et al,9 in a Japanese study, showed that valacyclovir reaches a level of bioavailability that is 3 to 5 times more than acyclovir and may add some benefit to recovery when used in conjunction with prednisolone, particularly in more severe cases of Bell’s palsy.

The Hato study was a prospective, multicenter, randomized, placebo-controlled study that investigated the effects of valacyclovir (1000 mg/d for 5 days) and prednisolone in comparison with the effects of placebo and prednisolone for the treatment of Bell’s palsy.

The study outcomes included complete recovery from palsy; patients were followed until recovery occurred or more than 6 months in cases with severe prognosis. The patients in the Hato study had an average Yanagihara score of 15 when rating their facial palsy (which falls between House–Brackmann grades 4 and 5).

The overall rate of recovery of those treated with valacyclovir and prednisolone (96.5%) was significantly better (P<.05) than the rate among those treated with placebo and prednisolone (89.7%). In cases of complete or severe palsy, the rates of patients treated with both agents vs prednisolone alone who recovered were 95.7% (n=92) and 86.6% (n=82) (P<.05; NNT=11).

One big difference between the Sullivan and Hato studies is that the patients recruited for the Hato study had much more severe facial palsy (rated between 4 and 5) than in the Sullivan study (average=3.6), which suggests that there may be a use for valacyclovir in treating patients with complete facial palsy.6 Patients were all recruited from tertiary care centers as opposed to mainly from primary care settings as in the Sullivan study, consistent with the greater severity of cases in the Hato study.

Outcome assessors were not blinded to treatment assignments or stage of assessment in the Hato study, raising major concerns about the validity of the findings given the nature of facial paralysis as an outcome measure. We find the Sullivan study a more rigorous and convincing study. Nonetheless, future research may verify their findings and support the use of valacyclovir in the most severe cases of Bell’s palsy. For now, we are not convinced.

CHALLENGES TO IMPLEMENTATION: Easy to put into practice

Thankfully, some changes in practice are easy to implement. This is one of them. For those who prefer to prescribe prednisone, the dose of prednisolone used in the study, 25 mg bid, is equivalent to 60 mg of prednisone.

PURL surveillance system methodology
This study was selected and evaluated using FPIN’s Priority Updates from the Research Literature (PURL) surveillance system methodology.

Practice changer

A 10-day course of corticosteroids (prednisolone 25 mg twice daily) started within 72 hours significantly improves the chances of complete recovery. There is no added benefit from acyclovir.1

Strength of recommendation

A: Based on a large, well-designed randomized controlled trial

Sullivan FM, Swan IR, Donnan PT, et al. Early treatment with prednisolone or acyclovir in Bell’s palsy. N Engl J Med 2007;357:1598–1607.

 

Illustrative case

A 45-year-old man presents to your outpatient clinic within 24 hours after onset of left-sided facial nerve weakness and inability to smile on one side of his face. He asks for a therapy to help improve his symptoms quickly, as his daughter is getting married in a few months, and he will be in the wedding pictures.

Is there a treatment that will hasten his complete recovery?

Background: Insufficient statistical power until now

Many of us treat patients with Bell’s palsy with both corticosteroids and antiviral medications, such as acyclovir or valacyclovir, largely on the basis of pathophysiologic reasoning, because we’ve had no clear guidance from outcome studies. Until now, outcome studies have had mixed findings, and have been inconclusive.2 Most outcome studies have lacked the statistical power to either detect or to rule out potential benefits convincingly. The study by Sullivan and colleagues is the first to have a sufficiently large study sample from which to draw more definitive conclusions based on patient-oriented outcomes.

Steroid plus antiviral makes sense, pathophysiologically

Corticosteroids are thought to decrease inflammation of the facial nerve during an episode of facial paralysis. Some have postulated that herpes simplex virus type I may be a cause of facial nerve paralysis, hence the treatment with antivirals.3

Most of our PURL surveillance system clinician reviewers said that they prescribe both corticosteroids and antivirals.

Guidelines: “probably, possibly”

For example, a report of the Quality Standards Subcommittee of the American Academy of Neurology concluded that benefit from both steroids and antivirals has not been well established in patients with Bell’s palsy. However, the report states that evidence suggests that steroids are safe and probably effective, while antivirals are also safe and possibly effective.4

In contrast, UpToDate suggests treating all patients seen within a week of symptom onset with corticosteroids (prednisone 60–80 mg daily) plus valacyclovir (1 g 3 times daily) for 1 week.5

CLINICAL CONTEXT: Quality of life, risk of permanent harm

Bell’s palsy, defined as an acute peripheral facial weakness of unknown cause, has an annual incidence of 20 to 32 per 100,000. Most patients recover completely, with or without treatment, but 20% to 30% can have permanent facial weakness or paralysis.

The time to resolution is a quality of life issue for those in whom disease does not resolve spontaneously.6

We think that this study provides convincing evidence that acyclovir is not indicated for Bell’s palsy and that corticosteroids are.

STUDY SUMMARY: 10-day treatment, starting promptly

This double-blind, placebo-controlled, randomized, multifactorial trial compared recovery of facial nerve function for patients randomized to receive 10 days of treatment with prednisolone (25 mg twice daily), acyclovir (400 mg 5 times daily), both agents, or placebo (lactose).

Inclusion criteria

Patients had to be at least 16 years of age (average age=44), with unilateral facial nerve weakness of no identifiable cause (eg, a diagnosis of Bell’s palsy). They were recruited mostly through their family doctors (75%) but also through emergency rooms and dental offices, and were referred to otolaryngologists at 17 Scottish hospitals within 72 hours.

The degree of initial facial paralysis was moderate to severe, based on the House-Brackmann scale, a widely used system for grading recovery from facial nerve paralysis. After the onset of symptoms, most patients (53.8%) initiated treatment within 24 hours, 32.1% within 48 hours, and 14.1% within 72 hours. Patients were assessed at baseline, 3 months, and 9 months.

Exclusion criteria

Exclusion criteria included pregnancy, breastfeeding, uncontrolled diabetes (Hb A1c >8.0%), peptic ulcer disease, suppurative otitis media, herpes zoster, multiple sclerosis, systemic infection, sarcoid or other rare disorder, and inability to give informed consent.

FIGURE
Facial weakness or paralysis may be permanent


Although most patients with Bell’s palsy recover completely, with or without treatment, 20% to 30% can have permanent facial weakness or paralysis.

Primary outcome: Complete recovery

The study was designed to test the effectiveness of prednisolone and acyclovir’s effects on facial nerve recovery. The House-Brackmann scale was used to score recovery. The scale divides patients into 1 of 6 categories depending on the severity of facial nerve dysfunction, with grade 1 describing normal function and grade 6 indicating total paralysis.

 

 

The scale was applied to photographs of patients taken while smiling, raising eyebrows, at rest, and closing eyes. The photographs were assessed and graded independently by 3 experts: an otolaryngologist, a neurologist, and a plastic surgeon. They were unaware of the study group assignment or stage of assessment.

Of 496 patients who completed the study, 357 recovered fully at 3 months, with no further treatment needed. Of the remaining patients, 80 had fully recovered at 9 months and 59 still had some facial-nerve deficit. At 3 months, there was a significant difference in recovery rates in prednisolone comparison groups: 83% with prednisolone vs 63.6% without prednisolone, a difference of 19.4 percentage points (95% confidence interval [CI], 11.7 to 27.1; P<.001, number needed to treat [NNT]=5). There was no significant difference in recovery rates in acyclovir comparison groups: 71.2% with acyclovir vs 75% not treated with acyclovir, a difference of 4.5 % percentage points (95% CI, –12.4 to 3.3; unadjusted P=.30; adjusted P=.50). At 9 months, the rates of complete recovery were 94.4% in prednisolone treated groups vs 81.6% in no prednisolone treatment groups (NNT=8) (TABLE).

TABLE
Complete recovery was significantly higher in the group that received prednisolone without an antiviral

 ACYCLOVIR ARMPLACEBO ARM
 ACYCLOVIR + PREDNISOLONEACYCLOVIR + PLACEBOPLACEBO + PREDNISOLONEPLACEBO + PLACEBO
Number of patients who completed therapy124123127122
% complete recovery* at 9 months92.7%78.0%96.1%85.2%
* Grade 1 on House-Brackmann scale, indicating normal function.
Source: Adapted from Sullivan et al.1
 

Adverse events

Adverse events included an expected range of minor symptoms associated with use of prednisolone and acyclovir, such as dizziness and vomiting. During the study, 3 patients died under circumstances unrelated to treatment: 2 were receiving double placebo and 1 received only acyclovir.1

WHAT’S NEW: A treatment based on patient-oriented evidence

Neither corticosteroids nor antivirals are new treatments for Bell’s palsy. What is new is that we know what works (corticosteroids) and what does not work (antivirals). This randomized controlled trial finally gives us the evidence on patient-oriented outcomes that we need to make confident recommendations, primarily because it enrolled twice as many patients as all trials compiled for the Cochrane systematic reviews on this topic.7,8

As an interesting side note, this is a good case study of how pathophysiologic reasoning sometimes leads us to good medical practice (corticosteroids in this case) and sometimes does not (antivirals in this case).

Isn’t it good to know that we can actually help patients with Bell’s palsy with corticosteroids and that antivirals are not necessary?

 

 

 

CAVEATS: Valacyclovir

Hato et al,9 in a Japanese study, showed that valacyclovir reaches a level of bioavailability that is 3 to 5 times more than acyclovir and may add some benefit to recovery when used in conjunction with prednisolone, particularly in more severe cases of Bell’s palsy.

The Hato study was a prospective, multicenter, randomized, placebo-controlled study that investigated the effects of valacyclovir (1000 mg/d for 5 days) and prednisolone in comparison with the effects of placebo and prednisolone for the treatment of Bell’s palsy.

The study outcomes included complete recovery from palsy; patients were followed until recovery occurred or more than 6 months in cases with severe prognosis. The patients in the Hato study had an average Yanagihara score of 15 when rating their facial palsy (which falls between House–Brackmann grades 4 and 5).

The overall rate of recovery of those treated with valacyclovir and prednisolone (96.5%) was significantly better (P<.05) than the rate among those treated with placebo and prednisolone (89.7%). In cases of complete or severe palsy, the rates of patients treated with both agents vs prednisolone alone who recovered were 95.7% (n=92) and 86.6% (n=82) (P<.05; NNT=11).

One big difference between the Sullivan and Hato studies is that the patients recruited for the Hato study had much more severe facial palsy (rated between 4 and 5) than in the Sullivan study (average=3.6), which suggests that there may be a use for valacyclovir in treating patients with complete facial palsy.6 Patients were all recruited from tertiary care centers as opposed to mainly from primary care settings as in the Sullivan study, consistent with the greater severity of cases in the Hato study.

Outcome assessors were not blinded to treatment assignments or stage of assessment in the Hato study, raising major concerns about the validity of the findings given the nature of facial paralysis as an outcome measure. We find the Sullivan study a more rigorous and convincing study. Nonetheless, future research may verify their findings and support the use of valacyclovir in the most severe cases of Bell’s palsy. For now, we are not convinced.

CHALLENGES TO IMPLEMENTATION: Easy to put into practice

Thankfully, some changes in practice are easy to implement. This is one of them. For those who prefer to prescribe prednisone, the dose of prednisolone used in the study, 25 mg bid, is equivalent to 60 mg of prednisone.

PURL surveillance system methodology
This study was selected and evaluated using FPIN’s Priority Updates from the Research Literature (PURL) surveillance system methodology.

References

1. Sullivan FM, Swan IR, Donnan PT, et al. Early treatment with prednisolone or acyclovir in Bell’s palsy. N Engl J Med 2007;357:1598-1607.

2. Esslen E. Investigations on the localization and pathogenesis of meato-labyrinthine facial palsies. In: Esslen E (ed): The Acute Facial Palsies. Berlin: Springer-Verlag; 1977:41-91.

3. Ramsey MJ, DerSimonian R, Holtel MR, Burgess LP. Corticosteroid treatment for idiopathic facial nerve paralysis: a meta-analysis. Laryngoscope 2000;110:335-341.

4. Grogan PM, Gronseth GS. Practice parameter: Steroids, acyclovir, and surgery for Bell’s palsy (an evidence-based review): report of the Quality Standards Subcommittee of the American Academy of Neurology. Neurology 2001;56:830-836.

5. Ronthal M. Bell’s palsy. UpToDate [online database]. Updated September 7, 2007. Available at: www.uptodate.com. Accessed on November 20, 2007.

6. Gilden DH, Tyler KL. Bell’s palsy—is glucocorticoid treatment enough? N Engl J Med 2007;357:1653-1655.

7. Salinas RA, Alvarez G, Alvarez MI, Ferreira J. Corticosteroids for Bell’s palsy (idiopathic facial paralysis) (Cochrane Review). Cochrane Database Syst Rev 2002;(1):CD001942.-

8. Allen D, Dunn L. Acyclovir or valacyclovir for Bell’s palsy (idiopathic facial paralysis). Cochrane Database Syst Rev 2004;(3):CD001869.-

9. Hato N, Yamada H, Kohno H, et al. Valacyclovir and prednisolone treatment for Bell’s palsy: a multicenter, randomized, placebo-controlled study. Otol Neurotol 2007;28:408-413.

References

1. Sullivan FM, Swan IR, Donnan PT, et al. Early treatment with prednisolone or acyclovir in Bell’s palsy. N Engl J Med 2007;357:1598-1607.

2. Esslen E. Investigations on the localization and pathogenesis of meato-labyrinthine facial palsies. In: Esslen E (ed): The Acute Facial Palsies. Berlin: Springer-Verlag; 1977:41-91.

3. Ramsey MJ, DerSimonian R, Holtel MR, Burgess LP. Corticosteroid treatment for idiopathic facial nerve paralysis: a meta-analysis. Laryngoscope 2000;110:335-341.

4. Grogan PM, Gronseth GS. Practice parameter: Steroids, acyclovir, and surgery for Bell’s palsy (an evidence-based review): report of the Quality Standards Subcommittee of the American Academy of Neurology. Neurology 2001;56:830-836.

5. Ronthal M. Bell’s palsy. UpToDate [online database]. Updated September 7, 2007. Available at: www.uptodate.com. Accessed on November 20, 2007.

6. Gilden DH, Tyler KL. Bell’s palsy—is glucocorticoid treatment enough? N Engl J Med 2007;357:1653-1655.

7. Salinas RA, Alvarez G, Alvarez MI, Ferreira J. Corticosteroids for Bell’s palsy (idiopathic facial paralysis) (Cochrane Review). Cochrane Database Syst Rev 2002;(1):CD001942.-

8. Allen D, Dunn L. Acyclovir or valacyclovir for Bell’s palsy (idiopathic facial paralysis). Cochrane Database Syst Rev 2004;(3):CD001869.-

9. Hato N, Yamada H, Kohno H, et al. Valacyclovir and prednisolone treatment for Bell’s palsy: a multicenter, randomized, placebo-controlled study. Otol Neurotol 2007;28:408-413.

Issue
The Journal of Family Practice - 57(1)
Issue
The Journal of Family Practice - 57(1)
Page Number
22-25
Page Number
22-25
Publications
Publications
Topics
Article Type
Display Headline
For Bell’s palsy, start steroids early; no need for an antiviral
Display Headline
For Bell’s palsy, start steroids early; no need for an antiviral
Legacy Keywords
Bell's palsy;Bell;herpetic;facial;paralysis;herpes;viral;neurological;nervous system;steroids;corticosteroids;prednisolone;acyclovir;treatment
Legacy Keywords
Bell's palsy;Bell;herpetic;facial;paralysis;herpes;viral;neurological;nervous system;steroids;corticosteroids;prednisolone;acyclovir;treatment
Sections
PURLs Copyright

Copyright © 2008 The Family Physicians Inquiries Network.
All rights reserved.

Disallow All Ads
Alternative CME
Use ProPublica
Hide sidebar & use full width
render the right sidebar.
Conference Recap Checkbox
Not Conference Recap
Clinical Edge
Media Files

When not to use beta-blockers in seniors with hypertension

Article Type
Changed
Fri, 06/19/2020 - 11:38
Display Headline
When not to use beta-blockers in seniors with hypertension
Practice changer

Beta-blockers should not be used to treat hypertension in patients older than age 60 unless they have another compelling indication to use these agents, such as heart failure or ischemic heart disease.1,2

Strength of recommendation

A: Based on a well-done meta-analyses

Khan N, Mcalister FA. Re-examining the efficacy of beta-blockers for the treatment of hypertension: a meta-analysis. CMAJ 2006; 174:1737– 1742.1

Wiysonge CS, Bradley H, Mayosi BM, et al. Beta-blockers for hypertension. Cochrane Database Syst Rev 2007; (1):CD002003.2

 

Illustrative case

A 70-year-old man with newly diagnosed hypertension comes to your office. You don’t want to prescribe a diuretic due to his history of gout. He has no history of coronary artery disease or heart failure.

What is the best antihypertensive agent for him?

Background: Guidelines do not reflect new evidence

Guidelines for the use of beta-blockers in the elderly do not reflect current evidence.

JNC recommendations

The 2003 JNC 7 Report recommended the same antihypertensive medications for adults of all ages.3 (JNC 7 is the most recent report from the Joint National Committee on Prevention, Detection, Evaluation, and Treatment of High Blood Pressure.)

JNC 7 recommends thiazide diuretics for first-line treatment of hypertension, and recommends other drugs—including beta-blockers, calcium-channel blockers, angiotensin-converting enzyme (ACE) inhibitors, and angiotensin receptor blockers (ARBs)—for first-line therapy if a thiazide is contraindicated, or in combination with thiazides for higher initial blood pressure.

Compelling indications. Beta-blockers are recommended in the JNC 7 Report as first-line therapy in patients with “compelling indications” such as ischemic heart disease and heart failure.

Clinical context: Seniors taking beta-blockers to their detriment?

Many elderly patients are on beta-blockers, perhaps to their detriment. Treatment choices for hypertension can have an enormous impact on outcomes among older patients:

Two thirds of US adults 60 years of age and older have hypertension, mostly isolated systolic hypertension.4,5

Multiple studies, including the Systolic Hypertension in the Elderly Program and the Systolic Hypertension in Europe, have shown that lowering blood pressure with pharmacologic interventions in older patients can reduce the risk of cardiovascular events and possibly dementia.6

Beta-blockers have been a mainstay of hypertension treatment for many decades and we suspect continue to be widely used as first-line therapy in patients for whom the evidence now indicates they are inferior.

Heart failure and angina are indications for beta-blockers

New evidence does not alter the 2003 JNC 7 recommendations to use beta-blockers as first-line therapy in patients with “compelling indications” such as ischemic heart disease and heart failure.

Study summaries

Two well-done reviews of beta-blocker trials show that they are inferior for first-line hypertension treatment in the elderly who do not have heart failure or angina.

2007 Cochrane review

The 2007 Cochrane review2 analyzed randomized trials that compared beta-blockers for hypertension in adults 18 years of age and older to each of the other major classes of antihypertensives.

Conclusion. This meta-analysis showed a “relatively weak effect of beta-blockers to reduce stroke, and the absence of effect on coronary heart disease when compared with placebo or no treatment” and a “trend toward worse outcomes in comparison with calcium channel blockers, renin-angiotensin system inhibitors, and thiazide diuretics.”

This meta-analysis included all adults and did not make any conclusions based on age.

2006 CMAJ meta-analysis

The Kahn and McAlister meta-analysis1 pooled data from 21 randomized hyper-tension trials (including 6 placebo-controlled trials) that evaluated the efficacy of beta-blockers as first-line therapy for hypertension in preventing major cardiovascular outcomes (death, nonfatal MI, or nonfatal stroke).

The results were analyzed by age group: trials enrolling patients with a mean age of 60 years or older at baseline vs trials enrolling patients with a mean age of under 60 years.

Conclusion. They concluded that in trials comparing other antihypertensive medications with beta-blockers, all agents showed similar efficacy in younger patients, while in older patients, beta-blockers were associated with a higher risk of both composite events and strokes ( TABLE ).

TABLE
Adverse outcomes more likely in seniors taking a beta-blocker vs other antihypertensives1

ADVERSE OUTCOME PATIENTS UNDER AGE 60PATIENTS AGE 60 AND OVER
 ADVERSE OUTCOMES LESS LIKELY WITH A BETA-BLOCKERADVERSE OUTCOMES MORE LIKELY WITH A BETA-BLOCKER
Composite outcomes (death, stroke, or MI)RR=0.97 (95% CI, 0.88–1.07)RR=1.06 (95% CI, 1.01–1.1)
StrokeRR=0.99 (95% CI, 0.67–1.44)RR=1.18 (95% CI, 1.07–1.3)
RR, relative risk of adverse outcomes, in randomized clinical trials of hypertensive patients treated with beta-blockers, compared with other antihypertensive drugs.
 

 

 

What’s new?: The age distinction

These 2 meta-analyses1,2 help overturn a long-held belief about the value of beta-blockers for the treatment of hypertension. Beta-blockers may not be a good first-line choice for any hypertensive patient—and the evidence clearly shows they are not a good first-line choice for patients over 60 years old.

Two earlier systematic reviews did raise the concern about using beta-blockers as first-line treatment for hypertension (even when thiazides are not contraindicated).

The first systematic review to raise this concern was a 1998 study of 10 hypertension trials in more than 16,000 patients, ages 60 and older. This review showed that diuretics were superior to beta-blockers in reducing cardiovascular and all-cause mortality—which supports the JNC 7 recommendation to choose a thiazide diuretic as the first-line drug of choice.7

The second study, a meta-analysis published in 2005, also concluded that beta-blockers should no longer be considered first-line therapy for hypertension, due to a 16% increase in the relative risk of stroke compared with other agents. This meta-analysis, however, did not report outcomes by patient age.8

Beta-blockers are not 1st-line, even if thiazides are contraindicated

What is new about the Kahn and McAlister evidence is that beta-blockers should not be the first-line drug of choice even when thiazide diuretics are contraindicated. Their study included a larger number of trials (21 trials vs 13 in the 2005 meta-analysis), which allowed the investigators to examine outcomes in patients younger than 60 and in those 60 years and older.

Caveats: Continue beta-blockers for the right reasons

Patients over 60 with ischemic heart disease or heart failure should still be prescribed beta-blockers for heart failure and angina. Also, in older patients with hypertension who need multiple agents to control their blood pressure, a beta-blocker could be added as a third or fourth agent in addition to a diuretic, ACE inhibitor, ARB, or calcium-channel blocker. Metoprolol is a good choice, as it is inexpensive and proven to reduce mortality in patients with a history of MI or heart failure.

Atenolol may underperform

In a meta-analysis of 31 trials, Freemantle9 found that after MI, acebutolol, metoprolol, propranolol, and timolol significantly reduced mortality, while there was no mortality reduction with atenolol. Similarly, in heart failure, only bisoprolol, metoprolol, and carvedilol have evidence to support a reduction in mortality.10

Although atenolol is one of the most commonly prescribed beta-blockers due to its low cost and once-daily dosing, it may be the least effective. In a systematic review of 9 hypertension studies, Carlberg11 showed that atenolol was no more effective than placebo at reducing MI, cardiovascular mortality, or all-cause mortality, and that patients on atenolol had significantly higher mortality than those taking other antihypertensives. Khan and McAlister do not differentiate between atenolol and other beta-blockers in their meta-analysis.1

Challenges to implementation: Letting go

The evidence supporting this change in practice has been accumulating over time. The change itself represents a significant reversal of long-standing belief in the value of beta-blockers as an antihypertensive agent. For each individual patient, the risk is not dramatic even though the cumulative “harm” from using a beta-blocker compared to other options is potentially staggering because so many people over 60 have hypertension.

We suspect that the main challenge will be changing the beliefs of both physicians and patients. Once doctors are convinced that beta-blockers are not indicated for uncomplicated hypertension in patients over 60, changing medications in the millions of older patients who have been taking a beta-blocker for some time and have become comfortable with it will take tact and excellent communication skills.

Providing patient information may help. Sources for patients are available free or at low cost at www.nhlbi.nih.gov/health/public/heart/index.htm#hbp. These materials explain that diuretics are inexpensive and are the preferred drugs for initial treatment of hypertension.

Cost comparison

In patients over 60 who can’t tolerate a thiazide, the least expensive option is an ACE inhibitor. For example, in the Target and Walmart discount generic programs, benazepril, captopril, enalapril, and lisinopril are all available for $4 per month.

PURLs methodology
This study was selected and evaluated using FPIN’s Priority Updates from the Research Literature (PURL) Surveillance System methodology. The criteria and findings leading to the selection of this study as a PURL can be accessed at www.jfponline.com/purls.

Files
References

1. Khan N, McAlister FA. Re-examining the efficacy of beta-blockers for the treatment of hypertension: a meta-analysis. CMAJ 2006;174:1737-1742.

2. Wiysonge CS, Bradley H, Mayosi BM, et al. Beta-blockers for hypertension. Cochrane Database Syst Rev 2007;(1):CD002003.-

3. The Seventh report of the Joint National Committee on Prevention, Detection, Evaluation, and Treatment of High Blood Pressure. US Department of Health and Human Services/National Heart, Lung and Blood Institute; 2003. Available at: www.nhlbi.nih.gov/guidelines/hypertension. Accessed on Dec 11, 2007.

4. Ostchega Y, Dillon CF, Hughes JP, Carroll M, Yoon S. Trends in hypertension prevalence, awareness, treatment, and control in older US adults: data from the National Health and Nutrition Examination Survey 1988 to 2004. J Am Geriatr Soc 2007;55:1056-1065.

5. Chobanian A. Isolated systolic hypertension in the elderly. N Engl J Med 2007;357:789-796.

6. Waeber B. Trials in isolated systolic hypertension: an update. Curr Hypertens Rep 2003;5:329-336.

7. Messerli FH, Grossman E, Goldbourt U. Are beta-blockers efficacious as first-line therapy for hypertension in the elderly? a systematic review. JAMA 1998;279:1903-1907.

8. Lindholm LH, Carlberg B, Samuelsson O. Should beta blockers remain first choice in the treatment of primary hypertension? A meta-analysis. Lancet 2005;366:1545-1553.

9. Freemantle N, Cleland J, Young P, Mason J, Harrison J. Beta blockade after myocardial infarction: systematic review and meta regression analysis. BMJ 1999;318:1730-1737.

10. Ong HT. Beta blockers in hypertension and cardiovascular disease. BMJ 2007;334:946-949.

11. Carlberg B, Samuelsson O, Lindholm LH. Atenolol in hypertension: is it a wise choice? Lancet 2004;364:1684-1689.

Article PDF
Author and Disclosure Information

Sarah-Anne Schumann, MD;
John Hickner, MD MSc
Department of Family Medicine, The University of Chicago

Issue
The Journal of Family Practice - 57(1)
Publications
Topics
Page Number
18-21
Legacy Keywords
hypertension;high blood pressure;cardiovascular;beta-blocker;b-blocker;aged;elderly;seniors;60;failure;ischemic;JNC 7;diuretic;angiotensin-converting;ACE;inhibitor;ARB;thiazide;calcium;channel;blocker;CCB;first-line;therapy
Sections
Files
Files
Author and Disclosure Information

Sarah-Anne Schumann, MD;
John Hickner, MD MSc
Department of Family Medicine, The University of Chicago

Author and Disclosure Information

Sarah-Anne Schumann, MD;
John Hickner, MD MSc
Department of Family Medicine, The University of Chicago

Article PDF
Article PDF
Practice changer

Beta-blockers should not be used to treat hypertension in patients older than age 60 unless they have another compelling indication to use these agents, such as heart failure or ischemic heart disease.1,2

Strength of recommendation

A: Based on a well-done meta-analyses

Khan N, Mcalister FA. Re-examining the efficacy of beta-blockers for the treatment of hypertension: a meta-analysis. CMAJ 2006; 174:1737– 1742.1

Wiysonge CS, Bradley H, Mayosi BM, et al. Beta-blockers for hypertension. Cochrane Database Syst Rev 2007; (1):CD002003.2

 

Illustrative case

A 70-year-old man with newly diagnosed hypertension comes to your office. You don’t want to prescribe a diuretic due to his history of gout. He has no history of coronary artery disease or heart failure.

What is the best antihypertensive agent for him?

Background: Guidelines do not reflect new evidence

Guidelines for the use of beta-blockers in the elderly do not reflect current evidence.

JNC recommendations

The 2003 JNC 7 Report recommended the same antihypertensive medications for adults of all ages.3 (JNC 7 is the most recent report from the Joint National Committee on Prevention, Detection, Evaluation, and Treatment of High Blood Pressure.)

JNC 7 recommends thiazide diuretics for first-line treatment of hypertension, and recommends other drugs—including beta-blockers, calcium-channel blockers, angiotensin-converting enzyme (ACE) inhibitors, and angiotensin receptor blockers (ARBs)—for first-line therapy if a thiazide is contraindicated, or in combination with thiazides for higher initial blood pressure.

Compelling indications. Beta-blockers are recommended in the JNC 7 Report as first-line therapy in patients with “compelling indications” such as ischemic heart disease and heart failure.

Clinical context: Seniors taking beta-blockers to their detriment?

Many elderly patients are on beta-blockers, perhaps to their detriment. Treatment choices for hypertension can have an enormous impact on outcomes among older patients:

Two thirds of US adults 60 years of age and older have hypertension, mostly isolated systolic hypertension.4,5

Multiple studies, including the Systolic Hypertension in the Elderly Program and the Systolic Hypertension in Europe, have shown that lowering blood pressure with pharmacologic interventions in older patients can reduce the risk of cardiovascular events and possibly dementia.6

Beta-blockers have been a mainstay of hypertension treatment for many decades and we suspect continue to be widely used as first-line therapy in patients for whom the evidence now indicates they are inferior.

Heart failure and angina are indications for beta-blockers

New evidence does not alter the 2003 JNC 7 recommendations to use beta-blockers as first-line therapy in patients with “compelling indications” such as ischemic heart disease and heart failure.

Study summaries

Two well-done reviews of beta-blocker trials show that they are inferior for first-line hypertension treatment in the elderly who do not have heart failure or angina.

2007 Cochrane review

The 2007 Cochrane review2 analyzed randomized trials that compared beta-blockers for hypertension in adults 18 years of age and older to each of the other major classes of antihypertensives.

Conclusion. This meta-analysis showed a “relatively weak effect of beta-blockers to reduce stroke, and the absence of effect on coronary heart disease when compared with placebo or no treatment” and a “trend toward worse outcomes in comparison with calcium channel blockers, renin-angiotensin system inhibitors, and thiazide diuretics.”

This meta-analysis included all adults and did not make any conclusions based on age.

2006 CMAJ meta-analysis

The Kahn and McAlister meta-analysis1 pooled data from 21 randomized hyper-tension trials (including 6 placebo-controlled trials) that evaluated the efficacy of beta-blockers as first-line therapy for hypertension in preventing major cardiovascular outcomes (death, nonfatal MI, or nonfatal stroke).

The results were analyzed by age group: trials enrolling patients with a mean age of 60 years or older at baseline vs trials enrolling patients with a mean age of under 60 years.

Conclusion. They concluded that in trials comparing other antihypertensive medications with beta-blockers, all agents showed similar efficacy in younger patients, while in older patients, beta-blockers were associated with a higher risk of both composite events and strokes ( TABLE ).

TABLE
Adverse outcomes more likely in seniors taking a beta-blocker vs other antihypertensives1

ADVERSE OUTCOME PATIENTS UNDER AGE 60PATIENTS AGE 60 AND OVER
 ADVERSE OUTCOMES LESS LIKELY WITH A BETA-BLOCKERADVERSE OUTCOMES MORE LIKELY WITH A BETA-BLOCKER
Composite outcomes (death, stroke, or MI)RR=0.97 (95% CI, 0.88–1.07)RR=1.06 (95% CI, 1.01–1.1)
StrokeRR=0.99 (95% CI, 0.67–1.44)RR=1.18 (95% CI, 1.07–1.3)
RR, relative risk of adverse outcomes, in randomized clinical trials of hypertensive patients treated with beta-blockers, compared with other antihypertensive drugs.
 

 

 

What’s new?: The age distinction

These 2 meta-analyses1,2 help overturn a long-held belief about the value of beta-blockers for the treatment of hypertension. Beta-blockers may not be a good first-line choice for any hypertensive patient—and the evidence clearly shows they are not a good first-line choice for patients over 60 years old.

Two earlier systematic reviews did raise the concern about using beta-blockers as first-line treatment for hypertension (even when thiazides are not contraindicated).

The first systematic review to raise this concern was a 1998 study of 10 hypertension trials in more than 16,000 patients, ages 60 and older. This review showed that diuretics were superior to beta-blockers in reducing cardiovascular and all-cause mortality—which supports the JNC 7 recommendation to choose a thiazide diuretic as the first-line drug of choice.7

The second study, a meta-analysis published in 2005, also concluded that beta-blockers should no longer be considered first-line therapy for hypertension, due to a 16% increase in the relative risk of stroke compared with other agents. This meta-analysis, however, did not report outcomes by patient age.8

Beta-blockers are not 1st-line, even if thiazides are contraindicated

What is new about the Kahn and McAlister evidence is that beta-blockers should not be the first-line drug of choice even when thiazide diuretics are contraindicated. Their study included a larger number of trials (21 trials vs 13 in the 2005 meta-analysis), which allowed the investigators to examine outcomes in patients younger than 60 and in those 60 years and older.

Caveats: Continue beta-blockers for the right reasons

Patients over 60 with ischemic heart disease or heart failure should still be prescribed beta-blockers for heart failure and angina. Also, in older patients with hypertension who need multiple agents to control their blood pressure, a beta-blocker could be added as a third or fourth agent in addition to a diuretic, ACE inhibitor, ARB, or calcium-channel blocker. Metoprolol is a good choice, as it is inexpensive and proven to reduce mortality in patients with a history of MI or heart failure.

Atenolol may underperform

In a meta-analysis of 31 trials, Freemantle9 found that after MI, acebutolol, metoprolol, propranolol, and timolol significantly reduced mortality, while there was no mortality reduction with atenolol. Similarly, in heart failure, only bisoprolol, metoprolol, and carvedilol have evidence to support a reduction in mortality.10

Although atenolol is one of the most commonly prescribed beta-blockers due to its low cost and once-daily dosing, it may be the least effective. In a systematic review of 9 hypertension studies, Carlberg11 showed that atenolol was no more effective than placebo at reducing MI, cardiovascular mortality, or all-cause mortality, and that patients on atenolol had significantly higher mortality than those taking other antihypertensives. Khan and McAlister do not differentiate between atenolol and other beta-blockers in their meta-analysis.1

Challenges to implementation: Letting go

The evidence supporting this change in practice has been accumulating over time. The change itself represents a significant reversal of long-standing belief in the value of beta-blockers as an antihypertensive agent. For each individual patient, the risk is not dramatic even though the cumulative “harm” from using a beta-blocker compared to other options is potentially staggering because so many people over 60 have hypertension.

We suspect that the main challenge will be changing the beliefs of both physicians and patients. Once doctors are convinced that beta-blockers are not indicated for uncomplicated hypertension in patients over 60, changing medications in the millions of older patients who have been taking a beta-blocker for some time and have become comfortable with it will take tact and excellent communication skills.

Providing patient information may help. Sources for patients are available free or at low cost at www.nhlbi.nih.gov/health/public/heart/index.htm#hbp. These materials explain that diuretics are inexpensive and are the preferred drugs for initial treatment of hypertension.

Cost comparison

In patients over 60 who can’t tolerate a thiazide, the least expensive option is an ACE inhibitor. For example, in the Target and Walmart discount generic programs, benazepril, captopril, enalapril, and lisinopril are all available for $4 per month.

PURLs methodology
This study was selected and evaluated using FPIN’s Priority Updates from the Research Literature (PURL) Surveillance System methodology. The criteria and findings leading to the selection of this study as a PURL can be accessed at www.jfponline.com/purls.

Practice changer

Beta-blockers should not be used to treat hypertension in patients older than age 60 unless they have another compelling indication to use these agents, such as heart failure or ischemic heart disease.1,2

Strength of recommendation

A: Based on a well-done meta-analyses

Khan N, Mcalister FA. Re-examining the efficacy of beta-blockers for the treatment of hypertension: a meta-analysis. CMAJ 2006; 174:1737– 1742.1

Wiysonge CS, Bradley H, Mayosi BM, et al. Beta-blockers for hypertension. Cochrane Database Syst Rev 2007; (1):CD002003.2

 

Illustrative case

A 70-year-old man with newly diagnosed hypertension comes to your office. You don’t want to prescribe a diuretic due to his history of gout. He has no history of coronary artery disease or heart failure.

What is the best antihypertensive agent for him?

Background: Guidelines do not reflect new evidence

Guidelines for the use of beta-blockers in the elderly do not reflect current evidence.

JNC recommendations

The 2003 JNC 7 Report recommended the same antihypertensive medications for adults of all ages.3 (JNC 7 is the most recent report from the Joint National Committee on Prevention, Detection, Evaluation, and Treatment of High Blood Pressure.)

JNC 7 recommends thiazide diuretics for first-line treatment of hypertension, and recommends other drugs—including beta-blockers, calcium-channel blockers, angiotensin-converting enzyme (ACE) inhibitors, and angiotensin receptor blockers (ARBs)—for first-line therapy if a thiazide is contraindicated, or in combination with thiazides for higher initial blood pressure.

Compelling indications. Beta-blockers are recommended in the JNC 7 Report as first-line therapy in patients with “compelling indications” such as ischemic heart disease and heart failure.

Clinical context: Seniors taking beta-blockers to their detriment?

Many elderly patients are on beta-blockers, perhaps to their detriment. Treatment choices for hypertension can have an enormous impact on outcomes among older patients:

Two thirds of US adults 60 years of age and older have hypertension, mostly isolated systolic hypertension.4,5

Multiple studies, including the Systolic Hypertension in the Elderly Program and the Systolic Hypertension in Europe, have shown that lowering blood pressure with pharmacologic interventions in older patients can reduce the risk of cardiovascular events and possibly dementia.6

Beta-blockers have been a mainstay of hypertension treatment for many decades and we suspect continue to be widely used as first-line therapy in patients for whom the evidence now indicates they are inferior.

Heart failure and angina are indications for beta-blockers

New evidence does not alter the 2003 JNC 7 recommendations to use beta-blockers as first-line therapy in patients with “compelling indications” such as ischemic heart disease and heart failure.

Study summaries

Two well-done reviews of beta-blocker trials show that they are inferior for first-line hypertension treatment in the elderly who do not have heart failure or angina.

2007 Cochrane review

The 2007 Cochrane review2 analyzed randomized trials that compared beta-blockers for hypertension in adults 18 years of age and older to each of the other major classes of antihypertensives.

Conclusion. This meta-analysis showed a “relatively weak effect of beta-blockers to reduce stroke, and the absence of effect on coronary heart disease when compared with placebo or no treatment” and a “trend toward worse outcomes in comparison with calcium channel blockers, renin-angiotensin system inhibitors, and thiazide diuretics.”

This meta-analysis included all adults and did not make any conclusions based on age.

2006 CMAJ meta-analysis

The Kahn and McAlister meta-analysis1 pooled data from 21 randomized hyper-tension trials (including 6 placebo-controlled trials) that evaluated the efficacy of beta-blockers as first-line therapy for hypertension in preventing major cardiovascular outcomes (death, nonfatal MI, or nonfatal stroke).

The results were analyzed by age group: trials enrolling patients with a mean age of 60 years or older at baseline vs trials enrolling patients with a mean age of under 60 years.

Conclusion. They concluded that in trials comparing other antihypertensive medications with beta-blockers, all agents showed similar efficacy in younger patients, while in older patients, beta-blockers were associated with a higher risk of both composite events and strokes ( TABLE ).

TABLE
Adverse outcomes more likely in seniors taking a beta-blocker vs other antihypertensives1

ADVERSE OUTCOME PATIENTS UNDER AGE 60PATIENTS AGE 60 AND OVER
 ADVERSE OUTCOMES LESS LIKELY WITH A BETA-BLOCKERADVERSE OUTCOMES MORE LIKELY WITH A BETA-BLOCKER
Composite outcomes (death, stroke, or MI)RR=0.97 (95% CI, 0.88–1.07)RR=1.06 (95% CI, 1.01–1.1)
StrokeRR=0.99 (95% CI, 0.67–1.44)RR=1.18 (95% CI, 1.07–1.3)
RR, relative risk of adverse outcomes, in randomized clinical trials of hypertensive patients treated with beta-blockers, compared with other antihypertensive drugs.
 

 

 

What’s new?: The age distinction

These 2 meta-analyses1,2 help overturn a long-held belief about the value of beta-blockers for the treatment of hypertension. Beta-blockers may not be a good first-line choice for any hypertensive patient—and the evidence clearly shows they are not a good first-line choice for patients over 60 years old.

Two earlier systematic reviews did raise the concern about using beta-blockers as first-line treatment for hypertension (even when thiazides are not contraindicated).

The first systematic review to raise this concern was a 1998 study of 10 hypertension trials in more than 16,000 patients, ages 60 and older. This review showed that diuretics were superior to beta-blockers in reducing cardiovascular and all-cause mortality—which supports the JNC 7 recommendation to choose a thiazide diuretic as the first-line drug of choice.7

The second study, a meta-analysis published in 2005, also concluded that beta-blockers should no longer be considered first-line therapy for hypertension, due to a 16% increase in the relative risk of stroke compared with other agents. This meta-analysis, however, did not report outcomes by patient age.8

Beta-blockers are not 1st-line, even if thiazides are contraindicated

What is new about the Kahn and McAlister evidence is that beta-blockers should not be the first-line drug of choice even when thiazide diuretics are contraindicated. Their study included a larger number of trials (21 trials vs 13 in the 2005 meta-analysis), which allowed the investigators to examine outcomes in patients younger than 60 and in those 60 years and older.

Caveats: Continue beta-blockers for the right reasons

Patients over 60 with ischemic heart disease or heart failure should still be prescribed beta-blockers for heart failure and angina. Also, in older patients with hypertension who need multiple agents to control their blood pressure, a beta-blocker could be added as a third or fourth agent in addition to a diuretic, ACE inhibitor, ARB, or calcium-channel blocker. Metoprolol is a good choice, as it is inexpensive and proven to reduce mortality in patients with a history of MI or heart failure.

Atenolol may underperform

In a meta-analysis of 31 trials, Freemantle9 found that after MI, acebutolol, metoprolol, propranolol, and timolol significantly reduced mortality, while there was no mortality reduction with atenolol. Similarly, in heart failure, only bisoprolol, metoprolol, and carvedilol have evidence to support a reduction in mortality.10

Although atenolol is one of the most commonly prescribed beta-blockers due to its low cost and once-daily dosing, it may be the least effective. In a systematic review of 9 hypertension studies, Carlberg11 showed that atenolol was no more effective than placebo at reducing MI, cardiovascular mortality, or all-cause mortality, and that patients on atenolol had significantly higher mortality than those taking other antihypertensives. Khan and McAlister do not differentiate between atenolol and other beta-blockers in their meta-analysis.1

Challenges to implementation: Letting go

The evidence supporting this change in practice has been accumulating over time. The change itself represents a significant reversal of long-standing belief in the value of beta-blockers as an antihypertensive agent. For each individual patient, the risk is not dramatic even though the cumulative “harm” from using a beta-blocker compared to other options is potentially staggering because so many people over 60 have hypertension.

We suspect that the main challenge will be changing the beliefs of both physicians and patients. Once doctors are convinced that beta-blockers are not indicated for uncomplicated hypertension in patients over 60, changing medications in the millions of older patients who have been taking a beta-blocker for some time and have become comfortable with it will take tact and excellent communication skills.

Providing patient information may help. Sources for patients are available free or at low cost at www.nhlbi.nih.gov/health/public/heart/index.htm#hbp. These materials explain that diuretics are inexpensive and are the preferred drugs for initial treatment of hypertension.

Cost comparison

In patients over 60 who can’t tolerate a thiazide, the least expensive option is an ACE inhibitor. For example, in the Target and Walmart discount generic programs, benazepril, captopril, enalapril, and lisinopril are all available for $4 per month.

PURLs methodology
This study was selected and evaluated using FPIN’s Priority Updates from the Research Literature (PURL) Surveillance System methodology. The criteria and findings leading to the selection of this study as a PURL can be accessed at www.jfponline.com/purls.

References

1. Khan N, McAlister FA. Re-examining the efficacy of beta-blockers for the treatment of hypertension: a meta-analysis. CMAJ 2006;174:1737-1742.

2. Wiysonge CS, Bradley H, Mayosi BM, et al. Beta-blockers for hypertension. Cochrane Database Syst Rev 2007;(1):CD002003.-

3. The Seventh report of the Joint National Committee on Prevention, Detection, Evaluation, and Treatment of High Blood Pressure. US Department of Health and Human Services/National Heart, Lung and Blood Institute; 2003. Available at: www.nhlbi.nih.gov/guidelines/hypertension. Accessed on Dec 11, 2007.

4. Ostchega Y, Dillon CF, Hughes JP, Carroll M, Yoon S. Trends in hypertension prevalence, awareness, treatment, and control in older US adults: data from the National Health and Nutrition Examination Survey 1988 to 2004. J Am Geriatr Soc 2007;55:1056-1065.

5. Chobanian A. Isolated systolic hypertension in the elderly. N Engl J Med 2007;357:789-796.

6. Waeber B. Trials in isolated systolic hypertension: an update. Curr Hypertens Rep 2003;5:329-336.

7. Messerli FH, Grossman E, Goldbourt U. Are beta-blockers efficacious as first-line therapy for hypertension in the elderly? a systematic review. JAMA 1998;279:1903-1907.

8. Lindholm LH, Carlberg B, Samuelsson O. Should beta blockers remain first choice in the treatment of primary hypertension? A meta-analysis. Lancet 2005;366:1545-1553.

9. Freemantle N, Cleland J, Young P, Mason J, Harrison J. Beta blockade after myocardial infarction: systematic review and meta regression analysis. BMJ 1999;318:1730-1737.

10. Ong HT. Beta blockers in hypertension and cardiovascular disease. BMJ 2007;334:946-949.

11. Carlberg B, Samuelsson O, Lindholm LH. Atenolol in hypertension: is it a wise choice? Lancet 2004;364:1684-1689.

References

1. Khan N, McAlister FA. Re-examining the efficacy of beta-blockers for the treatment of hypertension: a meta-analysis. CMAJ 2006;174:1737-1742.

2. Wiysonge CS, Bradley H, Mayosi BM, et al. Beta-blockers for hypertension. Cochrane Database Syst Rev 2007;(1):CD002003.-

3. The Seventh report of the Joint National Committee on Prevention, Detection, Evaluation, and Treatment of High Blood Pressure. US Department of Health and Human Services/National Heart, Lung and Blood Institute; 2003. Available at: www.nhlbi.nih.gov/guidelines/hypertension. Accessed on Dec 11, 2007.

4. Ostchega Y, Dillon CF, Hughes JP, Carroll M, Yoon S. Trends in hypertension prevalence, awareness, treatment, and control in older US adults: data from the National Health and Nutrition Examination Survey 1988 to 2004. J Am Geriatr Soc 2007;55:1056-1065.

5. Chobanian A. Isolated systolic hypertension in the elderly. N Engl J Med 2007;357:789-796.

6. Waeber B. Trials in isolated systolic hypertension: an update. Curr Hypertens Rep 2003;5:329-336.

7. Messerli FH, Grossman E, Goldbourt U. Are beta-blockers efficacious as first-line therapy for hypertension in the elderly? a systematic review. JAMA 1998;279:1903-1907.

8. Lindholm LH, Carlberg B, Samuelsson O. Should beta blockers remain first choice in the treatment of primary hypertension? A meta-analysis. Lancet 2005;366:1545-1553.

9. Freemantle N, Cleland J, Young P, Mason J, Harrison J. Beta blockade after myocardial infarction: systematic review and meta regression analysis. BMJ 1999;318:1730-1737.

10. Ong HT. Beta blockers in hypertension and cardiovascular disease. BMJ 2007;334:946-949.

11. Carlberg B, Samuelsson O, Lindholm LH. Atenolol in hypertension: is it a wise choice? Lancet 2004;364:1684-1689.

Issue
The Journal of Family Practice - 57(1)
Issue
The Journal of Family Practice - 57(1)
Page Number
18-21
Page Number
18-21
Publications
Publications
Topics
Article Type
Display Headline
When not to use beta-blockers in seniors with hypertension
Display Headline
When not to use beta-blockers in seniors with hypertension
Legacy Keywords
hypertension;high blood pressure;cardiovascular;beta-blocker;b-blocker;aged;elderly;seniors;60;failure;ischemic;JNC 7;diuretic;angiotensin-converting;ACE;inhibitor;ARB;thiazide;calcium;channel;blocker;CCB;first-line;therapy
Legacy Keywords
hypertension;high blood pressure;cardiovascular;beta-blocker;b-blocker;aged;elderly;seniors;60;failure;ischemic;JNC 7;diuretic;angiotensin-converting;ACE;inhibitor;ARB;thiazide;calcium;channel;blocker;CCB;first-line;therapy
Sections
PURLs Copyright

Copyright © 2008 The Family Physicians Inquiries Network.
All rights reserved.

Disallow All Ads
Alternative CME
Use ProPublica
Hide sidebar & use full width
render the right sidebar.
Conference Recap Checkbox
Not Conference Recap
Clinical Edge
Article PDF Media
Media Files

Annual zoledronic acid infusion lowers risk of fracture, death

Article Type
Changed
Mon, 01/14/2019 - 11:37
Display Headline
Annual zoledronic acid infusion lowers risk of fracture, death
Practice changer

For patients with a recent hip fracture, intravenous zoledronic acid annually is an option for reducing the risk of new fractures and death.1

Strength of recommendation (SOR)

B: based on one well-designed randomized controlled trial

Lyles KW, Colon-Emeric CS, Magaziner JF, et al. Zoledronic acid and clinical fractures and mortality after hip fracture. N Engl J Med 2007;357: 1799–1809. Epub Sep 17.

 

Illustrative Case

A 75-year-old woman comes to see you 1 month after she had surgery to repair a hip fracture. She was diagnosed with osteoporosis 3 years prior to the hip fracture and is currently taking calcium and vitamin D. She tried taking an oral bisphosphonate but couldn’t tolerate the gastrointestinal side effects. What treatment can you recommend to reduce her risk of sustaining another fracture?

Background: First fracture heightens risk

Patients with a prior hip fracture have 2.5 times the risk of a new fracture compared to age-matched persons without a previous hip fracture.2 Women who have hip fractures are 3 times more likely to die in the first 6 months after the fracture than women of the same age and health status without fractures.3 Ten million people in the US have osteoporosis and 300,000 per year suffer hip fracture.4

Guidelines from the National Osteoporosis Foundation (NOF) and the Institute for Clinical Systems Improvement (ICSI) include these recommendations for hip fracture patients: discuss adequacy of total calcium and vitamin D intake; address home safety and falls prevention; and encourage specific exercises for muscle strength. They also recommend treating all patients with a prior hip or vertebral fracture with an antiresorptive agent. Options include oral bisphosphonates (alendronate, ibandronate, or risedronate), calcitonin intranasal spray or subcutaneous calcitonin, hormone therapy, parathyroid hormone, and raloxifene.5,6

Clinical context: Are we doing our best?

Most patients with hip fracture are not properly evaluated or treated for osteoporosis. A 2002 study of 500 hip fracture patients treated at 4 Midwestern health systems found that only 12% to 24% of patients had a DXA (dual-energy x-ray absorptiometry) scan either before or after hip fracture, 5% to 27% of the patients received documented advice to take adequate calcium and vitamin D, and 5% to 37% received a prescription for any antiresorptive medication (bisphosphonate [2% to 10%], estrogen, calcitonin, or raloxifene).7

Bisphosphonates are effective but compliance is poor

Bisphosphonates are effective in preventing recurrence of hip fracture. One cohort study that included over 35,000 women over age 45 who had received a bisphosphonate prescription showed that patients who are adherent to treatment have a 44.5% relative risk reduction over 2 years and an absolute risk reduction of 0.8%, for an NNT of 125.

However, compliance with oral bisphosphonate therapy is poor; only 20% of the women in this study persisted with the therapy for 24 months.8 Patients must take these medications first thing in the morning with 8 ounces of water and then remain upright for 30 to 60 minutes before eating or drinking. Gastrointestinal side effects, including dyspepsia, nausea, and reflux disease, occur in about 25% of patients, and there is a small risk of developing gastric or duodenal ulcers.

Study summary

The HORIZON Recurrent Fracture Trial was an international, randomized, double blind, placebo-controlled trial of 2127 patients with a recent hip fracture.

  • The primary endpoint was a new clinical fracture.
  • Secondary endpoints included the change in bone mineral density in the non-fractured hip, new vertebral and hip fractures, and pre-specified safety endpoints, including death.

Patients. Women and men age 50 or older who had undergone a surgical repair of a minimal trauma hip fracture in the previous 90 days were eligible for the study. Ninety-one percent of the patients were white, 76% were female, and the mean age was 74.5 years. Forty-one percent of patients had a T score at the femoral neck of –2.5 or less at baseline (meeting diagnostic criteria for osteoporosis).

 

 

 

Method. Patients were randomized to receive either intravenous zoledronic acid 5 mg or placebo within 90 days of surgical repair of a hip fracture and yearly thereafter for the duration of the study. Patients with serum 25-hydroxyvitamin D levels <15 ng/mL received a loading dose of vitamin D (50,000–125,000 IU) 14 days before the first dose of the study drug. All patients were given daily calcium (1000–1500 mg) and vitamin D (800–1200 IU) supplements. Simultaneous treatment with nasal calcitonin, selective estrogen-receptor modulators, hormone replacement, tobolone, and external hip protectors was permitted “at the discretion of the investigators,” and 10.5% of the study patients did receive one of these other osteoporosis therapies.

Patients were followed for up to 5 years. Bone mineral density was tested by DXA at the hip and femoral neck at baseline and then annually. The median follow-up time was 1.9 years; 71.3% of patients completing the trial, and 3% of patients were lost to follow-up.

Results. The patients assigned to zoledronic acid had a 5.3% absolute risk reduction for new clinical fractures, yielding an NNT of 19 over 2 years to prevent one new clinical fracture ( TABLE 1 ). Bone mineral density of the contralateral hip increased in the zoledronic acid group by 2.6% after 1 year, 4.7% after 2 years, and 5.5% after 3 years and declined in the placebo group by 1.0%, 0.7%, and 0.9% respectively.

There was a 3.7% absolute risk reduction of death, with an NNT of 27 for 2 years to prevent one death.

Adverse events that were more common in the zoledronic acid group included fever (8.7% vs 3.1%) and musculoskeletal pain (3.1% vs. 1.2%). There were no reported cases of jaw osteonecrosis in either group and no statistically significant delay in the healing of fractures with zoledronic acid. Both groups had similar rates of renal and cardiovascular events, including atrial fibrillation and stroke.

Novartis provided funding for the study. An independent data and safety monitoring board oversaw the conduct and safety of the study and recommended that it be stopped early after having surpassed the pre-specified efficacy boundaries. Independent statisticians confirmed the data analysis that was performed by the sponsor.1

TABLE 1
The HORIZON study: Rates of fracture and death were lower in the zoledronic group compared to placebo1

OUTCOMEPLACEBO (N=1062)ZOLEDRONIC ACID (N=1065)HAZARD RATIO (95% CI)P VALUE
Any fracture139 (13.9%)92 (8.6%)0.65 (0.50-0.84).001
Hip fracture33 (3.5%)23 (2.0%)0.70 (0.41-1.19).18
Vertebral fracture39 (3.8%)21 (1.7%)0.54 (0.32–0.92).02
Death141 (13.3%)101 (9.6%)0.72 (0.56-0.93).01
Rates of fracture were calculated by Kaplan-Meier methods at 24 months and are not simple percentages

What’s new: The adherence advantage

The obvious advantage of zoledronic acid over other bisphosphonates is the high level of adherence that is possible under the controlled environment of a once yearly infusion administered under medical supervision. Considering the low rates of adherence to oral bisphosphonates, this is a significant medical advance.

This study shows that a yearly infusion of zoledronic acid is highly effective in preventing subsequent clinical fractures in patients who have recently suffered a hip fracture. It is the first randomized-controlled trial of a bisphosphonate for secondary prevention in patients with recent hip fracture, regardless of their bone mineral density status.

In a previous 3-year randomized controlled trial of 5mg zoledronic acid once yearly vs placebo for postmenopausal women with osteoporosis, the risk of vertebral fractures was reduced by 70% (3.3% vs 10.9% placebo) and the risk of hip fracture was reduced by 41% (1.4% vs 2.5% placebo).9

The NNT of 19 for 2 years to prevent one clinical fracture and NNT of 27 for 2 years to prevent one death strikes us as a good bargain compared to many modern medical interventions. Are these results too good to be true? We don’t think so.

No serious design flaws

This was a well done trial with no serious flaws in design. The number of deaths, however, was relatively small, so the NNT may be as high as 50 by our rough calculations. As a “worst case” for the benefit, however, this still seems worthwhile.

Caveats

Patients with uncorrected hypocalcemia or creatinine clearance <35 mL/min should not take bisphosphonates, including zoledronic acid. Although this study did not show any evidence of an increased risk of atrial fibrillation or report any cases of osteonecrosis of the jaw, providers should monitor patients for these potential adverse events.

 

 

 

Challenges to implementation: $1200 per dose

The FDA approved Reclast (zoledronic acid 5 mg) as a once-yearly treatment for postmenopausal osteoporosis and Paget’s disease in August 2007. (Zometa is the brand name for zoledronic acid 4 mg that is indicated for multiple myeloma, bone metastasis, and hypercalcemia of malignancy.) Medicare and most insurance plans will reimburse Reclast infusion for these FDA-approved indications when billed by a provider using a CPT code. It is administered intravenously over 15-minutes and there are no risks beyond those associated with local infiltration.

The greatest barrier to implementing this practice for solo physicians or small group practices will likely be the up front expense of buying the drug; one dose is approximately $1200. Patients can be referred to larger practices or hospitals with the capital to have zoledronic acid on hand and the capability of providing the infusion.

The cost is comparable to the annual cost of oral bisphosphonates and less than the cost of the other IV bisphosphonate (ibandronate), which is administered every 3 months ( TABLE 2 ).

TABLE 2
Bisphosphonates for osteoporosis: Routes, dosage, and cost

GENERIC NAMEBRAND NAMEROUTE OF ADMINISTRATIONDOSE AND FREQUENCYAPPROXIMATE ANNUAL COST
AlendronateFosamaxOral10 mg/d or 70 mg/week$960–$1120
IbandronateBonivaOral or IV2.5 mg/d, 150 mg monthly (PO) or 3 mg IV every 3 months$1140–$1200 PO, $1980 IV
RisedronateActonelOral5 mg/d or 35 mg/week$1000–$1100
Zoledronic acidReclastIV5 mg once a year$1200

PURLS methodology

This study was selected and evaluated using FPIN’s Priority Updates from the Research Literature Surveillance System methodology. The criteria and findings leading to the selection of this study as a PURL can be accessed at here.

Files
References

1. Lyles KW, Colon-Emeric CS, Magaziner JF, et al. Zoledronic acid and clinical fractures and mortality after hip fracture. N Engl J Med 2007;357:1799-1809.Epub Sept 17.

2. Colon-Emeric C, Kuchibhatla M, Pieper C, et al. The contribution of hip fracture to risk of subsequent fractures: data from two longitudinal studies. Osteoporosis Int 2003;14:879-883.

3. Empana JP, Dargent-Molina P, Breart G, et al. Effect of hip fracture on mortality in elderly women: the EPIDOS prospective study. J Am Geriatr Soc 2004;52:685-690.

4. National Osteoporosis Foundation. About osteoporosis: fast facts. Available at: www.nof.org/osteoporosis/diseasefacts.htm. Accessed on October 4, 2007.

5. National Osteoporosis Foundation. Physician’s guide to prevention and treatment of osteoporosis. Washington, DC: National Osteoporosis Foundation; 2005.

6. Institute for Clinical Systems Improvement (ICSI). Diagnosis and treatment of osteoporosis. Bloomington (MN): Institute for Clinical Systems Improvement; 2006.

7. Harrington JT, Broy SB, Derosa AM, et al. Hip fracture patients are not treated for osteoporosis: a call to action. Arthritis Rheum 2002;47:651-654.

8. Siris ES, Harris ST, Rosen CJ, et al. Adherence to bisphosphonate therapy and fracture rates in osteoporotic women: relationship to vertebral and nonvertebral fractures from 2 US claims databases. Mayo Clin Proc 2006;81:1013-1022.

9. Black DM, Delmas PD, Eastell R, et al. Once-yearly zoledronic acid for treatment of postmenopausal osteoporosis. N Engl J Med 2007;356:1809-1822.

Author and Disclosure Information

Sarah-Anne Schumann, MD;
John Hickner, MD, MSc
Department of Family Medicine, The University of Chicago

Issue
The Journal of Family Practice - 56(12)
Publications
Topics
Page Number
1013-1016
Legacy Keywords
bone;bones;fracture;osteoporosis;zoledronic;acid,treatment;Lyles;NEJM;annual;risk
Sections
Files
Files
Author and Disclosure Information

Sarah-Anne Schumann, MD;
John Hickner, MD, MSc
Department of Family Medicine, The University of Chicago

Author and Disclosure Information

Sarah-Anne Schumann, MD;
John Hickner, MD, MSc
Department of Family Medicine, The University of Chicago

Practice changer

For patients with a recent hip fracture, intravenous zoledronic acid annually is an option for reducing the risk of new fractures and death.1

Strength of recommendation (SOR)

B: based on one well-designed randomized controlled trial

Lyles KW, Colon-Emeric CS, Magaziner JF, et al. Zoledronic acid and clinical fractures and mortality after hip fracture. N Engl J Med 2007;357: 1799–1809. Epub Sep 17.

 

Illustrative Case

A 75-year-old woman comes to see you 1 month after she had surgery to repair a hip fracture. She was diagnosed with osteoporosis 3 years prior to the hip fracture and is currently taking calcium and vitamin D. She tried taking an oral bisphosphonate but couldn’t tolerate the gastrointestinal side effects. What treatment can you recommend to reduce her risk of sustaining another fracture?

Background: First fracture heightens risk

Patients with a prior hip fracture have 2.5 times the risk of a new fracture compared to age-matched persons without a previous hip fracture.2 Women who have hip fractures are 3 times more likely to die in the first 6 months after the fracture than women of the same age and health status without fractures.3 Ten million people in the US have osteoporosis and 300,000 per year suffer hip fracture.4

Guidelines from the National Osteoporosis Foundation (NOF) and the Institute for Clinical Systems Improvement (ICSI) include these recommendations for hip fracture patients: discuss adequacy of total calcium and vitamin D intake; address home safety and falls prevention; and encourage specific exercises for muscle strength. They also recommend treating all patients with a prior hip or vertebral fracture with an antiresorptive agent. Options include oral bisphosphonates (alendronate, ibandronate, or risedronate), calcitonin intranasal spray or subcutaneous calcitonin, hormone therapy, parathyroid hormone, and raloxifene.5,6

Clinical context: Are we doing our best?

Most patients with hip fracture are not properly evaluated or treated for osteoporosis. A 2002 study of 500 hip fracture patients treated at 4 Midwestern health systems found that only 12% to 24% of patients had a DXA (dual-energy x-ray absorptiometry) scan either before or after hip fracture, 5% to 27% of the patients received documented advice to take adequate calcium and vitamin D, and 5% to 37% received a prescription for any antiresorptive medication (bisphosphonate [2% to 10%], estrogen, calcitonin, or raloxifene).7

Bisphosphonates are effective but compliance is poor

Bisphosphonates are effective in preventing recurrence of hip fracture. One cohort study that included over 35,000 women over age 45 who had received a bisphosphonate prescription showed that patients who are adherent to treatment have a 44.5% relative risk reduction over 2 years and an absolute risk reduction of 0.8%, for an NNT of 125.

However, compliance with oral bisphosphonate therapy is poor; only 20% of the women in this study persisted with the therapy for 24 months.8 Patients must take these medications first thing in the morning with 8 ounces of water and then remain upright for 30 to 60 minutes before eating or drinking. Gastrointestinal side effects, including dyspepsia, nausea, and reflux disease, occur in about 25% of patients, and there is a small risk of developing gastric or duodenal ulcers.

Study summary

The HORIZON Recurrent Fracture Trial was an international, randomized, double blind, placebo-controlled trial of 2127 patients with a recent hip fracture.

  • The primary endpoint was a new clinical fracture.
  • Secondary endpoints included the change in bone mineral density in the non-fractured hip, new vertebral and hip fractures, and pre-specified safety endpoints, including death.

Patients. Women and men age 50 or older who had undergone a surgical repair of a minimal trauma hip fracture in the previous 90 days were eligible for the study. Ninety-one percent of the patients were white, 76% were female, and the mean age was 74.5 years. Forty-one percent of patients had a T score at the femoral neck of –2.5 or less at baseline (meeting diagnostic criteria for osteoporosis).

 

 

 

Method. Patients were randomized to receive either intravenous zoledronic acid 5 mg or placebo within 90 days of surgical repair of a hip fracture and yearly thereafter for the duration of the study. Patients with serum 25-hydroxyvitamin D levels <15 ng/mL received a loading dose of vitamin D (50,000–125,000 IU) 14 days before the first dose of the study drug. All patients were given daily calcium (1000–1500 mg) and vitamin D (800–1200 IU) supplements. Simultaneous treatment with nasal calcitonin, selective estrogen-receptor modulators, hormone replacement, tobolone, and external hip protectors was permitted “at the discretion of the investigators,” and 10.5% of the study patients did receive one of these other osteoporosis therapies.

Patients were followed for up to 5 years. Bone mineral density was tested by DXA at the hip and femoral neck at baseline and then annually. The median follow-up time was 1.9 years; 71.3% of patients completing the trial, and 3% of patients were lost to follow-up.

Results. The patients assigned to zoledronic acid had a 5.3% absolute risk reduction for new clinical fractures, yielding an NNT of 19 over 2 years to prevent one new clinical fracture ( TABLE 1 ). Bone mineral density of the contralateral hip increased in the zoledronic acid group by 2.6% after 1 year, 4.7% after 2 years, and 5.5% after 3 years and declined in the placebo group by 1.0%, 0.7%, and 0.9% respectively.

There was a 3.7% absolute risk reduction of death, with an NNT of 27 for 2 years to prevent one death.

Adverse events that were more common in the zoledronic acid group included fever (8.7% vs 3.1%) and musculoskeletal pain (3.1% vs. 1.2%). There were no reported cases of jaw osteonecrosis in either group and no statistically significant delay in the healing of fractures with zoledronic acid. Both groups had similar rates of renal and cardiovascular events, including atrial fibrillation and stroke.

Novartis provided funding for the study. An independent data and safety monitoring board oversaw the conduct and safety of the study and recommended that it be stopped early after having surpassed the pre-specified efficacy boundaries. Independent statisticians confirmed the data analysis that was performed by the sponsor.1

TABLE 1
The HORIZON study: Rates of fracture and death were lower in the zoledronic group compared to placebo1

OUTCOMEPLACEBO (N=1062)ZOLEDRONIC ACID (N=1065)HAZARD RATIO (95% CI)P VALUE
Any fracture139 (13.9%)92 (8.6%)0.65 (0.50-0.84).001
Hip fracture33 (3.5%)23 (2.0%)0.70 (0.41-1.19).18
Vertebral fracture39 (3.8%)21 (1.7%)0.54 (0.32–0.92).02
Death141 (13.3%)101 (9.6%)0.72 (0.56-0.93).01
Rates of fracture were calculated by Kaplan-Meier methods at 24 months and are not simple percentages

What’s new: The adherence advantage

The obvious advantage of zoledronic acid over other bisphosphonates is the high level of adherence that is possible under the controlled environment of a once yearly infusion administered under medical supervision. Considering the low rates of adherence to oral bisphosphonates, this is a significant medical advance.

This study shows that a yearly infusion of zoledronic acid is highly effective in preventing subsequent clinical fractures in patients who have recently suffered a hip fracture. It is the first randomized-controlled trial of a bisphosphonate for secondary prevention in patients with recent hip fracture, regardless of their bone mineral density status.

In a previous 3-year randomized controlled trial of 5mg zoledronic acid once yearly vs placebo for postmenopausal women with osteoporosis, the risk of vertebral fractures was reduced by 70% (3.3% vs 10.9% placebo) and the risk of hip fracture was reduced by 41% (1.4% vs 2.5% placebo).9

The NNT of 19 for 2 years to prevent one clinical fracture and NNT of 27 for 2 years to prevent one death strikes us as a good bargain compared to many modern medical interventions. Are these results too good to be true? We don’t think so.

No serious design flaws

This was a well done trial with no serious flaws in design. The number of deaths, however, was relatively small, so the NNT may be as high as 50 by our rough calculations. As a “worst case” for the benefit, however, this still seems worthwhile.

Caveats

Patients with uncorrected hypocalcemia or creatinine clearance <35 mL/min should not take bisphosphonates, including zoledronic acid. Although this study did not show any evidence of an increased risk of atrial fibrillation or report any cases of osteonecrosis of the jaw, providers should monitor patients for these potential adverse events.

 

 

 

Challenges to implementation: $1200 per dose

The FDA approved Reclast (zoledronic acid 5 mg) as a once-yearly treatment for postmenopausal osteoporosis and Paget’s disease in August 2007. (Zometa is the brand name for zoledronic acid 4 mg that is indicated for multiple myeloma, bone metastasis, and hypercalcemia of malignancy.) Medicare and most insurance plans will reimburse Reclast infusion for these FDA-approved indications when billed by a provider using a CPT code. It is administered intravenously over 15-minutes and there are no risks beyond those associated with local infiltration.

The greatest barrier to implementing this practice for solo physicians or small group practices will likely be the up front expense of buying the drug; one dose is approximately $1200. Patients can be referred to larger practices or hospitals with the capital to have zoledronic acid on hand and the capability of providing the infusion.

The cost is comparable to the annual cost of oral bisphosphonates and less than the cost of the other IV bisphosphonate (ibandronate), which is administered every 3 months ( TABLE 2 ).

TABLE 2
Bisphosphonates for osteoporosis: Routes, dosage, and cost

GENERIC NAMEBRAND NAMEROUTE OF ADMINISTRATIONDOSE AND FREQUENCYAPPROXIMATE ANNUAL COST
AlendronateFosamaxOral10 mg/d or 70 mg/week$960–$1120
IbandronateBonivaOral or IV2.5 mg/d, 150 mg monthly (PO) or 3 mg IV every 3 months$1140–$1200 PO, $1980 IV
RisedronateActonelOral5 mg/d or 35 mg/week$1000–$1100
Zoledronic acidReclastIV5 mg once a year$1200

PURLS methodology

This study was selected and evaluated using FPIN’s Priority Updates from the Research Literature Surveillance System methodology. The criteria and findings leading to the selection of this study as a PURL can be accessed at here.

Practice changer

For patients with a recent hip fracture, intravenous zoledronic acid annually is an option for reducing the risk of new fractures and death.1

Strength of recommendation (SOR)

B: based on one well-designed randomized controlled trial

Lyles KW, Colon-Emeric CS, Magaziner JF, et al. Zoledronic acid and clinical fractures and mortality after hip fracture. N Engl J Med 2007;357: 1799–1809. Epub Sep 17.

 

Illustrative Case

A 75-year-old woman comes to see you 1 month after she had surgery to repair a hip fracture. She was diagnosed with osteoporosis 3 years prior to the hip fracture and is currently taking calcium and vitamin D. She tried taking an oral bisphosphonate but couldn’t tolerate the gastrointestinal side effects. What treatment can you recommend to reduce her risk of sustaining another fracture?

Background: First fracture heightens risk

Patients with a prior hip fracture have 2.5 times the risk of a new fracture compared to age-matched persons without a previous hip fracture.2 Women who have hip fractures are 3 times more likely to die in the first 6 months after the fracture than women of the same age and health status without fractures.3 Ten million people in the US have osteoporosis and 300,000 per year suffer hip fracture.4

Guidelines from the National Osteoporosis Foundation (NOF) and the Institute for Clinical Systems Improvement (ICSI) include these recommendations for hip fracture patients: discuss adequacy of total calcium and vitamin D intake; address home safety and falls prevention; and encourage specific exercises for muscle strength. They also recommend treating all patients with a prior hip or vertebral fracture with an antiresorptive agent. Options include oral bisphosphonates (alendronate, ibandronate, or risedronate), calcitonin intranasal spray or subcutaneous calcitonin, hormone therapy, parathyroid hormone, and raloxifene.5,6

Clinical context: Are we doing our best?

Most patients with hip fracture are not properly evaluated or treated for osteoporosis. A 2002 study of 500 hip fracture patients treated at 4 Midwestern health systems found that only 12% to 24% of patients had a DXA (dual-energy x-ray absorptiometry) scan either before or after hip fracture, 5% to 27% of the patients received documented advice to take adequate calcium and vitamin D, and 5% to 37% received a prescription for any antiresorptive medication (bisphosphonate [2% to 10%], estrogen, calcitonin, or raloxifene).7

Bisphosphonates are effective but compliance is poor

Bisphosphonates are effective in preventing recurrence of hip fracture. One cohort study that included over 35,000 women over age 45 who had received a bisphosphonate prescription showed that patients who are adherent to treatment have a 44.5% relative risk reduction over 2 years and an absolute risk reduction of 0.8%, for an NNT of 125.

However, compliance with oral bisphosphonate therapy is poor; only 20% of the women in this study persisted with the therapy for 24 months.8 Patients must take these medications first thing in the morning with 8 ounces of water and then remain upright for 30 to 60 minutes before eating or drinking. Gastrointestinal side effects, including dyspepsia, nausea, and reflux disease, occur in about 25% of patients, and there is a small risk of developing gastric or duodenal ulcers.

Study summary

The HORIZON Recurrent Fracture Trial was an international, randomized, double blind, placebo-controlled trial of 2127 patients with a recent hip fracture.

  • The primary endpoint was a new clinical fracture.
  • Secondary endpoints included the change in bone mineral density in the non-fractured hip, new vertebral and hip fractures, and pre-specified safety endpoints, including death.

Patients. Women and men age 50 or older who had undergone a surgical repair of a minimal trauma hip fracture in the previous 90 days were eligible for the study. Ninety-one percent of the patients were white, 76% were female, and the mean age was 74.5 years. Forty-one percent of patients had a T score at the femoral neck of –2.5 or less at baseline (meeting diagnostic criteria for osteoporosis).

 

 

 

Method. Patients were randomized to receive either intravenous zoledronic acid 5 mg or placebo within 90 days of surgical repair of a hip fracture and yearly thereafter for the duration of the study. Patients with serum 25-hydroxyvitamin D levels <15 ng/mL received a loading dose of vitamin D (50,000–125,000 IU) 14 days before the first dose of the study drug. All patients were given daily calcium (1000–1500 mg) and vitamin D (800–1200 IU) supplements. Simultaneous treatment with nasal calcitonin, selective estrogen-receptor modulators, hormone replacement, tobolone, and external hip protectors was permitted “at the discretion of the investigators,” and 10.5% of the study patients did receive one of these other osteoporosis therapies.

Patients were followed for up to 5 years. Bone mineral density was tested by DXA at the hip and femoral neck at baseline and then annually. The median follow-up time was 1.9 years; 71.3% of patients completing the trial, and 3% of patients were lost to follow-up.

Results. The patients assigned to zoledronic acid had a 5.3% absolute risk reduction for new clinical fractures, yielding an NNT of 19 over 2 years to prevent one new clinical fracture ( TABLE 1 ). Bone mineral density of the contralateral hip increased in the zoledronic acid group by 2.6% after 1 year, 4.7% after 2 years, and 5.5% after 3 years and declined in the placebo group by 1.0%, 0.7%, and 0.9% respectively.

There was a 3.7% absolute risk reduction of death, with an NNT of 27 for 2 years to prevent one death.

Adverse events that were more common in the zoledronic acid group included fever (8.7% vs 3.1%) and musculoskeletal pain (3.1% vs. 1.2%). There were no reported cases of jaw osteonecrosis in either group and no statistically significant delay in the healing of fractures with zoledronic acid. Both groups had similar rates of renal and cardiovascular events, including atrial fibrillation and stroke.

Novartis provided funding for the study. An independent data and safety monitoring board oversaw the conduct and safety of the study and recommended that it be stopped early after having surpassed the pre-specified efficacy boundaries. Independent statisticians confirmed the data analysis that was performed by the sponsor.1

TABLE 1
The HORIZON study: Rates of fracture and death were lower in the zoledronic group compared to placebo1

OUTCOMEPLACEBO (N=1062)ZOLEDRONIC ACID (N=1065)HAZARD RATIO (95% CI)P VALUE
Any fracture139 (13.9%)92 (8.6%)0.65 (0.50-0.84).001
Hip fracture33 (3.5%)23 (2.0%)0.70 (0.41-1.19).18
Vertebral fracture39 (3.8%)21 (1.7%)0.54 (0.32–0.92).02
Death141 (13.3%)101 (9.6%)0.72 (0.56-0.93).01
Rates of fracture were calculated by Kaplan-Meier methods at 24 months and are not simple percentages

What’s new: The adherence advantage

The obvious advantage of zoledronic acid over other bisphosphonates is the high level of adherence that is possible under the controlled environment of a once yearly infusion administered under medical supervision. Considering the low rates of adherence to oral bisphosphonates, this is a significant medical advance.

This study shows that a yearly infusion of zoledronic acid is highly effective in preventing subsequent clinical fractures in patients who have recently suffered a hip fracture. It is the first randomized-controlled trial of a bisphosphonate for secondary prevention in patients with recent hip fracture, regardless of their bone mineral density status.

In a previous 3-year randomized controlled trial of 5mg zoledronic acid once yearly vs placebo for postmenopausal women with osteoporosis, the risk of vertebral fractures was reduced by 70% (3.3% vs 10.9% placebo) and the risk of hip fracture was reduced by 41% (1.4% vs 2.5% placebo).9

The NNT of 19 for 2 years to prevent one clinical fracture and NNT of 27 for 2 years to prevent one death strikes us as a good bargain compared to many modern medical interventions. Are these results too good to be true? We don’t think so.

No serious design flaws

This was a well done trial with no serious flaws in design. The number of deaths, however, was relatively small, so the NNT may be as high as 50 by our rough calculations. As a “worst case” for the benefit, however, this still seems worthwhile.

Caveats

Patients with uncorrected hypocalcemia or creatinine clearance <35 mL/min should not take bisphosphonates, including zoledronic acid. Although this study did not show any evidence of an increased risk of atrial fibrillation or report any cases of osteonecrosis of the jaw, providers should monitor patients for these potential adverse events.

 

 

 

Challenges to implementation: $1200 per dose

The FDA approved Reclast (zoledronic acid 5 mg) as a once-yearly treatment for postmenopausal osteoporosis and Paget’s disease in August 2007. (Zometa is the brand name for zoledronic acid 4 mg that is indicated for multiple myeloma, bone metastasis, and hypercalcemia of malignancy.) Medicare and most insurance plans will reimburse Reclast infusion for these FDA-approved indications when billed by a provider using a CPT code. It is administered intravenously over 15-minutes and there are no risks beyond those associated with local infiltration.

The greatest barrier to implementing this practice for solo physicians or small group practices will likely be the up front expense of buying the drug; one dose is approximately $1200. Patients can be referred to larger practices or hospitals with the capital to have zoledronic acid on hand and the capability of providing the infusion.

The cost is comparable to the annual cost of oral bisphosphonates and less than the cost of the other IV bisphosphonate (ibandronate), which is administered every 3 months ( TABLE 2 ).

TABLE 2
Bisphosphonates for osteoporosis: Routes, dosage, and cost

GENERIC NAMEBRAND NAMEROUTE OF ADMINISTRATIONDOSE AND FREQUENCYAPPROXIMATE ANNUAL COST
AlendronateFosamaxOral10 mg/d or 70 mg/week$960–$1120
IbandronateBonivaOral or IV2.5 mg/d, 150 mg monthly (PO) or 3 mg IV every 3 months$1140–$1200 PO, $1980 IV
RisedronateActonelOral5 mg/d or 35 mg/week$1000–$1100
Zoledronic acidReclastIV5 mg once a year$1200

PURLS methodology

This study was selected and evaluated using FPIN’s Priority Updates from the Research Literature Surveillance System methodology. The criteria and findings leading to the selection of this study as a PURL can be accessed at here.

References

1. Lyles KW, Colon-Emeric CS, Magaziner JF, et al. Zoledronic acid and clinical fractures and mortality after hip fracture. N Engl J Med 2007;357:1799-1809.Epub Sept 17.

2. Colon-Emeric C, Kuchibhatla M, Pieper C, et al. The contribution of hip fracture to risk of subsequent fractures: data from two longitudinal studies. Osteoporosis Int 2003;14:879-883.

3. Empana JP, Dargent-Molina P, Breart G, et al. Effect of hip fracture on mortality in elderly women: the EPIDOS prospective study. J Am Geriatr Soc 2004;52:685-690.

4. National Osteoporosis Foundation. About osteoporosis: fast facts. Available at: www.nof.org/osteoporosis/diseasefacts.htm. Accessed on October 4, 2007.

5. National Osteoporosis Foundation. Physician’s guide to prevention and treatment of osteoporosis. Washington, DC: National Osteoporosis Foundation; 2005.

6. Institute for Clinical Systems Improvement (ICSI). Diagnosis and treatment of osteoporosis. Bloomington (MN): Institute for Clinical Systems Improvement; 2006.

7. Harrington JT, Broy SB, Derosa AM, et al. Hip fracture patients are not treated for osteoporosis: a call to action. Arthritis Rheum 2002;47:651-654.

8. Siris ES, Harris ST, Rosen CJ, et al. Adherence to bisphosphonate therapy and fracture rates in osteoporotic women: relationship to vertebral and nonvertebral fractures from 2 US claims databases. Mayo Clin Proc 2006;81:1013-1022.

9. Black DM, Delmas PD, Eastell R, et al. Once-yearly zoledronic acid for treatment of postmenopausal osteoporosis. N Engl J Med 2007;356:1809-1822.

References

1. Lyles KW, Colon-Emeric CS, Magaziner JF, et al. Zoledronic acid and clinical fractures and mortality after hip fracture. N Engl J Med 2007;357:1799-1809.Epub Sept 17.

2. Colon-Emeric C, Kuchibhatla M, Pieper C, et al. The contribution of hip fracture to risk of subsequent fractures: data from two longitudinal studies. Osteoporosis Int 2003;14:879-883.

3. Empana JP, Dargent-Molina P, Breart G, et al. Effect of hip fracture on mortality in elderly women: the EPIDOS prospective study. J Am Geriatr Soc 2004;52:685-690.

4. National Osteoporosis Foundation. About osteoporosis: fast facts. Available at: www.nof.org/osteoporosis/diseasefacts.htm. Accessed on October 4, 2007.

5. National Osteoporosis Foundation. Physician’s guide to prevention and treatment of osteoporosis. Washington, DC: National Osteoporosis Foundation; 2005.

6. Institute for Clinical Systems Improvement (ICSI). Diagnosis and treatment of osteoporosis. Bloomington (MN): Institute for Clinical Systems Improvement; 2006.

7. Harrington JT, Broy SB, Derosa AM, et al. Hip fracture patients are not treated for osteoporosis: a call to action. Arthritis Rheum 2002;47:651-654.

8. Siris ES, Harris ST, Rosen CJ, et al. Adherence to bisphosphonate therapy and fracture rates in osteoporotic women: relationship to vertebral and nonvertebral fractures from 2 US claims databases. Mayo Clin Proc 2006;81:1013-1022.

9. Black DM, Delmas PD, Eastell R, et al. Once-yearly zoledronic acid for treatment of postmenopausal osteoporosis. N Engl J Med 2007;356:1809-1822.

Issue
The Journal of Family Practice - 56(12)
Issue
The Journal of Family Practice - 56(12)
Page Number
1013-1016
Page Number
1013-1016
Publications
Publications
Topics
Article Type
Display Headline
Annual zoledronic acid infusion lowers risk of fracture, death
Display Headline
Annual zoledronic acid infusion lowers risk of fracture, death
Legacy Keywords
bone;bones;fracture;osteoporosis;zoledronic;acid,treatment;Lyles;NEJM;annual;risk
Legacy Keywords
bone;bones;fracture;osteoporosis;zoledronic;acid,treatment;Lyles;NEJM;annual;risk
Sections
PURLs Copyright

Copyright © 2007 The Family Physicians Inquiries Network.
All rights reserved.

Disallow All Ads
Alternative CME
Use ProPublica
Media Files